トップページ数学
1002コメント355KB
不等式への招待 第8章 [無断転載禁止]©2ch.net
レス数が1000を超えています。これ以上書き込みはできません。
0001不等式ヲタ ( ゚∀゚)
垢版 |
2017/06/25(日) 17:20:59.55ID:dLSgUfzK
ある人は蝶を集め、ある人は切手を収集し、ある人は不等式を集める…
          ___          ----- 参考文献 和書[3] P.65 -----
    |┃三 ./  ≧ \   
    |┃   |::::  \ ./ | 
    |┃ ≡|::::: (● (● |  不等式と聞ゐちゃぁ
____.|ミ\_ヽ::::... .ワ......ノ     黙っちゃゐられねゑ!
    |┃=__    \           ハァハァ…
    |┃ ≡ )  人 \ ガラッ

まとめWiki http://wiki.livedoor.jp/loveinequality/

過去スレ
・不等式スレッド (第1章)http://science3.2ch.net/test/read.cgi/math/1072510082/
・不等式への招待 第2章 http://science6.2ch.net/test/read.cgi/math/1105911616/
・不等式への招待 第3章 http://science6.2ch.net/test/read.cgi/math/1179000000/
・不等式への招待 第4章 http://science6.2ch.net/test/read.cgi/math/1245060000/
・不等式への招待 第5章 http://uni.2ch.net/test/read.cgi/math/1287932216/
・不等式への招待 第6章 http://uni.2ch.net/test/read.cgi/math/1332950303/
・不等式への招待 第7章 http://rio2016.2ch.net/test/read.cgi/math/1362834879/
・過去スレのミラー置き場 http://cid-d357afbb34f5b26f.skydrive.live.com/browse.aspx/.Public/

姉妹サイト(?)
キャスフィ 高校数学板 不等式スレ2
http://www.casphy.com/bbs/test/read.cgi/highmath/1359202700/l50
Yahoo! 掲示板 トップ > 科学 > 数学
http://messages.yahoo.co.jp/bbs?action=t&;board=1835554&sid=1835554&type=r&first=1
0002132人目の素数さん
垢版 |
2017/06/25(日) 17:21:50.49ID:dLSgUfzK
不等式の和書
[1] 不等式(数学クラシックス11),ハーディ・リトルウッド・ポリヤ,シュプリンガー,2003年
   http://amazon.jp/o/ASIN/4431710566
[2] 不等式(数学選書),大関信雄・青木雅計,槇書店,1967年(絶版)
[3] 不等式への招待(数学ゼミナール6),大関信雄・大関清太,近代科学社,1987年
   http://amazon.jp/dp/4844372661
[4] 不等式入門(数学のかんどころシリーズ9),大関清太,共立出版,2012年
   http://www.kyoritsu-pub.co.jp/bookdetail/9784320019898
[5] 不等式入門(数学ライブラリー教養篇4),渡部隆一,森北出版,2005年
   http://amazon.jp/o/ASIN/4627010494
[6] 不等式の工学への応用、海津聰、森北出版,2004年
   http://amazon.jp/o/ASIN/4627075812
[7] 不等式(モノグラフ4),染取弘,科学新興新社,1990年
   http://amazon.jp/o/ASIN/4894281740
[8] 不等式 〜 21世紀の代数的不等式論 〜,安藤哲哉,数学書房,2012年
  http://amazon.jp/dp/4903342700
[9] 美しい不等式の世界: 数学オリンピックの問題を題材として,佐藤淳郎(訳),朝倉書店,2013年
  http://amazon.jp/dp/4254111371
[10] 思考力を鍛える不等式(大学への数学・別冊)、栗田哲也、東京出版、2014年
  http://www.amazon.co.jp/dp/4887422091
0003132人目の素数さん
垢版 |
2017/06/25(日) 17:22:38.22ID:dLSgUfzK
不等式の項目を含む和書
[1] 数学トレッキングツアー第3章「相加平均≧相乗平均」,東京理科大学数学教育研究所,教育出版,2006年
   http://amazon.jp/o/ASIN/4316801988
[2] 獲得金メダル! 国際数学オリンピック第1章「不等式」,小林一章,朝倉書店,2011年
   http://www.asakura.co.jp/books/isbn/978-4-254-11132-3/
[3] 数学オリンピック事典,数学オリンピック財団,朝倉書店,2001年
   http://amazon.jp/o/ASIN/4254110871
[4] 三角法の精選103問(シリーズ:数学オリンピックへの道 2),T.アンドレースク・Z.フェン著,朝倉書店,2010年
   http://www.asakura.co.jp/books/isbn/978-4-254-11808-7/
[5] 最大値と最小値の数学,P.J.ナーイン,シュプリンガー,2010年
   http://amazon.jp/dp/4621062131
[6] 最大・最小(数学one Point双書24),服部泰,共立出版,1979年
   http://amazon.jp/dp/4320012445

不等式の洋書
[1] The Cauchy-Schwarz Master Class: An Introduction to the Art of Mathematical Inequalities,J. M. Steele,Cambridge Univ. Pr.,2004年
   http://amazon.jp/o/ASIN/052154677X
[2] Inequalities: A Mathematical Olympiad Approach,Birkhaeuser Basel,2009年
   http://amazon.jp/dp/3034600496
[3] Inequalities: Theorems (Techniques and Selected Problems),Zdravko Cvetkovski,Springer,2012年
   http://amazon.jp/gp/product/3642237916
[4] Analytic Inequalities,Xingzhi Zhan,Dragoslav S., Dr. Mitrinovic,Springer,1970年
   http://www.amazon.co.jp/dp/3642999727
[5] Matrix Inequalities (Lecture Notes in Mathematics, No.1790),Xingzhi Zhan,Springer,2002年
   http://amazon.jp/dp/3540437983
[6] Matrix Analysis (Graduate Texts in Mathematics),Rajendra Bhatia,Springer,1996年
   http://amazon.jp/dp/0387948465
0004132人目の素数さん
垢版 |
2017/06/25(日) 17:23:11.77ID:dLSgUfzK
不等式の記事
[1] 特集 「現代の不等式」 (数理科学 No.386) ,サイエンス社,1995年8月号(絶版)
[2] 特集 「不等式の世界」 (数学セミナー No.2-569) ,日本評論社,2009年2月号
   http://amazon.jp/o/ASIN/B001O9UIZ8
[3] 連載 「不等式の骨組み」 (大学への数学 vol.53,全12回,各4ページ),栗田哲也,東京出版,2009年4月号-2010年3月号,2014年に書籍化(不等式の和書[10])
   http://www.tokyo-s.jp/index.shtml

不等式の埋蔵地
[1] RGMIA http://rgmia.vu.edu.au/
[2] Crux Mathematicorum Synopses http://www.journals.cms.math.ca/CRUX/synopses/
[3] Maths problems http://www.kalva.demon.co.uk/
[4] Mathematical Inequalities & Applications http://www.ele-math.com/
[5] American Mathematical Monthly http://www.maa.org/pubs/monthly.html
[6] Problems in the points contest of KöMaL http://www.komal.hu/verseny/feladatok.e.shtml
[7] IMO リンク集 http://imo.math.ca/
[9] Mathematical Olympiads Correspondence Program http://www.cms.math.ca/Competitions/MOCP/
[10] Mathematical Excalibur http://www.math.ust.hk/excalibur/
[11] MathLinks Contest http://www.mathlinks.ro/Forum/contest.html
[12] MATH PROBLEM SOLVING WEB PAGE http://www.math.northwestern.edu/~mlerma/problem_solving/#proposed_problems (要自動登録)
[13] Wolfram MathWorld http://mathworld.wolfram.com/
[14] GRA20 Problem Solving Group http://www.mat.uniroma2.it/~tauraso/GRA20/main.html
[15] American Mathematical Monthly Problems http://www.mat.uniroma2.it/~tauraso/AMM/amm.html
[16] Journal of Inequalities and Applications http://www.hindawi.com/journals/jia/
[17] すうじあむ http://suseum.jp/gd/all_berry_list/3504

海外不等式ヲタの生息地
[1] Journal of Inequalities in Pure and Applied Mathematics http://jipam.vu.edu.au/
[2] MIA Journal http://www.mia-journal.com/
[3] MathLinks Math Forum http://www.mathlinks.ro/Forum/forum-55.html
0005132人目の素数さん
垢版 |
2017/06/25(日) 17:23:50.34ID:dLSgUfzK
  いいぜ ヘ(^o^)ヘ
        |∧
        /

てめえが
不等式を
集めるってなら

         /
      (^o^)/
     /( )
    / / >

   (^o^) 三
   (\\ 三
   < \ 三

`\
(/o^)
( / まずは
/く そのふざけた
   不等式を証明するッ!
0006132人目の素数さん
垢版 |
2017/06/25(日) 17:30:41.80ID:dLSgUfzK
     ///////
    ///////____________
    ///////  ̄ ̄ ̄ ̄ ̄ ̄ ̄ ̄ ̄| ̄ ̄
   ///////      ___    (~) チリンチリン
   ///////     /  ≧ \  ノ,,
  ///////     |::::: (● (● |    
  ///////      ヽ::::... .ワ.....ノ    日本の夏
 ///////      (つ へへ つ      不等式の夏
0007132人目の素数さん
垢版 |
2017/06/25(日) 18:09:14.93ID:dLSgUfzK
         \     不等式と言えば?         / Schurムズ… Jensen最強  Lehmusって
          \        ∧_∧ ∩AM-GMだろ / ∧_∧     ∧_∧      ∧_∧
Markovの不等式 \      ( ・∀・)ノ______  /  ( ;・∀・)    (; ´Д`)    (´Д`; )
の証明おしえて ∧ ∧\    (入   ⌒\つ  /|. /  ⊂   ⊂ )    ( つ ⊂ )    ( ⊃   ⊃
         (゚Д゚ )_\    ヾヽ  /\⌒)/  |/     〉 〉\\   〉 〉 く く   //( (
     / ̄ ̄∪ ∪ /| \  || ⌒| ̄ ̄ ̄|    /     (__) (_)  (_.)(_)  (_) (__)
   /∧_∧Polyaを読め \    ∧∧∧∧ /           
  / (;´∀` )_/       \  < 不    > レスの半分は自作自演、残りはなりすましでできてるよ
 || ̄(     つ ||/         \< 等 ま >  集めた不等式を同化し改良するから、抵抗は無意味だよ
 || (_○___)  ||            < 式    > 群生体だから無限の知識と無尽蔵の体力を持ってるよ
――――――――――――――― .< ヲ た >―――――――――――――――――――――
         ∧_∧ いつもながら < タ   >    ∧_∧テヘッ∧_∧   / ̄ ̄ ̄ ̄ ̄ ̄ ̄ ̄
         ( ;´∀`) 見事じゃのぉ <か  > \   ( ´∀`)  (´∀` )<不等式(大関・青柳)
    _____(つ_ と)___       ./∨∨∨ 不\ (    )__(    )  \__復刊キボンヌ!
 . / \        ___ \キタァ  /  ∧_∧等 \∧ ∧   ∧ ∧  ̄ ̄ ̄/.//| ̄ ̄ ̄ ̄ ̄
 .<\※ \____.|i\___ヽ.ウヒョ ./γ(⌒)・∀・ ) 式 \   ;) (     ;)   / ┃| |
  ヽ\ ※ ※ ※|i i|.====B|i.ヽ  /(YYて)ノ   ノ  ヲ  \↑ ̄ ̄↑\)_/     |__|/
    \`ー──-.|\.|_|◎_|_.i‐>/ \  ̄ ̄ ̄ ̄\  タ   \数ヲタ  | | ┃
      ̄ ̄ ̄ ̄|. | ̄ ̄ ̄ ̄| / ||ヽ|| ̄ ̄ ̄ ̄||  め    \   .|_)
0008132人目の素数さん
垢版 |
2017/06/25(日) 18:09:37.90ID:dLSgUfzK
三角不等式
AM-GM不等式
Cauchyの不等式
Chebyshevの不等式
Holderの不等式
Jensenの不等式
並べ替え不等式
Maclaurinの不等式
Newtonの不等式
Power Mean不等式
Minkovskiの不等式
Bernoulliの不等式
Muirheadの不等式
Karamataの不等式
ぬるぽビッチの不等式
0009132人目の素数さん
垢版 |
2017/06/25(日) 18:37:41.25ID:MnGS57Na
>>7
不等式(大関・青柳)
復刊キボンヌ!

不等式への招待
は尼でオンデマンド版が買えるけど
0010◆2VB8wsVUoo
垢版 |
2017/06/25(日) 18:45:17.11ID:i2ZaylUY
★★★忖度と処世術に汚染された日本人:権威主義的な支配と損したくない人達★★★
  〜〜〜芳雄氏が言う『研究者としての基本的態度』とは一体何だろうか〜〜〜

佐藤幹夫:自分自身の素朴な疑問に真剣に耳を傾ける。⇒不滅の金字塔を打ち立てる。
糞父芳雄:人間関係を駆使し他人を操り根回しを行う。⇒ハリボテお教授として君臨。

隠蔽の財務省、嘘吐きの文科省、そして問答無用に屈服させる官邸。コレでも先進国?

(佐藤師がしてたのは本物の研究だ。だが)芳雄氏がしてたのはケケケ、ケンキュウ。
外見を繕って偉そう見せさえすれば何でもヨロシ。ほんで教授になりさえすれば研究の
中身なんて何でもヨロシ。そもそも論文なんてモンは、外国の権威ある雑誌に掲載され
さえすれば、その中身のギロンなんて何でもヨロシ。そやし適当に書いてしまえ〜〜〜
中身がダメだと知ってて、ソレでもSTAP論文を外国に投稿して受理される。発覚したら
適当に言い逃れる醜い態度。オツムのダメな大学院生に「虚偽の良品ラベル」を貼って
世間に出荷するハリボテ大学は詐欺行為そのもの。世間に媚びを売って客商売に徹し、
『売れさえすれば学生の脳の質なんて何でもヨロシ』と居直る大学。そしてブランド名
だけを見て仕入れる世間。●●は一流大学やさかい、きっと優秀なエリートやろwww

中身を何も説明しないで、問答無用に上から押し付ける。ソレをイチャモンで騒いで、
そして邪魔して潰そうとする周囲の下々。大学教員も国会議事堂も、そして馬鹿板人の
遣ってる事も皆同じだ。日本人はバカ民族であり、今は外国にもちゃんとバレてるので
海外からも軽蔑されるだけであり、そのうちにどの国からも信用されなくなるだろう。

近視眼的で打算的な人生観を息子に押し付ける父親と、大脳に栄養が足りてない連中が
跋扈する永田町や霞が関に支配される国に住む不幸、一体どうしてくれるというのか。

■■■馬鹿板を続けたらオツムがスポンジ脳になるのでサッサと足を洗うべき。■■■

0011132人目の素数さん
垢版 |
2017/06/25(日) 18:45:50.05ID:dLSgUfzK
>>9
( ゚∀゚)つ [2] 不等式(数学選書),大関信雄・青木雅計,槇書店,1967年(絶版)
0012◆2VB8wsVUoo
垢版 |
2017/06/25(日) 18:49:54.11ID:i2ZaylUY
★★★忖度と処世術に汚染された日本人:権威主義的な支配と損したくない人達★★★
  〜〜〜芳雄氏が言う『研究者としての基本的態度』とは一体何だろうか〜〜〜

佐藤幹夫:自分自身の素朴な疑問に真剣に耳を傾ける。⇒不滅の金字塔を打ち立てる。
糞父芳雄:人間関係を駆使し他人を操り根回しを行う。⇒ハリボテお教授として君臨。

隠蔽の財務省、嘘吐きの文科省、そして問答無用に屈服させる官邸。コレでも先進国?

(佐藤師がしてたのは本物の研究だ。だが)芳雄氏がしてたのはケケケ、ケンキュウ。
外見を繕って偉そう見せさえすれば何でもヨロシ。ほんで教授になりさえすれば研究の
中身なんて何でもヨロシ。そもそも論文なんてモンは、外国の権威ある雑誌に掲載され
さえすれば、その中身のギロンなんて何でもヨロシ。そやし適当に書いてしまえ〜〜〜
中身がダメだと知ってて、ソレでもSTAP論文を外国に投稿して受理される。発覚したら
適当に言い逃れる醜い態度。オツムのダメな大学院生に「虚偽の良品ラベル」を貼って
世間に出荷するハリボテ大学は詐欺行為そのもの。世間に媚びを売って客商売に徹し、
『売れさえすれば学生の脳の質なんて何でもヨロシ』と居直る大学。そしてブランド名
だけを見て仕入れる世間。●●は一流大学やさかい、きっと優秀なエリートやろwww

中身を何も説明しないで、問答無用に上から押し付ける。ソレをイチャモンで騒いで、
そして邪魔して潰そうとする周囲の下々。大学教員も国会議事堂も、そして馬鹿板人の
遣ってる事も皆同じだ。日本人はバカ民族であり、今は外国にもちゃんとバレてるので
海外からも軽蔑されるだけであり、そのうちにどの国からも信用されなくなるだろう。

近視眼的で打算的な人生観を息子に押し付ける父親と、大脳に栄養が足りてない連中が
跋扈する永田町や霞が関に支配される国に住む不幸、一体どうしてくれるというのか。

■■■馬鹿板を続けたらオツムがスポンジ脳になるのでサッサと足を洗うべき。■■■

0013132人目の素数さん
垢版 |
2017/06/26(月) 00:45:51.18ID:69lL3x+q
>>12 こんの、ハゲーーーっ!!
0014132人目の素数さん
垢版 |
2017/06/26(月) 02:02:42.73ID:WMgNNINg
1001 1001 Over 1000 Thread
このスレッドは1000を超えました。
もう書けないので、新しいスレッドを立ててくださいです。。。
life time: 1569日 3時間 9分 28秒

約4年かかってて草
0015132人目の素数さん
垢版 |
2017/06/26(月) 03:33:28.36ID:ChRIm5Q7
全国521駅「10年累計鉄道自殺数」ランキング
2016年06月22日
西八王子駅(東京)……39件
桶川駅(埼玉)…………34件
川崎駅(神奈川)………31件
新小岩駅(東京)………30件
新宿駅(東京)…………30件
八王子駅(東京)………30件
http://toyokeizai.net/articles/-/123503


JR川崎駅前にマタハリー(ピア、サントロぺ)のパチンコ台が約1800台、パチスロ台が約1000台ほどある。

その台はすべて、遠隔操作されています。

大勝ちしてる人のほとんどが内子です(ピアは内子の人数が日本一多い、詐欺犯罪組織です)。

今は大手のパチンコ店の大当たりはすべて遠隔大当たりなんです。

大当たりはアホ幹部がパソコンを1、3回クリックして大当たりさせています。

借金が原因で自殺してる人が多いけど、その原因は遠隔大当たりしかないパチンコ、パチスロなんです。

新小岩と新宿にはマルハンとエスパスがあります(エスバスは新宿歌舞伎町で一番大きなパチンコ店)。

西八王子駅の隣駅の八王子駅にはピアがあります(八王子駅にはパチンコ店がたくさんあります)。
0026132人目の素数さん
垢版 |
2017/06/28(水) 11:25:43.61ID:GT7HZs9l
Σ(a_i)^2≧(1/n)(Σ(a_i))^2

和は1からnまで
a_iは実数です

これって成り立ちますかね?


a^2+b^2≧(1/2)(a+b)^2
a^2+b^2+c^2≧(1/3)(a+b+c)^2
みたいな感じです

成り立つならその証明を、成り立たないなら反例をおしえてほしいです
0029132人目の素数さん
垢版 |
2017/06/28(水) 13:01:24.59ID:qKgfuKoo
>>26
成り立つことの証明は
分からない問題はここに書いてね428
の>116に書いてあるよ。
0030132人目の素数さん
垢版 |
2017/06/28(水) 13:05:44.13ID:qKgfuKoo
>>26
B=2Σ_{1≦i<j≦n}(a_i・a_j)
から各積 a_i・a_j 1≦i<j≦n を取り出すときは
B の 2Σ_{1≦i<j≦n}(a_i・a_j) の部分から取るという話ね。
0042132人目の素数さん
垢版 |
2017/06/29(木) 11:27:16.00ID:W3RXb80R
〔問題216〕
実数a〜dについて
(aa+ac+cc) (bb+bd+dd)≧(3/4) (ab+bc+cd)^2,
(aa+ac+cc) (bb+bd+dd)≧(3/4) (ad-bc)^2,
0047132人目の素数さん
垢版 |
2017/06/30(金) 18:19:38.24ID:g/dkToLH
>>42
左辺が pp+pq+qq の形になるのは、アイゼンシュタイン整数Z[ω]のノルムみたいなもの?

ナゴヤ△と関係あるの賀茂鴨
0048132人目の素数さん
垢版 |
2017/06/30(金) 18:39:42.49ID:g/dkToLH
>>47
 z1 = a - cω,
 z2 = d - bω  (a〜d∈Z)
をアイゼンシュタイン整数とすると、
 z1・z2 = (ad-bc) - (ab+bc+cd)ω,
0051132人目の素数さん
垢版 |
2017/07/06(木) 20:49:24.79ID:nv6IrYms
実数 x,y,z が x^2 + y^2 + z^2 =1 をみたすとき、
(x-y)(y-z)(z-x)、(2x-y)(2y-z)(2z-x) の最大値を求めよ。
0052132人目の素数さん
垢版 |
2017/07/07(金) 01:47:29.44ID:aKMbWmCY
>>51
右:
y は x、z の中間にある、とする。
y を x、z の中間で動かすとき、
|x-y| |y-z| ≦ (1/4)|z-x|^2,
∴y=(x+z)/2(等間隔)のとき最大で
(与式)≦(1/4)|z-x|^3 ≦ 1/√2,
等号成立は(x,y,z)=(±1/√2, 0, 干1/√2)
0054132人目の素数さん
垢版 |
2017/07/08(土) 03:52:44.12ID:E7CWjLAg
>>53

sin(x) + cos(x) = y とおく。
1 - sin(x)^5 - cos(x)^5
 = (1/2) {1-sin(x)} {1-cos(x)} F(sin(x)+cos(x))
 = (1/4) (1-y)^2 F(y)
 ≧0,

F(y) = 4+3y+2yy+y^3 ≧ 8 - 5√2 > 0,

1 + sin(x)^5 + cos(x)^5
 = (1/2) {1+sin(x)} {1+cos(x)} F(sin(x)+cos(x))
 = (1/4) (1+y)^2 F(-y),
 ≧ 0,

F(-y) = 4-3y+2yy-y^3 ≧ F(√2) = 8 - 5√2 > 0,

を使うとか。
0055132人目の素数さん
垢版 |
2017/07/08(土) 12:59:39.37ID:E7CWjLAg
>>54 

補足
F(y) = F(-√2) + (√2 +y) {2 + (1 -(1/√2) +y)^2}
≧ F(-√2)
= 8 -5√2,

訂正
1 + sin(x)^5 + cos(x)^5
 = (1/2) {1+sin(x)} {1+cos(x)} F(−sin(x)−cos(x))
 = (1/4) (1+y)^2 F(-y),
 ≧ 0,
0056132人目の素数さん
垢版 |
2017/07/08(土) 13:38:17.62ID:E7CWjLAg
>>47-50

7 =|5+8ω|=|5ω+8|  … ナゴヤ

ただし、1+ω+ω^2 =0.


>>52
(x,y,z) は単位球面上の点。
x,zを止めてyだけ動かすのは無理
0058132人目の素数さん
垢版 |
2017/07/09(日) 17:40:44.04ID:hraGPmBR
〔Golden-Thompsonの不等式〕
A、Bがエルミート行列のとき、
 tr{exp(A+B)}≦ tr{exp(A)exp(B)}

S.Golden(1965)、C.J.Thompson(1965)
数セミ増刊「数学の問題 第(2)集」日本評論社(1978)No.96

No.96
0059132人目の素数さん
垢版 |
2017/07/10(月) 03:41:28.14ID:pArAdsTp
>>956 (3)

{Σ[n=1〜∞] (x/n)^n}^(1/x)≒ e^(1/e + 4/x + …)

Lim[x→∞]{Σ[n=1〜∞] (x/n)^n}^(1/x)= e^(1/e)= 1.444667861
0060132人目の素数さん
垢版 |
2017/07/12(水) 23:08:45.38ID:4DpnFpJn
       |
   \  __  /
   _ (m) _ピコーンの等式
      |ミ|
    /  `´  \
     ('A`)
     ノヽノヽ
       くく
0063132人目の素数さん
垢版 |
2017/07/13(木) 01:04:00.92ID:aYclV8OY
>>62
オノの不等式
> 1914年に T.オノはこの式が任意の三角形について成り立つと予想したが、
> 1916年に Balitrand によって予想が誤りであることと、鋭角三角形であればこの式が成り立つことが示された。

T.オノって何者だ?
0064132人目の素数さん
垢版 |
2017/07/13(木) 01:06:35.77ID:aYclV8OY
Ono Inequality
鋭角三角形の3辺の長さを a, b, c, 面積を S とするとき、
27(b^2 + c^2 - a^2)(c^2 + a^2 - b^2)(a^2 + b^2 - c^2) ≦ (4S)^2
0065132人目の素数さん
垢版 |
2017/07/13(木) 01:19:17.26ID:aYclV8OY
不等式スレの第1章より前から集めているコレクションから引っ張り出してきた。
(つい最近まで出典をメモする習慣がなかったことを激しく後悔…)

実数 a,b,c に対して、
(b^2 + c^2 - a^2)(c^2 + a^2 - b^2)(a^2 + b^2 - c^2) ≦ {(b+c-a)(c+a-b)(a+b-c)}^2

さて、a,b,cを鋭角三角形の3辺の長さとして、この右辺と Ono Inequality の右辺の大小とか定まるかな?
0066132人目の素数さん
垢版 |
2017/07/13(木) 01:22:59.99ID:aYclV8OY
任意の三角形の3辺の長さ a,b,c に対して、
(a+b-c)(b+c-a)(c+a-b) ≦ abc
(a+b-c)^a*(b+c-a)^b*(c+a-b)^c ≦ a^a*b^b*c^c

       |
   \  __  /
   _ (m) _ピコーン、コンナノ アッタナァ
      |ミ|
    /  `´  \
     (゚∀゚)
     ノヽノヽ
       くく
0067132人目の素数さん
垢版 |
2017/07/13(木) 03:52:28.15ID:oVTfqBd/
>>65
a,b,cが鋭角△をなすとき
(bb+cc-aa)(cc+aa-bb)(aa+bb-cc) ≦ {(b+c-a)(c+a-b)(a+b-c)}^2 ≦(4S/√3)^3 ≦ (2s/3)^6,
S=△ABC、 s=(a+b+c)/2.

(左)
(bb+cc-aa)(cc+aa-bb)=(cc)^2 -(aa-bb)^2
=[c^2 - (a-b)^2]^2 - 2(aa+bb-cc)(a-b)^2
≦[c^2 - (a-b)^2]^2     (←鋭角)
=[(b+c-a)(c+a-b)]^2,
循環的に掛けて平方根。

(中)
相加-相乗平均より
 a+b+c ≧ 3{(b+c-a)(c+a-b)(a+b-c)}^(1/3),
 s ≧ 3{(s-a)(s-b)(s-c)}^(1/3),

S^2 = s(s-a)(s-b)(s-c)   (←ヘロンの公式)
 ≧ 3{(s-a)(s-b)(s-c)}^(4/3),

∴{(b+c-a)(c+a-b)(a+b-c)}^2 ≦ (4S/√3)^3,

(右)
S^2 = s(s-a)(s-b)(s-c) ≦ 3(s/3)^4,

∴(4S/√3)^3 ≦(2s/3)^6.
0068132人目の素数さん
垢版 |
2017/07/13(木) 04:08:19.11ID:oVTfqBd/
>>66

a+b-c=2z,b+c-a=2x,c+a-b=2y とおく。(*)
x,y,zは任意の正数。
abc - (a+b-c)(b+c-a)(c+a-b) = (y+z)(z+x)(x+y) - 8xyz
 = x(y-z)^2 + y(z-x)^2 + z(x-y)^2
 ≧ 0,
等号は x=y=z、つまり a=b=c (正△)

* Ravi変換とかいうらしい。
0069132人目の素数さん
垢版 |
2017/07/13(木) 05:10:22.83ID:aYclV8OY
(1)
正の数 a,b,c に対して、
(a+b+c)^5 ≧ 27(ab+bc+ca)(ab^2 + bc^2 +ca^2)

(2)
ab+bc+ca=3 をみたす正の数 a,b,c に対して、
a+b+c ≧ abc+1

(3)
a+b+c=3 をみたす正の数 a,b,c に対して、
(a^2 + bc^4)(b^2 + ca^4)(c^2 + ab^4) ≦ 64
____________________
  <〇√
   ‖
  くく



関係ないが、27って よく出てくるよな。

[第6章.908]
a,b,c>0のとき、{(a+b+c)(ab+bc+ca)}^2≧27abc(a^3+b^3+c^3)

[第5章.560]
a,b,cが三角形の三辺の長さのとき、
8/27 ≦ (a+b)(b+c)(c+a)/{(a+2b)(b+2c)(c+2a)},

[第5章.573]
1/4<(a+b)(b+c)(c+a)/(a+b+c)^3≦8/27 [1991 IMO]

[第5章.667]
正の数a、b、c、dに対して
 2(ab + ac + ad + bc + bd + cd)^3 ≧ 27(abc + abd + acd + bcd)^2

[第2章.144]
a, b, c≧0、a+b+c=1 のとき、a^2b+b^2c+c^2a ≦ 4/27 [1999 CMO]
0073132人目の素数さん
垢版 |
2017/07/13(木) 17:42:43.45ID:oVTfqBd/
>>71
(4)
(b+c)/a=x, (c+a)/b=y, (a+b)/c=z とおく。

x^3 + y^3 + z^3 = {(x+y+z)^3 +5s(ss-3t) +3(s^3-4st+9u)}/9 ≧ (1/9)(x+y+z)^3,

x+y+z = 6+(a/b+b/a-2)+(b/c+c/b-2)+(c/a+a/c-2)≧ 6,

>>72

B.3987
 中の b+c に注目する。
 (a+b+c)(b+c+d)=(b+c)(a+b+c+d)+ ad
 ≧(b+c){(a+b)+(c+d)}
 ≧ 2{√(a+b)}(b+c){√(c+d)},
 循環的に掛ける。

B.3989
 a=2cos(A),b=2cos(B),c=2cos(C) とおく。A+B+C=π
 cos(x)は下に凸だから
 a+b+c = 2{cos(A)+cos(B)+cos(C)}≦ 6cos((A+B+C)/3)= 6cos(π/3) = 3,

ご参考
 http://ameblo.jp/ineqfebot-sol/
0074132人目の素数さん
垢版 |
2017/07/13(木) 17:54:31.64ID:oVTfqBd/
>>73 訂正

B.3989
 cos(x)は|x|<π/2 で上に凸でした。

(別解)
 a=2sin(A/2),b=2sin(B/2),c=2sin(C/2) とおく。以下同様
0075132人目の素数さん
垢版 |
2017/07/13(木) 18:37:29.12ID:oVTfqBd/
>>72

A.422
Σ[i=1,n] x(i) = x(n+1) = S とおく。
Σ[i=1,n] x(i)^2 ≧ SS/n,
y=√x は上に凸だから
(左辺)^2 ≦ n{ Σ[i=1,n] x(i) [S -x(i)] }
  = n{ SS -Σ[i=1,n] x(i)^2 }
  ≦ n (SS - SS/n)}
  = (n-1) SS,

(右辺)^2 = SΣ[i=1,n] [S - x(i)]
  = S (n S - S)
  = (n-1) SS,
0076132人目の素数さん
垢版 |
2017/07/14(金) 01:59:14.12ID:54s0BI7v
>>72

A.422
(左辺)^2 ≦ n{Σ[i=1,n] x(i)[S-x(i)] }
  ≦{Σ[i=1,n] x(i)} {Σ[j=1,n] [S-x(j)]} (チェビシェフ)
  = S・(n-1)S
でもいいか...

〔B.3987.改〕
n個の正数{a,b,c, …,z}がある。
連続するk項の和を巡回的に掛けたものを P_k とおく。
P_1 = abcd…z,
P_2 =(a+b)(b+c)(c+d)……(z+a),
P_3 = (a+b+c)(b+c+d)……(z+a+b),
P_4 = (a+b+c+d)(b+c+d+e)……(z+a+b+c),
このとき、
 (P_k)^2 ≧ P_{k-1}・P_{k+1},
 P_{mn} ≧ (m^n)P_n,
を示せ。
0077132人目の素数さん
垢版 |
2017/07/14(金) 02:41:47.40ID:5qutPAyo
>>72
蒐集癖に火がついたでござる ( ゚∀゚) ハァハァ…

以下、a, b, c は a^2 + b^2 + c^2 + abc = 4 をみたす正の実数とする。←非負実数でいいよね?多分…
(1) a+b+c ≧ ab+bc+ca
(2) abc+2 ≧ ab+bc+ca ≧ abc
(3) a+b+c<3
(4) (2+a)(2+b)(2+c) ≧27abc
(5) sqrt{(2-a)/(2+a)} + sqrt{(2-b)/(2+b)} + sqrt{(2-c)/(2+c)} ≧ 3

(5)は、リンク先を見ると
sqrt{(2-a)/(2+a)} + sqrt{(2-b)/(2+b)} + sqrt{(2-c)/(2+c)} ≧ 3sqrt{3} ≧ sqrt(4-a^2) + sqrt(4-b^2) + sqrt(4-b^2)
と書いている者もいる。証明は未確認。


民明書房刊 「不等式ヲタの異常な蒐集癖、または私は如何にして心配するのを止めて不等式を愛するようになったか」より
(1) 出典のmemoがないでござる。過去スレにあるかも…
(2) USAMO 2001 https://artofproblemsolving.com/wiki/index.php?title=2001_USAMO_Problems/Problem_3
(3) >>72 B.3989 https://www.komal.hu/verseny/feladat.cgi?a=honap&;h=200703&t=mat&l=en
(4) https://artofproblemsolving.com/community/c6h527320p2994128
(5) https://artofproblemsolving.com/community/c6h527320s1_a2b2c2abc4_two_inequalities_sm
0078132人目の素数さん
垢版 |
2017/07/14(金) 04:47:25.52ID:54s0BI7v
>>77

(3) はイランMO-2002、A16 かな?

Solution 見ても出典が無い。ほんとに KoMaL


「博士の愛した不等式」慎重文庫(2005)
0080132人目の素数さん
垢版 |
2017/07/14(金) 10:25:53.58ID:54s0BI7v
>>66

a+b-c=z,b+c-a=x,c+a-b=y とおく。(*)
x,y,zは任意の正数。
a+b+c = x+y+z,
xy = cc-(a-b)^2 ≦ cc,
yz = aa-(b-c)^2 ≦ aa,
zx = bb-(c-a)^2 ≦ bb,

log(左辺)= a log(z)+ b log(x)+ c log(y)
 = (y/2)log(yz) + (z/2)log(zx) + (x/2)log(xy)
 ≦ y log(a) + z log(b) + x log(c)
 ≦ a log(a) + b log(b) + c log(c)  (←チェビシェフ)
 = log(右辺),
0081132人目の素数さん
垢版 |
2017/07/14(金) 13:02:23.89ID:54s0BI7v
>>69

[第5章.667]
a+b+c+d = s,ab+ac+ad+bc+bd+cd = t,abc+abd+acd+bcd = u とおく。
 2tt - (9/2)su =(ab-cd)^2 + (ac-bd)^2 + (ad-bc)^2 + (1/4)(aa+bb)(c-d)^2 + … ≧ 0,
 2st - 12u =(a+b)(c-d)^2 + (a+c)(b-d)^2 + … + (c+d)(a-b)^2 ≧ 0,
∴ 2t^3 ≧ 27uu,
0082132人目の素数さん
垢版 |
2017/07/14(金) 18:29:25.42ID:54s0BI7v
〔B.3987.改〕の略証を

>>76
 a_2 + a_3 + … + a_k = s とおく。
 (a_1+a_2+…+a_k)(a2+a3+…+a(k+1)) = (a_1 + s)(s + a_(k+1)) > s{a_1 + s + a_(k+1)}
 巡回的に掛ける。

>>79
 k=L のときは >>76
 k<L のときも
 {P_k P_L}/{P_(k-1) P_(L+1)}={(P_k)^2/P_(k-1)P_(k+1)}×{(P_(k+1))^2/P_k P_(k+2)}×
…… ×{(P_L)^2/P_(L-1)P_(L+1)} > 1,
0083132人目の素数さん
垢版 |
2017/07/15(土) 03:33:48.86ID:jZ3tY0g5
>>69

[第2章.144]
0 ≦ a ≦b,c としてよい。
4(a+b+c)^3 - 27(aab+bbc+cca+abc) = 9a(aa+bb+cc-ab-bc-ca) + (4b+c-5a)(a+b-2c)^2 ≧0,
等号成立は (a,b,c) = (0,2/3,1/3) とその rotation

カナダMO-1995 A.5
安藤哲哉:「不等式」数学書房(2012) 例題2.2.12(7)
0084132人目の素数さん
垢版 |
2017/07/15(土) 03:52:45.75ID:jZ3tY0g5
>>69

[第6章.908]
s = a+b+c, t = ab+bc+ca, u = abc とおく。
st = (aaa+bbb+ccc)+(abb+bcc+caa)+(aab+bbc+cca) = S+p+q ≧ 3(Spq)^(1/3),
pq = T+uS+3uu ≧ 3(3STU)^(1/3) ≧ 3√(3SU),
∴ S+p+q ≧ 3(Spq)^(1/3) ≧ 3√(3Su),
ここに、S=aaa+bbb+ccc、T=(ab)^3+(bc)^3+(ca)^3、U=(abc)^3.

Casphy!-不等式2-177
0085132人目の素数さん
垢版 |
2017/07/15(土) 04:39:34.33ID:yYh8jteX
そういえば、数蝉2017.08のエレガント第2問が、関数の最大最小値問題だったね。締切まで答えは書けないけど。
0086132人目の素数さん
垢版 |
2017/07/15(土) 07:59:24.12ID:OkWeDr+1
学コンの答えを締切前に発表したら刑事事件に発展するの?
業務妨害?
0087◆2VB8wsVUoo
垢版 |
2017/07/15(土) 08:12:09.69ID:qAOI4WFY
■■■輝かしい日本の未来の学問は、馬鹿板をしない国民一人一人が作るもの。■■■

0098132人目の素数さん
垢版 |
2017/07/15(土) 09:30:51.82ID:yYh8jteX
学コン厨がage荒らしをして、¥が荒らす。
面白スレや数セミスレでもよく見かける数学板の風物詩。
0099◆2VB8wsVUoo
垢版 |
2017/07/15(土) 09:53:34.66ID:qAOI4WFY
■■■輝かしい日本の未来の学問は、馬鹿板をしない国民一人一人が作るもの。■■■

0101◆2VB8wsVUoo
垢版 |
2017/07/15(土) 15:08:09.64ID:qAOI4WFY
◆◆◆馬鹿板をスルと脳が馬鹿汁漬けになってアホになります。そやし止めるべき。◆◆◆

0113◆2VB8wsVUoo
垢版 |
2017/07/16(日) 09:31:15.72ID:lJ3jPa7S
◇◇◇希望に満ちた明るい日本の将来は、馬鹿板を許さないネット社会の実現から。◇◇◇

0114132人目の素数さん
垢版 |
2017/07/16(日) 10:49:30.65ID:kYKIO7xV
x,y,z>0に対して、{(x+y)/z}^3 + {(y+z)/x}^3 + {(z+x)/y}^3 ≧ 24

少しずつ未整理の不等式コレクションを整理中。相変わらず出典不明。
引越し前のダンボールから出てきた紙なので、2009〜2010頃の入試問題だろうと思う。
もしかしたら海外の出題サイトから見つけたのかもしれないが…。
出典分かる人いたら教えて栗。

   ∩ _ _   ≡=−
   ミ(゚∀゚ ) ≡=−分数不等式! 巡回不等式! ヒャッホー!
    ミ⊃ ⊃    ≡=−
     (⌒ __)っ   ≡=−
     し'´≡=−
0116◆2VB8wsVUoo
垢版 |
2017/07/16(日) 11:10:58.67ID:lJ3jPa7S
◇◇◇希望に満ちた明るい日本の将来は、馬鹿板を許さないネット社会の実現から。◇◇◇

0120132人目の素数さん
垢版 |
2017/07/16(日) 11:45:07.73ID:kYKIO7xV
条件不等式のデータベースを作りたいね。
たとえば、上のような a^2 + b^2 + c^2 + abc = 4 かつ a,b,c>0 のときに成り立つ不等式がいろいろあるけど、
条件を代入して検索したら、それをみたす不等式がずらーっと出てくるような。
0122132人目の素数さん
垢版 |
2017/07/16(日) 12:54:40.01ID:kYKIO7xV
>>70
結局、a,b,c>0 かつ ab+bc+ca=3 のとき、a+b+c≧3 と 1≧abc が成立し、それをコッソリ使っていたのか…。
 a+b+c ≧ 3 ≧ 2+abc

種明かしされると何でもないけど、a+b+c≧2+abc をパッと見たとき、次数を合わせるために、
左辺と右辺の第1項に ab+bc+ca、右辺第2項に 3 を掛けてみて…、ずっと悩んでいた。
0124132人目の素数さん
垢版 |
2017/07/16(日) 13:46:47.29ID:kYKIO7xV
>>123
対称性を崩したくないのと、計算が面倒そうで、一文字消去は考えもしなかった。
0135132人目の素数さん
垢版 |
2017/07/17(月) 09:27:21.13ID:2cOdQU+V
>>118 (6)
最大になる位置は
(a,b,c)=(1.16745、1.83254、0)≒(7/6、11/6、0)
の辺りなので、
a+b≒3、b+c≒11/6、c+a≒7/6、
を利用して相乗-相加平均する。
(a+b)^(1/2)≦{(a+b)+ 3}/(2 √3)= 0.288675(a+b)+ 0.8660254,
(b+c)^(1/3)≦{(b+c)+(11/6)+(11/6)}/{3 (11/6)^(2/3)}= 0.222528(b+c)+ 0.815935
(c+a)^(1/4)≦{(c+a) +(7/6)+(7/6)+(7/6)}/{4 (7/6)^(3/4)}= 0.222705(c+a)+ 0.7794674

(左辺)≦ 0.511380 (a+b+c) + 2.4614278 ≦ 3.995568  (← a+b+c≦3)

ただし、条件 a+b+c≦3 を使い、出題よりも広い範囲で考えている。

出題の最大値 〜 3.9147720586
(a,b,c)=(1.17121、1.35653、0.396885)
0136132人目の素数さん
垢版 |
2017/07/17(月) 10:01:47.44ID:SY6Y6f40
[2009 大阪教育大]
(1) 実数 a,b が、a>0、ab≧4 をみたすとき、a+b≧4 を示せ。
(2) 実数 x,y が、x>0、(x^8)(y-x^2)≧4 をみたすとき、x(x+y)≧4 を示せ。

(1)のヒントがなかったら、(2)はどうするんだろう。(1)があってもムズいが…。
0138◆2VB8wsVUoo
垢版 |
2017/07/17(月) 10:36:15.95ID:PMZXT70X
▼▼▼馬鹿板からは身を引き、日々学問に真剣に取り組む姿勢こそが人の道である。▼▼▼

0139132人目の素数さん
垢版 |
2017/07/17(月) 13:05:29.18ID:2cOdQU+V
>>136

(1) (a+b)^2 = 4ab + (a-b)^2 ≧ 4ab ≧ 4^2

(2) y ≧ xx + 4/x^8 = 4/x^3 + (x - 2/x^4)^2 ≧ 4/x^3,

  x(x+y) ≧ x(x + 4/x^3) = xx + 4/xx = 4 + (x - 2/x)^2 ≧ 4,
0140◆2VB8wsVUoo
垢版 |
2017/07/17(月) 14:25:41.60ID:PMZXT70X
▼▼▼馬鹿板からは身を引き、日々学問に真剣に取り組む姿勢こそが人の道である。▼▼▼

0141132人目の素数さん
垢版 |
2017/07/17(月) 15:40:07.66ID:SY6Y6f40
>>77
> a, b, c は a^2 + b^2 + c^2 + abc = 4 をみたす正の実数とする。
> (1) a+b+c ≧ ab+bc+ca
>
> 出典のmemoがないでござる。過去スレにあるかも…

過去スレを漁ってみたら、たぶん、以下の問題と混同してしまったっぽい。
条件式が ab+bc+ca+abc=4 で違う。申し訳ないでござる。
反例をうまく見つけられんけど、 a^2 + b^2 + c^2 + abc = 4 のときには成り立つのかな?


[不等式スレ第4章 701]
> 701 名前:132人目の素数さん[sage] 投稿日:2010/08/29(日) 23:19:11
> a,b,c≧0、ab+bc+ca+abc=4のとき、a+b+c≧ab+bc+caを示せ
> 「大学への数学 2010-7 宿題」
>
> (解1) b+c=s、bc=t とおくと、a=(4-s)/(1+t)で、
> 0 < t ≦ (s^2)/4 で f(t) = -t^2+(s-1)t+s^2-4s+4 ≧ 0 を示す
>
> (解2 >>143) a≦b≦c とおくと a≦1≦c で、
> a+b+c-(ab+bc+ca) = {ac(1-a)(c-1)+(a+c-2)^2}/(1+ac) ≧ 0
>
> 解説には、「今のところ対称性を崩さない綺麗なジャイアンは見つかっていない」とある
0142◆2VB8wsVUoo
垢版 |
2017/07/17(月) 16:26:18.13ID:PMZXT70X
▼▼▼馬鹿板からは身を引き、日々学問に真剣に取り組む姿勢こそが人の道である。▼▼▼

0153132人目の素数さん
垢版 |
2017/07/17(月) 19:35:16.45ID:SY6Y6f40
[不等式スレ 第3章 343、第4章 627]
> cos(sinx)>sin(cosx) をしめせ。 (xは任意の実数)

改造せずにはいられない。
-π/2 < x < π/2 のとき、cos(sin x) > cos x > sin(cos x)

  ∧,,∧
 (;`・ω・)  。・゚・⌒)  不等式 改造するよ!!
 /   o━ヽニニフ))
 しー-J
0154◆2VB8wsVUoo
垢版 |
2017/07/17(月) 19:52:08.08ID:PMZXT70X
▼▼▼馬鹿板からは身を引き、日々学問に真剣に取り組む姿勢こそが人の道である。▼▼▼

0156◆2VB8wsVUoo
垢版 |
2017/07/17(月) 20:24:26.35ID:PMZXT70X
▼▼▼馬鹿板からは身を引き、日々学問に真剣に取り組む姿勢こそが人の道である。▼▼▼

0157132人目の素数さん
垢版 |
2017/07/17(月) 20:34:26.11ID:SY6Y6f40
>>155

     |
 \  __  /
 _ (m) _ピコーン
    |ミ|
 /___\ 
 ./  ≧ \ 
 |::::  \ ./ | 
 |::::: (● (● | < 相加! 相乗か!
 ヽ::::... .ワ.....ノ
0158132人目の素数さん
垢版 |
2017/07/17(月) 21:08:58.62ID:SY6Y6f40
>>77 (2)
改造せずにはいられない。
a^2 + b^2 + c^2 ≧ abc+2 ≧ ab+bc+ca ≧ abc


               ゚・ 。  ・。
               。・゚・⌒)
  −=≡       o━ヽニニフ ))
 −=≡   ( ゚∀゚)彡。・゚。・⌒)
−=≡   ⊂   o━ヽニニフ ))
 −=≡   ( ⌒)  改造! 改造!
  −=≡  c し'
0161132人目の素数さん
垢版 |
2017/07/18(火) 04:49:04.11ID:gmN7VRE9
>>153
各辺が周期πをもつばあいは、(最寄りの mπ から π/2 以内にあるとして)mπずらすことが可能でござる。
(オリジナルの周期は2πゆえ)たとえば絶対値を付けて
 cos(sin(x))≧|cos(x)|≧ |sin(cos(x))|,
0162132人目の素数さん
垢版 |
2017/07/19(水) 04:31:14.76ID:OXFuyCoZ
>>153 (続き)

-π/2 ≦ x ≦π/2 に対して
 cos(sin(x))≧|cos(x)|≧|sin(cos(x))|
ゆえ、任意の実数に対して成り立つ。
 左の等号 x=mπ
 右の等号 x=mπ±π/2


〔類題〕
 0.107126944873 ≦ cos(sin(x))-|sin(cos(x))|≦ cos(1)〜 0.54030230
 左の等号 x=mπ±0.692728570
 右の等号 x=mπ±π/2
0163132人目の素数さん
垢版 |
2017/07/19(水) 05:59:21.30ID:3YGTFP1s
>>69 (1)
> 正の数 a,b,c に対して、
> (a+b+c)^5 ≧ 27(ab+bc+ca)(ab^2 + bc^2 +ca^2)

基本対称式 s,t,u に置き換えても、うまく証明できんでござる。
0165132人目の素数さん
垢版 |
2017/07/19(水) 07:03:54.93ID:3YGTFP1s
[不等式 第7章]
> 241 名前:132人目の素数さん[sage] 投稿日:2014/09/18(木) 00:44:36.72
> 0<x<y<π/2の時
> (tanx/x)^x+(siny/y)^y<(tany/y)^y+(sinx/x)^x
> を示せ

これも証明できていない…
0167132人目の素数さん
垢版 |
2017/07/19(水) 08:58:35.51ID:OXFuyCoZ
>>69 (1)
>>163
 0 ≦ a ≦ b, c としてよい。
 この場合は基本対称式よりも b+c-2a = x の方がいいんぢゃね?

(左辺)=(a+b+c)^5 =(3a+x)^5
    = 243a^5 + 405a^4x + 270aaaxx + 90aaxxx + 15ax^4 + x^5,

 ab+bc+ca = 3aa + 2a(b+c-2a)+(b-a)(c-a)≦ 3aa + 2ax +(1/4)xx,
 abb+bcc+caa = 3aaa+3aa(b+c-2a)+a(b+c-2a)^2+(b-a)(c-a)^2 ≦ 3aaa+3aax+axx+(4/27)xxx,
(右辺)=27(ab+bc+ca)(abb+bcc+caa)≦243a^5+405a^4x+(1053/4)aaaxx+(345/4)aaxxx+(59/4)ax^4+x^5,

(左辺)-(右辺)≧ a(27aa+15ax+xx)xx/4 ≧ 0,
0168132人目の素数さん
垢版 |
2017/07/19(水) 09:58:00.35ID:OXFuyCoZ
>>69 (1)
>>163
 3a = A, b+c-2a = x とおくと…

(左辺)/243 ={(a+b+c)/3}^5 =(A+x)^5
   = A^5 + 5A^4・x + 10AAAxx + 10AAxxx + 5Ax^4 + x^5,

 ab+bc+ca ≦ {AA + 2Ax + (3/4)xx}/3,
 abb+bcc+caa ≦{AAA +3AAx +3Axx +(4/3)xxx}/9,
(右辺)/243 = (ab+bc+ca)(abb+bcc+caa)/9
   ≦ A^5 + 5A^4・x +(9.75)AAAxx +(9.58333…)AAxxx +(4.91666…)Ax^4 + x^5,

(左辺)-(右辺)≧ A(3AA+5Ax+xx)xx/12 ≧ 0,

>>167 と同じだが…
0169132人目の素数さん
垢版 |
2017/07/19(水) 10:37:49.14ID:OXFuyCoZ
>>137

x(x+y) ≧ 4.283918322582003
(x=1.1960916895833343 y=2.3855052397246037)
 3x^10 + 2x^9 - 28 = 0 の正根
0170132人目の素数さん
垢版 |
2017/07/19(水) 17:31:41.22ID:3YGTFP1s
>>167
さんくす。今夜読んでみます。

Shapiroの巡回不等式のn=6のときの証明を、>>2 [4] を見ながらやってみたけど、途中で詰まったでござる。
n=3のときは、f(x)=x/(s-x) に Jensenでok?
0171132人目の素数さん
垢版 |
2017/07/19(水) 19:52:03.56ID:OXFuyCoZ
>>170
>>2 [3] 「不等式への招待」(1987)p.28-30 を読むと

 B_i = x_{i+1} + x_{i+2}
とおく。ただし x_{n+1} = x_1, x_{n+2} = x_2

コーシーより
 Σ[i=1,n] x_i / B_i ≧ (Σ[i=1,n] x_i)^2 / {Σ[j=1,n] x_j B_j},

ゆえ
 (Σ[i=1,n] x_i)^2 -(n/2)Σ[j=1,n] x_j B_j ≧ 0
を言えばよい。

n=3,5 の場合は
 {1/(n-1)}Σ[1≦i<j≦n] (xi-xj)^2 ≧ 0,

n=4 のとき
 (x1-x3)^2 + (x2-x4)^2 ≧ 0,

n=6 のとき
 (1/2){(y1-y2)^2 + (y2-y3)^2 + (y3-y1)^2} ≧ 0,
 ここに、y1=x1+x4、y2=x2+x5、y3=x3+x6
と思うけど…
0172132人目の素数さん
垢版 |
2017/07/20(木) 01:52:11.90ID:Oabzsbx8
>>170

n=3(Nesbitt)の方はそれで おk ですね。ほかにも
 
a/(b+c)=(1/2){(a+b)/(b+c) -1 +(c+a)/(a+b)}
を巡回的にたして相加-相乗平均する。

a/(b+c)=(a+b+c)/(b+c) - 1
を巡回的にたして相加-調和平均する。

など種々ありますね。
http://mathtrain.jp/nesbitt
0173132人目の素数さん
垢版 |
2017/07/20(木) 02:37:10.34ID:Oabzsbx8
ピコーン太郎が歌う…

I have a function u(x) which satisfies{p1(x) u '(x)}' + q1(x)u(x) = 0.

I have a function v(x) which satisfies{p2(x) v '(x)}' + q2(x)v(x) = 0.

mmmmmmmmmmmmmm

Picone identity

{1/u(x)^2}{u(x)[p1(x)u '(x) - p2(x)u(x)v'(x)/v(x)]} ' = {q2(x)-q1(x)} + {p1(x)-p2(x)}{u '(x)/u(x)}^2 + p2(x){u '(x)/u(x) - v '(x)/v(x)}^2,
0184132人目の素数さん
垢版 |
2017/07/20(木) 17:09:09.33ID:27eqirM3
>>167-168
難しいです…。 検索して別のを見つけたが、bを中央の項としたとき、
なぜ 4(a^2+ac+c^2)(ab+bc+ca) ≦ (a+c)^2*(a+b+c)^2 となるのか分かりませぬ。
https://artofproblemsolving.com/community/c6h1235638p6275527

さらに強い不等式が載っている。
a,b,c>0 のとき、108(a+b+c)^5 ≧ (ab+bc+ca)(3125(a^2b+b^2c+c^2a)-627abc)

>>171
n=6の式変形が神。
分かってて変形しないと出来そうにない。
0185◆2VB8wsVUoo
垢版 |
2017/07/20(木) 17:16:33.16ID:R+taoMN8
★★★馬鹿板は悪い習慣であり、大脳が劣化します。なので早く止めましょう。★★★

0186132人目の素数さん
垢版 |
2017/07/20(木) 17:16:57.28ID:27eqirM3
>>62-63
小野ちゃんの不等式から、三角形絡みの不等式を検索して、フランダースの不等式に辿り着いた。
ところが過去スレを検索すると、既に初代スレに載っていたでござった…。全く記憶にござらぬ…。

[不等式 第1章]
> 668 名前:580[sage] 投稿日:04/11/22(月) 11:39:24
> 【補題】A+B+C=π, 0<A,B,C<π のとき、
>  0 < sin(A)sin(B)sin(C) ≦ {(3√3)/2π}^3 ABC ≦ (3√3)/8.
>  -1 < cos(A)cos(B)cos(C) ≦ [1-cos(A)][1-cos(B)][1-cos(C)] ≦ 1/8.
>
>  フランダースの不等式 とか言うらしい...
>  http://mathworld.wolfram.com/FlandersInequality.html
> ぬるぽ


   ,.-─-、
   / /_wゝ-∠l
   ヾ___ノ,. - >
   /|/(ヽY__ノミ
  .{   rイ  ノ
パトラッシュ、疲れたろう。
僕も疲れたんだ…
何だかとても眠いんだ…パトラ…
0187◆2VB8wsVUoo
垢版 |
2017/07/20(木) 17:22:58.82ID:R+taoMN8
★★★忖度と処世術に汚染された日本人:権威主義的な支配と損したくない人達★★★
  〜〜〜芳雄氏が言う『研究者としての基本的態度』とは一体何だろうか〜〜〜

佐藤幹夫:自分自身の素朴な疑問に真剣に耳を傾ける。⇒不滅の金字塔を打ち立てる。
糞父芳雄:人間関係を駆使し他人を操り根回しを行う。⇒ハリボテお教授として君臨。

隠蔽の財務省、嘘吐きの文科省、そして問答無用に屈服させる官邸。コレでも先進国?

(佐藤師がしてたのは本物の研究だ。だが)芳雄氏がしてたのはケケケ、ケンキュウ。
外見を繕って偉そう見せさえすれば何でもヨロシ。ほんで教授になりさえすれば研究の
中身なんて何でもヨロシ。そもそも論文なんてモンは、外国の権威ある雑誌に掲載され
さえすれば、その中身のギロンなんて何でもヨロシ。そやし適当に書いてしまえ〜〜〜
中身がダメだと知ってて、ソレでもSTAP論文を外国に投稿して受理される。発覚したら
適当に言い逃れる醜い態度。オツムのダメな大学院生に「虚偽の良品ラベル」を貼って
世間に出荷するハリボテ大学は詐欺行為そのもの。世間に媚びを売って客商売に徹し、
『売れさえすれば学生の脳の質なんて何でもヨロシ』と居直る大学。そしてブランド名
だけを見て仕入れる世間。●●は一流大学やさかい、きっと優秀なエリートやろwww

中身を何も説明しないで、問答無用に上から押し付ける。ソレをイチャモンで騒いで、
そして邪魔して潰そうとする周囲の下々。大学教員も国会議事堂も、そして馬鹿板人の
遣ってる事も皆同じだ。日本人はバカ民族であり、今は外国にもちゃんとバレてるので
海外からも軽蔑されるだけであり、そのうちにどの国からも信用されなくなるだろう。

近視眼的で打算的な人生観を息子に押し付ける父親と、大脳に栄養が足りてない連中が
跋扈する永田町や霞が関に支配される国に住む不幸、一体どうしてくれるというのか。

■■■馬鹿板をスルと稲田朋美みたいな嘘吐きになります。そやし止めなさい。■■■

0189◆2VB8wsVUoo
垢版 |
2017/07/20(木) 17:41:27.43ID:R+taoMN8
★★★馬鹿板は悪い習慣であり、大脳が劣化します。なので早く止めましょう。★★★

0190132人目の素数さん
垢版 |
2017/07/20(木) 17:47:02.49ID:27eqirM3
ageるとコピペ荒らしが来るから、sage進行で行きましょう。
まぁ Jane Style を使っているから、荒らし自体は あぼーんされて見えないけど、
無駄にレスが消費されて、すぐに次スレを立てないといけなくなるから。
0201132人目の素数さん
垢版 |
2017/07/20(木) 18:39:18.12ID:27eqirM3
>>188
Wlog、bを中央の項として、
c(a-b)(b-c)≧0 ⇔ b(a^2+ac+c^2) ≧ a^2b+b^2c+c^2a

(a+b+c)^5
= (1/8)*{2b + (a+c) + (a+c)}^3*(a+b+c)^2
≧ (27/4)*b(a+c)^2*(a+b+c)^2
= (27/4)*b*{(a^2+ac+c^2) + (ab+bc+ca)}^2
≧ 27b(a^2+ac+c^2)(ab+bc+ca)
≧ 27(a^2b+b^2c+c^2a)(ab+bc+ca)

  ┏━━━┓
  ┃ Q.E.D. ┃
  ┗━┳━┛
( ゚∀゚) ノ
0202132人目の素数さん
垢版 |
2017/07/20(木) 18:40:59.56ID:27eqirM3
>>184
> さらに強い不等式が載っている。
> a,b,c>0 のとき、108(a+b+c)^5 ≧ (ab+bc+ca)(3125(a^2b+b^2c+c^2a)-627abc)

難しすぎて ズコー
        ∧∧
       ヽ(・ω・)/   
      \(.\ ノ
    、ハ,,、  ̄
     ̄
0203◆2VB8wsVUoo
垢版 |
2017/07/20(木) 19:07:22.98ID:R+taoMN8
★★★忖度と処世術に汚染された日本人:権威主義的な支配と損したくない人達★★★
  〜〜〜芳雄氏が言う『研究者としての基本的態度』とは一体何だろうか〜〜〜

佐藤幹夫:自分自身の素朴な疑問に真剣に耳を傾ける。⇒不滅の金字塔を打ち立てる。
糞父芳雄:人間関係を駆使し他人を操り根回しを行う。⇒ハリボテお教授として君臨。

隠蔽の財務省、嘘吐きの文科省、そして問答無用に屈服させる官邸。コレでも先進国?

(佐藤師がしてたのは本物の研究だ。だが)芳雄氏がしてたのはケケケ、ケンキュウ。
外見を繕って偉そう見せさえすれば何でもヨロシ。ほんで教授になりさえすれば研究の
中身なんて何でもヨロシ。そもそも論文なんてモンは、外国の権威ある雑誌に掲載され
さえすれば、その中身のギロンなんて何でもヨロシ。そやし適当に書いてしまえ〜〜〜
中身がダメだと知ってて、ソレでもSTAP論文を外国に投稿して受理される。発覚したら
適当に言い逃れる醜い態度。オツムのダメな大学院生に「虚偽の良品ラベル」を貼って
世間に出荷するハリボテ大学は詐欺行為そのもの。世間に媚びを売って客商売に徹し、
『売れさえすれば学生の脳の質なんて何でもヨロシ』と居直る大学。そしてブランド名
だけを見て仕入れる世間。●●は一流大学やさかい、きっと優秀なエリートやろwww

中身を何も説明しないで、問答無用に上から押し付ける。ソレをイチャモンで騒いで、
そして邪魔して潰そうとする周囲の下々。大学教員も国会議事堂も、そして馬鹿板人の
遣ってる事も皆同じだ。日本人はバカ民族であり、今は外国にもちゃんとバレてるので
海外からも軽蔑されるだけであり、そのうちにどの国からも信用されなくなるだろう。

近視眼的で打算的な人生観を息子に押し付ける父親と、大脳に栄養が足りてない連中が
跋扈する永田町や霞が関に支配される国に住む不幸、一体どうしてくれるというのか。

■■■馬鹿板をスルと稲田朋美みたいな嘘吐きになります。そやし止めなさい。■■■

0214132人目の素数さん
垢版 |
2017/07/21(金) 03:35:42.35ID:aIensghT
>>184

(a+c)(a+b+c) = (aa+ac+cc) + (ab+bc+ca)だから

{(a+c)(a+b+c)}^2 ≧ 4(aa+ac+cc)(ab+bc+ca).

bが a,c の中間になくてもいいんぢゃね?
0225132人目の素数さん
垢版 |
2017/07/21(金) 07:34:08.41ID:aIensghT
>>184 >>202

3a=A、b-a=y、c-a=z とおく。(x=y+z)

 a+b+c = A+x,
 ab+bc+ca =(AA +2Ax +3yz)/3,
 abc = (AAA +3AAx +9Ayz)/27,
 aab+bbc+cca = (AAA +3AAx +3Axx +9yyz)/9,

(a+b+c)^5 = (A+x)^5 = A^5 + 5A^4・x + 10AAAxx + 10AAxxx + 5Ax^4 + x^5,

27(ab+bc+ca)(abb+bcc+caa) = A^5 + 5A^4・x + AAA(9xx+3yz) + AA(6xxx+9xyz+9yyz) + A(9x+18y)xyz + 27yyyzz,

81(ab+bc+ca)abc = A^5 + 5A^4・x + AAA(6xx+12yz) + 27AAxyz + 27Ayyzz + 0,

A^i の係数の差(A^0 の項が 27yyyzz ≦ (2916/3125) x^5 であること等)を考慮して適当な重みを定める。
0226◆2VB8wsVUoo
垢版 |
2017/07/21(金) 09:11:09.38ID:9Y4dp9MH
★★★忖度と処世術に汚染された日本人:権威主義的な支配と損したくない人達★★★
  〜〜〜芳雄氏が言う『研究者としての基本的態度』とは一体何だろうか〜〜〜

佐藤幹夫:自分自身の素朴な疑問に真剣に耳を傾ける。⇒不滅の金字塔を打ち立てる。
糞父芳雄:人間関係を駆使し他人を操り根回しを行う。⇒ハリボテお教授として君臨。

隠蔽の財務省、嘘吐きの文科省、そして問答無用に屈服させる官邸。コレでも先進国?

(佐藤師がしてたのは本物の研究だ。だが)芳雄氏がしてたのはケケケ、ケンキュウ。
外見を繕って偉そう見せさえすれば何でもヨロシ。ほんで教授になりさえすれば研究の
中身なんて何でもヨロシ。そもそも論文なんてモンは、外国の権威ある雑誌に掲載され
さえすれば、その中身のギロンなんて何でもヨロシ。そやし適当に書いてしまえ〜〜〜
中身がダメだと知ってて、ソレでもSTAP論文を外国に投稿して受理される。発覚したら
適当に言い逃れる醜い態度。オツムのダメな大学院生に「虚偽の良品ラベル」を貼って
世間に出荷するハリボテ大学は詐欺行為そのもの。世間に媚びを売って客商売に徹し、
『売れさえすれば学生の脳の質なんて何でもヨロシ』と居直る大学。そしてブランド名
だけを見て仕入れる世間。●●は一流大学やさかい、きっと優秀なエリートやろwww

中身を何も説明しないで、問答無用に上から押し付ける。ソレをイチャモンで騒いで、
そして邪魔して潰そうとする周囲の下々。大学教員も国会議事堂も、そして馬鹿板人の
遣ってる事も皆同じだ。日本人はバカ民族であり、今は外国にもちゃんとバレてるので
海外からも軽蔑されるだけであり、そのうちにどの国からも信用されなくなるだろう。

近視眼的で打算的な人生観を息子に押し付ける父親と、大脳に栄養が足りてない連中が
跋扈する永田町や霞が関に支配される国に住む不幸、一体どうしてくれるというのか。

■■■馬鹿板をスルと稲田朋美みたいな嘘吐きになります。そやし止めなさい。■■■

0227132人目の素数さん
垢版 |
2017/07/21(金) 12:10:20.32ID:hHnI1U1h
>>71 (4)
AM-GMを2回。ユルユルでござった。改造の余地ありまくリング。

>>225
>>201のようなカラクリはないのかな?

>>166
> 14. Prove that for any positive numbers a1, a2, ... , an we have:
> a1/(a2+a3) + a2/(a3+a4) + ... + an-1/(an+a1) + an/(a1+a2) > n/4
http://webee.technion.ac.il/people/aditya/www.kalva.demon.co.uk/soviet/sov69.html

Shapiroよりユルユルだから、エレガントな証明方法があるんかなあ?
0228132人目の素数さん
垢版 |
2017/07/21(金) 12:59:17.15ID:aIensghT
>>227

>>225 のようにバラバラに砕いたのは、「エレガントなカラクリ」を知らぬが故でござる。(最終兵器)
  ご存知なれば、伝授願いたいぐらい。

14.
[初代スレ.497-502] のことでござるか?
されば n/3 に改良する習わし也。

(補題)
 a/(b+c) +2b/(c+d) > (a+b)/(b+c) + (b+c)/(c+d) -1.
0229132人目の素数さん
垢版 |
2017/07/21(金) 13:32:43.01ID:aIensghT
>>227 (続き)

14. Shapiro
[初代スレ.497-502]
[第2章.284-285]
[第3章.172-173, 218-220]
[第4章.463-470]

n≦6の解法
[第2章.889-890]
[第8章.170-172]
0230132人目の素数さん
垢版 |
2017/07/21(金) 20:28:46.33ID:hHnI1U1h
作ってみたけど、簡単な証明あるかな? ( ゚∀゚) ウヒョッ!

a,b,c>0 に対して、{ a/sqrt(b+c) + b/sqrt(c+a) + c/sqrt(a+b) }^2 ≧ 3(a+b+c)/2
0232132人目の素数さん
垢版 |
2017/07/22(土) 15:48:14.89ID:G0nvuSlz
>>230

(解1)
a+b+c=s とおく。
f(X) = X/√(s-X)= s/√(s-X) - √(s-X)
は下に凸ゆえ Jensen で
f(a)+ f(b)+ f(c)≧ 3f(s/3)= √(3s/2),

(解2)
x=b+c, y=c+a, z=a+b とおく。
 a/√(b+c)=(y+z-x)/(2√x)≧{2√(yz) -x}/(2√x),
したがって、
 a/√(b+c)+ b/√(c+a)+ c/√(a+b)
 ≧{√(yz/x)+ √(zx/y)+ √(xy/z)}/2     …(*)
 = (xy+yz+zx)/(2√xyz),

(左辺) ≧ (xy+yz+zx)^2 /(4xyz)≧ 3(x+y+z)/4   …(**)
 = 3(a+b+c)/2,


*){√(yz)-x}/√x +{√(zx)-y}/√y +{√(xy)-z}/√z
 ={(xy+yz+zx)-x√(yz) -y√(zx)-z√(xy)}/√(xyz)
 ={x(√y-√z)^2 + y(√z-√x)^2 + z(√x-√y)^2}/(2√xyz)
 ≧0,

**)(xy+yz+zx)^2 - 3xyz(x+y+z)={xx(y-z)^2 + yy(z-x)^2 + zz(x-y)^2}≧ 0,
0233132人目の素数さん
垢版 |
2017/07/22(土) 16:03:11.35ID:G0nvuSlz
さて、本題に戻って…

(a+b+c)^5 ≧(ab+bc+ca){27(K+1)(aab+bbc+cca) -81K・abc}

とおいて、A^i の係数を求めます。 >>225

A^3 の係数から
 K ≦ 1/3,

A^2 の係数から
 K ≦ 0.182688493788
 (等号成立は y/z = 1.52984518)

A^1 の係数から
 K ≦ 0.07648328329
 (等号成立は y/z = 1.5765615)

A^0 の係数から
 27yyyzz = (2916/3125)(5y/3)^3 (5z/2)^2 ≦ (2916/3125)(y+z)^5 = (2916/3125)x^5,
 K ≦(3125/2916)- 1 =(209/2916)= 0.0716735…
 (等号成立は y/z = 3/2)

なんか上限がだんだん下がって来て窮屈ですが・・・
 K =(209/2916)とすれば OKです。

------------------------------------------------
(訂正)
27(ab+bc+ca) (aab+bbc+cca)= A^5 + 5A^4・x + …
0234132人目の素数さん
垢版 |
2017/07/23(日) 09:39:32.76ID:p7xlQ3BC
>>232
さすがなり。 >>230の元になった問題は以下。
https://math.stackexchange.com/questions/1483425/olympiad-inequality-problem-with-abcabc-4?rq=1

 a,b,c>0、a+b+c+abc=4 に対して、
 (ab+bc+ca)*{ a/sqrt(b+c) + b/sqrt(c+a) + c/sqrt(a+b) }^2 ≧ (1/2)*(4-abc)^3

条件 a+b+c+abc=4 は、右辺を難しそうに見せるだけのノイズと見て削除して、

 a,b,c>0 に対して、
 (ab+bc+ca)*{ a/sqrt(b+c) + b/sqrt(c+a) + c/sqrt(a+b) }^2 ≧ (1/2)*(a+b+c)^3

これは一般化されたヘルダーの不等式から出てくるが、他に易しい証明ないかな?
この右辺を弄って >>230 を得る。
0235132人目の素数さん
垢版 |
2017/07/23(日) 09:50:23.09ID:p7xlQ3BC
>>234
>  a,b,c>0 に対して、
>  (ab+bc+ca)*{ a/sqrt(b+c) + b/sqrt(c+a) + c/sqrt(a+b) }^2 ≧ (1/2)*(a+b+c)^3
>
> これは一般化されたヘルダーの不等式から出てくるが、

について、蛇足。

{a(b+c)+b(c+a)+c(a+b)}^(1/3)*{ a/sqrt(b+c) + b/sqrt(c+a) + c/sqrt(a+b) }^(2/3) ≧ a+b+c
0236132人目の素数さん
垢版 |
2017/07/23(日) 10:08:15.89ID:yTyAIG7a
>>225 >>233
 参考のため残しておきまつ。

(a+b+c)^5 -(ab+bc+ca){27(1+K)(aab+bbc+cca)- 81K・abc}における

A^3 の係数:
 (1-3K)(yy-yz+zz),

A^2 の係数:
 (4-6K)yyy -(6+9K)yyz + 3yzz +(4-6K),

A^1 の係数:
 5y^4 -(7+27K)y^3・z -(6+9K)yyzz + (11-9K)yz^3 + 5z^4,

A^0 の係数:
 (y+z)^5 - 27(1+K)yyyzz,

y≧0、z≧0 において上記がすべて非負となるような K≧0
を取れば十分でござる。
(x=y+z を使った)
0238132人目の素数さん
垢版 |
2017/07/23(日) 19:25:31.71ID:yTyAIG7a
>>184 >>201 >>214 を改造...

a+b+c = s、ab+bc+ca = t とおく。

{(a+c)s}^2 - 4(aa+ac+cc)t = (aa+ac+cc - t)^2 = δ

ss - 3t ={(a-c)^2・t + δ}/(a+c)^2 ≧ t|(a-c)/(a+c)|^2,
0239132人目の素数さん
垢版 |
2017/07/24(月) 10:30:53.82ID:mq+pfYuQ
>>232
> *){√(yz)-x}/√x +{√(zx)-y}/√y +{√(xy)-z}/√z

{2√(yz)-x}/2√x +{2√(zx)-y}/2√y +{2√(xy)-z}/2√z を計算しないといけないのでは?
0241132人目の素数さん
垢版 |
2017/07/24(月) 14:26:58.56ID:mq+pfYuQ
>>240 をプチ改造。
> 0 ≦ x ≦ π/2 に対して、2*sqrt{(sin x)*(cos x)} ≦ 2^{(sin x)*(cos x)} ≦ sinx + cos x ≦ sqrt(2)
0242132人目の素数さん
垢版 |
2017/07/24(月) 15:01:01.78ID:qItz5GdJ
>>238
(大意)
ss/t の値は、a,b,c が似たり寄ったりのときは3よりチョト大きいだけだが、
a,b,c が極端に違うときは(2変数値の)4に近いよ。


>>239
略しすぎた…
{2√(yz) -x}/(2√x) = {√(yz)}/(2√x) + {√(yz) - x}/(2√x),
のように分けたのでござる。
後ろの項は 巡回和すれば ≧0 でござる。(*)

ついでながら(**)の方も 1/2 が抜けてますな。トホホ
0243132人目の素数さん
垢版 |
2017/07/24(月) 15:54:27.34ID:qItz5GdJ
>>240-241

t = 2sin(x)cos(x)とおくと、0≦t≦1

一方、f(t)= 2^t は下に凸で f(0)=1, f(1)=2, f(2)=4 を通る。

0≦t≦1 では
(t=1、t=2 の割線)より上で、(t=0、t=1 の割線)より下。
∴  2t ≦ f(t) ≦ 1 + t ≦ 2,
4sin(x)cos(x)≦ 2^{2sin(x)cos(x)}≦ 1 + 2sin(x)cos(x)≦ 2,
各辺≧0 ゆえ平方根をとる。
0244132人目の素数さん
垢版 |
2017/07/24(月) 16:08:03.65ID:mq+pfYuQ
>>243
実にエレガント!

元ネタは [数蝉NOTE (2005.08締切分)]
x, y≧0 かつ x^2 + y^2=1 のとき、2^(xy) ≦ x+y ≦ sqrt(2).

まず、0 ≦ (x-y)^2 = 1-2xy より、0 ≦ xy ≦ 1/2 だから、

右 : (x+y)^2 = 1+2xy ≦ 2.
左 : (x+y)^{1/(xy)} = (1+2xy)^{1/(2xy)} ≧ 1 + 2xy*{1/(2xy)} = 2.

ベルヌーイの不等式を用いて、鶏を割くに焉んぞ牛刀を用いん…。
0245132人目の素数さん
垢版 |
2017/07/24(月) 16:37:45.26ID:mq+pfYuQ
まぁ、いろんな証明方法が身につくから褒め言葉なんですがね → 牛刀。
0248132人目の素数さん
垢版 |
2017/07/24(月) 19:10:05.77ID:DNnE4oh/
おめでとう
君は質問スレと面白スレの次くらいに人がいるスレを見つけた!!
0261132人目の素数さん
垢版 |
2017/07/27(木) 23:44:58.10ID:1T4+Oazx
〔問題1.96改〕
x, y, z ≧ 0 のとき
 x^3 + y^3 + z^3 -3xyz ≧ 4|(x-y)(y-z)(z-x)|,

ルーマニアMO-2007(改)
[9] 佐藤、演習問題1.96(改) >>2

-------------------------------
(略証)
yはxとzの中間にあるとする。
(x-y)(y-z)≧0,
 xx+yy+zz-xy-yz-zx =(x-y)^2 +(x-y)(y-z)+(y-z)^2,
 x+y+z ≧ |x-y| + |y-z| + min{|x-y|,|y-z|},
辺々掛けて
 x^3 + y^3 + z^3 -3xyz ≧(|x-y| + |y-z|)^3
 = |x-z|^3
 ≧4|(x-y)(y-z)(z-x)|,
0262132人目の素数さん
垢版 |
2017/07/28(金) 00:01:37.78ID:j+jikqys
〔楠瀬の不等式〕
x, y, z≧0 のとき
x^3 + y^3 + z^3 -3xyz ≧ k|(x-y)(y-z)(z-x)|,
 但し k = √(9+6√3)= 4.403669475・・・・・

数セミ創刊30周年記念『エレガントな問題をもとむ』【優秀賞】受賞問題2
 出題: 1992年4月, p.79
 解説: 1992年7月, p.59-60
 [初代スレ.836-869]
0263132人目の素数さん
垢版 |
2017/07/28(金) 00:35:42.32ID:KBT/ECMI
なんで k の値が、上では 4 なのに、下では4より大きくなってるん? 上では等号は成立しないの?
0264132人目の素数さん
垢版 |
2017/07/28(金) 11:59:01.11ID:j+jikqys
>>263

 >>261 は少しユルいのですが、簡単・便利な式です。
 x=y=z のときは等号が成立します。

 >>262 のkの値は「限界」で、もうこれ以上改良できません。
0265132人目の素数さん
垢版 |
2017/07/28(金) 12:45:15.12ID:O9aq1xVP
今高2で不等式をうまく使えるようになりたいのですが不等式の入門書?的なものはありますか?
一応高校数学の範囲は全部終わっています
0267◆2VB8wsVUoo
垢版 |
2017/07/28(金) 12:52:31.99ID:tqhSG1tp
###政治家が愚かなのと同様に馬鹿板を行うのも愚かな行為。そやし止めるべき。###

0279◆2VB8wsVUoo
垢版 |
2017/07/28(金) 15:33:55.90ID:tqhSG1tp
###政治家が愚かなのと同様に馬鹿板を行うのも愚かな行為。そやし止めるべき。###

0280132人目の素数さん
垢版 |
2017/07/28(金) 15:36:12.30ID:KBT/ECMI
中身のある書き込みがあると 中身のないレスでageる奴が現れるような気がする。
0281132人目の素数さん
垢版 |
2017/07/28(金) 16:19:13.21ID:O9aq1xVP
今高2で不等式をうまく使えるようになりたいのですが不等式の入門書?的なものはありますか?
一応高校数学の範囲は全部終わっています
0282132人目の素数さん
垢版 |
2017/07/28(金) 16:19:42.45ID:O9aq1xVP
今高2で不等式をうまく使えるようになりたいのですが不等式の入門書?的なものはありますか?
一応高校数学の範囲は全部終わっています
0283132人目の素数さん
垢版 |
2017/07/28(金) 16:20:00.23ID:O9aq1xVP
今高2で不等式をうまく使えるようになりたいのですが不等式の入門書?的なものはありますか?
一応高校数学の範囲は全部終わっています
0286132人目の素数さん
垢版 |
2017/07/28(金) 17:01:45.94ID:Zm1hbnDt
>>284
すまんこ
0287132人目の素数さん
垢版 |
2017/07/28(金) 17:01:56.27ID:Zm1hbnDt
>>285
許してクレメンス
0299132人目の素数さん
垢版 |
2017/07/28(金) 23:18:23.72ID:KBT/ECMI
>>261
まずココが分かりません。
>  x+y+z ≧ |x-y| + |y-z| + min{|x-y|,|y-z|}

次にココ。辺々掛けたら |x-y|^3 + |y-z|^3 + C にならないかな?
> 辺々掛けて x^3 + y^3 + z^3 -3xyz ≧(|x-y| + |y-z|)^3

最後にココ。AM-GMでもないし何だろう?
>  |x-z|^3 ≧ 4|(x-y)(y-z)(z-x)|
0300◆2VB8wsVUoo
垢版 |
2017/07/28(金) 23:22:27.57ID:tqhSG1tp
###政治家が愚かなのと同様に馬鹿板を行うのも愚かな行為。そやし止めるべき。###

0311132人目の素数さん
垢版 |
2017/07/29(土) 10:40:59.10ID:0o5qwo4/
>>299
それでは|x-y|= a,|y-z|= b とおきましょう。

まず
0≦x≦y≦z のとき
 x+y+z = 3x + 2(y-x)+(z-y)≧ 2(y-x)+(z-y) = 2a + b,
x≧y≧z≧0 のとき
 x+y+z =(x-y)+ 2(y-z)+ 3z ≧(x-y)+ 2(y-z)= a + 2b,

次に、辺々掛けると
 (2a+b)(aa+ab+bb)= a^3 +(a+b)^3 ≧(a+b)^3,
 (a+2b)(aa+ab+bb)=(a+b)^3 + b^3 ≧(a+b)^3,

最後は、
 (a+b)^2 = 4ab +(a-b)^2 ≧ 4ab,
0312132人目の素数さん
垢版 |
2017/07/29(土) 11:10:20.36ID:f+sckW2v
sage厨が湧いてくるぞ
0313◆2VB8wsVUoo
垢版 |
2017/07/29(土) 11:11:33.50ID:2P2kn60N
###政治家が愚かなのと同様に馬鹿板を行うのも愚かな行為。そやし止めるべき。###

0325◆2VB8wsVUoo
垢版 |
2017/07/29(土) 13:37:43.41ID:2P2kn60N
###政治家が愚かなのと同様に馬鹿板を行うのも愚かな行為。そやし止めるべき。###

0326132人目の素数さん
垢版 |
2017/07/29(土) 16:33:20.23ID:N79FPBpM
ageる奴ってほんま糞だな
ケツに「>」をぶち込んで拡張してやりたい
0327◆2VB8wsVUoo
垢版 |
2017/07/29(土) 16:38:28.09ID:2P2kn60N
###政治家が愚かなのと同様に馬鹿板を行うのも愚かな行為。そやし止めるべき。###

0338132人目の素数さん
垢版 |
2017/07/31(月) 03:54:33.34ID:XzE3duxv
[数蝉2014.07, p.51]
△ABCに対して、
|sin (A-B)/2|*(cos A/2)*(cos B/2) + |sin (B-C)/2|*(cos B/2)*(cos C/2) ≧ |sin (C-A)/2|*(cos C/2)*(cos A/2)

  ○ < ショウメイ スルマデ アガッテ クルナ!
 く|)へ
  〉 ヾ○シ
 ̄ ̄7 ヘ/
  /  ノ
  |
 /
`|
0349132人目の素数さん
垢版 |
2017/07/31(月) 23:55:08.45ID:XzE3duxv
任意の実数 x, y に対して、(1 + x^2 + y^2)/{1 + x^2 + (x-y)^2} の最大値を求めよ。

  Σ○
   く|)へ。
    〉   〉
 ̄ ̄   ○ノ 道連れッホォォ!
.  /  <ヽ
   |  /, |
 /
 |
0350132人目の素数さん
垢版 |
2017/08/01(火) 11:40:49.38ID:MADJ3GR6
>>349

φ =(1+√5)/2 = 1.618034 とおくと、

φ(1+xx+yy)-(φ-1){1+xx+(x-y)^2}= 1 + (x/φ + y)^2 ≧ 1,

上限は
 (1+xx+yy)/{1+xx+(x-y)^2}< φ/(φ-1)= φ+1 = (3+√5)/2,


なお、蛇足だが
φ{1+xx+(x-y)^2}-(φ-1)(1+xx+yy)= 1 + (φx - y)^2 ≧ 1,

下限は
 (1+xx+yy)/{1+xx+(x-y)^2}>(φ-1)/φ = 2-φ = (3-√5)/2,
0351132人目の素数さん
垢版 |
2017/08/01(火) 11:53:02.53ID:MADJ3GR6
>>350 訂正

次の同値な2式を入れ替えてください。

φ{1+xx+(x-y)^2}-(φ-1)(1+xx+yy)= 1 + (φx - y)^2 ≧ 1,

φ(1+xx+yy)-(φ-1){1+xx+(x-y)^2}= 1 + (x/φ + y)^2 ≧ 1,

スマソ.
0352132人目の素数さん
垢版 |
2017/08/01(火) 14:40:16.98ID:XEmVHg+K
最大最小といえば、高校のときに解けなかった以下を思い出す。係数はうろ覚え。

任意の実数 x, y に対して、(x+2y+3)/(x^2 + 2y^2 + 3) のとりうる値の範囲を求めよ。

    
 ̄ ̄ \○ノ テヲ ハナセ!
.  /  <ヽ
   |  / |
 /  (○ノ コトワルッ!
 |    ( )
/   / |
0353132人目の素数さん
垢版 |
2017/08/01(火) 15:08:14.92ID:MADJ3GR6
>>352
 (xx+2yy+3) - 2(√2 -1)(x+2y+3) = (x+1-√2)^2 + 2(y+1-√2)^2 ≧ 0,
 (xx+2yy+3) + 2(√2 +1)(x+2y+3) = (x+1+√2)^2 + 2(y+1+√2)^2 ≧ 0,
両辺を xx+2yy+3 >0 で割って
 -(√2 -1)/2 ≦ (x+2y+3)/(xx+2yy+3) ≦ (√2 +1)/2,
でござるか。
0355132人目の素数さん
垢版 |
2017/08/02(水) 01:25:07.20ID:RQb3zemz
[元ネタ不明]
任意の実数 x, y, z に対して、次式の最小値を求めよ。
sqrt{x^2 + (y-1)^2} + sqrt{y^2 + (z-1)^2} + sqrt{z^2 + (x-1)^2}


ウリャッ!
 Oノ
. ノ\_・'ヽO.
  └ _ノ ヽ
      〉
     ヾ○シ
 ̄ ̄7 ヘ/
  /  ノ
 /
 |
0356132人目の素数さん
垢版 |
2017/08/02(水) 01:35:57.66ID:iuzeTNl6
>>353

3(xx+2yy) = (x+2y)^2 + 2(x-y)^2 ≧(x+2y)^2,

 等号成立は x=y のとき。

x+2y+3 = s とおくと、
(分母)≧(ss-6s+18)/3,

-(√2 -1)/2 ≦ 3s/(ss-6s+18)≦(√2 +1)/2,

でも出ますが...
0357132人目の素数さん
垢版 |
2017/08/02(水) 01:47:54.48ID:iuzeTNl6
>>355

√{xx + (1-y)^2}≧(|x|+|1-y|)/√2、etc.
等号成立は|x|=|1-y|、|y|=|1-z|、|z|=|1-x|

△不等式 |x|+|1-x|≧ 1、etc.
等号成立は 0≦x,y,z≦1

より、3/√2。(x=y=z=1/2のとき)
0358132人目の素数さん
垢版 |
2017/08/02(水) 04:05:52.99ID:RQb3zemz
0 ≦ x, y, z ≦1 のとき、(x+y+z)/3 + sqrt{x(1-x) + y(1-y) + z(1-z)} の最大値を求めよ。

  Σ○
   ノ()へ。
    〉   〉
 ̄ ̄ \○ノ 道連れッホォォ!
   /  ( )
   |  / |
 /  (○ノ ヒャッホォォォゥ!
 |    ( )
/   / |
0359132人目の素数さん
垢版 |
2017/08/02(水) 04:16:32.01ID:RQb3zemz
巡回不等式のコレクションが少ないことに気づいた2017の夏。

正の数 a, b, c に対して、a^3/b^2 + b^3/c^2 + c^3/a^2 ≧ a+b+c を示せ。

    
 ̄ ̄ \○ノ テヲ ハナセ!
.  /  <ヽ
   |  / |
 /  (○ノ コトワルッ!
 |    ( )
/   / |
    (○ノ ザケンナヨ!
     ( )
    / |
0360132人目の素数さん
垢版 |
2017/08/02(水) 13:10:06.22ID:iuzeTNl6
>>358
相加平均(x+y+z)/3 = A とおくと、0≦A≦1.

x(1-x)+ y(1-y)+ z(1-z)
 =(x+y+z)- (xx+yy+zz)
 ≦ 3(1-A)・A   (←1変数)
 ≦{[3(1-A)+ A]/2}^2
 ≦{(3-2A)/2}^2

(左辺)≦ A +(3-2A)/2 = 3/2,
等号成立は 3(1-A)=A、A=3/4、x=y=z= 3/4 のとき

>>359
{a^n,b^n,…,b^n}の相加-相乗平均で
 a^n +(n-1)b^n ≧ na・b^(n-1),
 (a^n)/b^(n-1)≧ na - (n-1)b,
巡回的にたす。
0361132人目の素数さん
垢版 |
2017/08/02(水) 17:07:11.46ID:RQb3zemz
>>360
さりげなく一般化とは、やはり神!
正の数 a, b, c に対して、a^n/b^(n-1) + b^n/c^(n-1) + c^n/a^(n-1) ≧ a+b+c.

気になるのは、
(1) Σ[cyc] a^(n+1)/b^n と Σ[cyc] a^n/b^(n-1) の大小
(2) Σ[cyc] a^(n-1)/b^n と 1/a + 1/b + 1/c の大小

(1)も(2)も≧が成り立ちそうな気がするけど、証明できていませぬ。
0362132人目の素数さん
垢版 |
2017/08/02(水) 17:46:27.58ID:RQb3zemz
最大最小値問題を1変数にしたら、何通りくらいの解法があるのでせう?

任意の実数 x に対して、(5-2x)/(x^2 - 4x + 6) のとりうる値の範囲を求めよ。


 パキッ
    
 ̄`;:'. ̄ \○ノ 
.  /  <ヽ
   |  / |
 /  (○ノ 
 |    ( )
/   / |
    (○ノ 
     ( )
    / |
0363132人目の素数さん
垢版 |
2017/08/02(水) 21:09:22.85ID:iuzeTNl6
>>338

sin(a-b)cos(a)cos(b)+ sin(b-c)cos(b)cos(c)+ sin(c-a)cos(c)cos(a) + sin(a-b)sin(b-c)sin(c-a)

 | sin(a-b),-cos(c),cos(c)|
= | cos(a),sin(b-c),-cos(a)|
 | -cos(b),cos(b),sin(c-a)|

= 0,

を利用するか…?
0364132人目の素数さん
垢版 |
2017/08/02(水) 21:26:42.42ID:iuzeTNl6
>>362

(5-2x)/(xx-4x+6)= 1 -(x-1)^2/(xx-4x+6) ≦ 1,

(5-2x)/(xx-4x+6)= -1/2 +(x-4)^2/{2(xx-4x+6)} ≧ -1/2,

等号成立はそれぞれ、x=1、x=4.
0365132人目の素数さん
垢版 |
2017/08/02(水) 22:07:04.56ID:iuzeTNl6
>>361

(1)(a^n)/b^(n-1)は a^(n+1)/(b^n)(n-1)個と a 1個の相乗平均だから明らか。

ついでに、{a^n,…,a^n,b^n}で相加-相乗平均すると、
  n a^(n+1) + b^(n+1) ≧ (n+1)(a^n)b,
  n a^(n+1)/(b^n) ≧(n+1)(a^n)/b^(n-1) - b,
 循環的にたすと
  n S_(n+1) ≧(n+1)S_n - S_1,
  {S_(n+1)- S_1}/(n+1)≧(S_n - S_1)/n,
 (S_n - S_1)/n も単調増加。


* (a^n)/b^(n-1)は a^(n+1)/(b^n)(n-1)個と a 1個の相乗平均だが。

(2) a = 1/A、b = 1/B、c = 1/C とおくと…
0366132人目の素数さん
垢版 |
2017/08/03(木) 02:08:30.11ID:HTpcwzgX
>>363
[数蝉2014.07, p.51,NOTE] の前のページで証明されていた不等式が以下。
実数 x, y, z >0 に対して、|(x-y)/(x+y)| + |(y-z)/(y+z)| ≧ |(x-z)/(x+z)| …(★)

これをRavi変換 (a=y+z、b=z+x、c=x+y)すると、次の不等式になる。
三角形の辺長a,b,cに対して、ab|a-b| + bc|b-c| ≧ ca|c-a|…(★★)

これを a, c に関する対称性から a≦c として、bの位置を3通りに分けて証明。
正弦定理と2倍角の定理で書き直すと、次のようになって不等式が得られるみたい。

ab|a-b| = 32R^3 (sin A/2)*(sin A/2)*(sin A/2)*(cos A/2)*(cos B/2)*|sin (A-B)/2|

この不等式をNOTEに投稿した人のコメントに、(★)の元ネタが考古学の本とある。
「新井宏、理系から見た考古学の論争点、大和書房、2007」
不等式のネタが他にもあるかもしれないと思い、図書館や書店を探したが無かった。←今ココ。

ところで、(★★)を弄って、何か不等式が作れないかなと弄ったことがある。たとえば次式とか。
ab|a-b| + bc|b-c| + ca|c-a| ≧ k(a+b+c)
2014の夏ってことは、もう3年前の話になるのか。今考えたら、両辺の次数が合わないから無理やん…。
3乗にするか?
0367132人目の素数さん
垢版 |
2017/08/03(木) 02:24:17.48ID:HTpcwzgX
>>365
> (a^n)/b^(n-1)は a^(n+1)/(b^n)(n-1)個と a 1個の相乗平均

ムムム、スゴスギル…。

> 循環的にたすと n S_(n+1) ≧(n+1)S_n - S_1,

これから (S_n)/n は単調減少も出るでござるな。

これを差の形にして、nを 1,2,…,n-1として和を取り、右辺を部分分数分解して計算したら、
 (S_n)/n ≧ s_1/n
となって、何も得られなかったでござる…。
0369132人目の素数さん
垢版 |
2017/08/03(木) 10:53:04.22ID:Dkz1wYp5
>>366

(x-y)/(x+y)+(y-z)/(y+z)+(z-x)/(z+x)+(x-y)(y-z)(z-x)/{(x+y)(y+z)(z+x)}

 |(x-y)(x+y), -1, 1|
= | 1,(y-z)/(y+z), -1|
 | -1, 1,(z-x)/(z+x)|

= 0,


ab(a-b)+ bc(b-c)+ ca(c-a)+(a-b)(b-c)(c-a)=

 |a-b,c,-c |
= |-a,b-c,a |
 | b,-b,c-a|

= 0,

でござるか…?
0370132人目の素数さん
垢版 |
2017/08/03(木) 12:33:10.33ID:Tp76V4JM
(1) 任意の実数 a, b, c に対して次の不等式が成り立つような実数 k の最小値は?
|ab(a-b)+bc(b-c)+ca(c-a)| <= k (a+b+c)^3

(2) 実関数 f(a,b,c)=ab|a-b|+bc|b-c|+ca|c-a| は (a+b+c)^3 で上から、下からいぜれも抑えられないことを示せ。
0371132人目の素数さん
垢版 |
2017/08/03(木) 15:54:13.04ID:HTpcwzgX
コレクションの中に、以下を発見。年度不明の学習院大ってmemoがあるが…。

三角形の3辺の長さ a, b, c に対して、a^2b(a-b) +b^2c(b-c) + c^2a(c-a) ≧0.
0372132人目の素数さん
垢版 |
2017/08/03(木) 19:23:58.55ID:Dkz1wYp5
>>371
 a=y+z,b=z+x,c=x+y とおく。(Ravi変換)
 (左辺) = 2{xy^3 +yz^3 +zx^3 -xyz(x+y+z)}
    = 2xy(y-z)^2 + 2yz(z-x)^2 + 2zx(x-y)^2
    ≧ 0.
 IMO-1983
 佐藤[9]演習問題2.24
 [第6章.793(71),828,833]
0373132人目の素数さん
垢版 |
2017/08/03(木) 19:38:29.84ID:Dkz1wYp5
>>365 の続き

* (a^n)/b^(n-1)は a^(n+1)/(b^n)(n-1)個と a 1個の相乗平均だが、
 それで(1)が明らかなワケではない。

相加-相乗平均
 n(3n+1)a^(n+1)/(b^n)+(n+1)b^(n+1)/(c^n)+ n c^(n+1)/(a^n)≧(3nn+3n+1)a^n/b^(n^1),
を巡回的にたす。
0374132人目の素数さん
垢版 |
2017/08/03(木) 20:03:27.80ID:HTpcwzgX
>>373
> * (a^n)/b^(n-1)は a^(n+1)/(b^n)(n-1)個と a 1個の相乗平均だが、
>  それで(1)が明らかなワケではない。

巡回的に加えて、(n-1)*S_(n+1) + S_1 ≧ n*S_n
この左辺に、証明済みの S_(n+1) ≧ S_1 を使って終わりじゃないの?
0375132人目の素数さん
垢版 |
2017/08/04(金) 10:49:48.11ID:1Od1zBAC
>>370
勘違いとかあったから訂正

(1) a+b+c > 0 を満たす任意の実数 a, b, c に対して次の不等式が成り立つような実数 k の最小値は?
|ab(a-b)+bc(b-c)+ca(c-a)| <= k (a+b+c)^3

(2) a+b+c > 0 を満たす任意の実数 a, b, c に対して次の不等式が成り立つような実数 k の最小値は?
ab|a-b|+bc|b-c|+ca|c-a| <= k (a+b+c)^3

(3) a+b+c >0 上の実関数 f(a,b,c)=ab|a-b|+bc|b-c|+ca|c-a| は (a+b+c)^3 で下から抑えられないことを示せ。
0376132人目の素数さん
垢版 |
2017/08/04(金) 14:00:43.32ID:EUBWZejf
>>2
> [3] 不等式への招待(数学ゼミナール6),大関信雄・大関清太,近代科学社,1987年

数年ぶりに読み返してみた。傑作だな。神書だわ!
0377132人目の素数さん
垢版 |
2017/08/04(金) 19:07:55.26ID:ajzxje+k
>>359
そのまま相加-相乗平均で
 (n+1)^2 a^(n+1)/(b^n)+(n+1)n b^(n+1)/(c^n)+ nn c^(n+1)/(a^n)≧(3nn+3n+1)a,
巡回的にたして
 S_(n+1)≧ S_1,

>>374 >>376
 そうですね。
0378132人目の素数さん
垢版 |
2017/08/04(金) 22:15:15.32ID:ajzxje+k
>>338

|sin((A-B)/2)|cos(A/2)cos(B/2)=|sin(A-B)+ sin(A)- sin(B)|/4
=|sin(A-B)|/4 +|sin(A)-sin(B)|/4
= sin|A-B|/4 +|sin(A)-sin(B)|/4,etc.

|sin(x)|+|sin(y)|≧|sin(x)cos(y)+ cos(x)sin(y)|=|sin(x+y)|,

あとは△不等式で。
0381132人目の素数さん
垢版 |
2017/08/05(土) 09:23:14.77ID:Ulw6Zmyj
>>375
(1) a≧b≧cとする。
|ab(a-b)+bc(b-c)+ca(c-a)|
= |(a-b)(b-c)(c-a)|
≦ {(|a-b|+|b-c|+|c-a)|)/3}^3
= (8/27)*(a-c)^3

(a+b+c)^3 - (a-c)^3 = (b+c)(3a^2+3ab+b^2+bc+c^2) > 0
0382132人目の素数さん
垢版 |
2017/08/05(土) 10:01:17.84ID:v2fSy4wb
>>381
最後三角不等式使ってるようだけど、正しくは |a-b|+|b-c| >= |a-c| です
不等号が逆
k=8/27のとき 例えば (a,b,c) = (1,-3,1) で成り立たない
0384132人目の素数さん
垢版 |
2017/08/05(土) 10:06:55.20ID:Ulw6Zmyj
>>382
最後は三角不等式じゃなくて、等式でござるなり。 a≧b≧cの仮定を用いて、
|a-b|+|b-c|+|c-a| = (a-b) + (b-c) + (a-c) = 2(a-c)
0385132人目の素数さん
垢版 |
2017/08/05(土) 10:15:05.87ID:Ulw6Zmyj
>>381
a,b,cは実数ということを忘れていたので、以下は0より大きくならんでござるな。

> (a+b+c)^3 - (a-c)^3 = (b+c)(3a^2+3ab+b^2+bc+c^2) > 0
0386132人目の素数さん
垢版 |
2017/08/05(土) 11:17:13.28ID:v2fSy4wb
>>384
そうか
かくいう自分も回答にミス発見してそもそも(a+b+c)^3で上からも下からも抑えられないことがわかってでござる
0388132人目の素数さん
垢版 |
2017/08/05(土) 19:20:38.72ID:Ulw6Zmyj
>>2 [10] 思考力を鍛える不等式(大学への数学・別冊)、栗田哲也、東京出版、2014年 より

(1) [10] P.28
a>b>c>0 に対して、(a-b)sqrt(x+c) + (b-c)sqrt(x+a) + (c-a)sqrt(x+b) < 0

a,b,cの大小関係いらないんじゃ?


(2) [2006 山形大(医)] [10] P.77
三角形の辺長 a,b,c に対して、(2+a^2)(2+b^2) > 2c^2

⇒ (2+a^2)(2+b^2) ≧ 2(a+b)^2 > 2c^2

a.b.c>0 に対して、(2+a^2)(2+b^2)(2+c^2) ≧ 9(ab+bc+ca) だから、
これらを組合せたりして、なにか改造できないかな?


(3) [10] P.82
a,b,c>0に対して、(abc)^2 + a^2 + b^2 + c^2 + 2 ≧ 2(ab+bc+ca)

aの関数として微分して証明しているけど、他の証明ないかな。平方和とか…


(4) [10] P.115, 116
四面体ABCDに対して、
(i) ∠AOB + ∠BOC > ∠COA
(ii) ∠AOB + ∠BOC + ∠COA < 2π

[1992 東大(後)] >>2 [10] P.116
空間内の相異なる4点A,B,C,Dに対して、
(iii) ∠ABC + ∠BCD + ∠CDA + ∠DAB ≦ 2π

(iii)の条件を四面体ABCDに限定したら、等号がなくなるだけかな?


(5) [10] P.120
四面体ABCDに対して、vec(OA), vec(OB), vec(OC), vec(OD) を a,b,c,dと略すとき、
|a| + |b| + |c| + |a+b+c| > |a+b| + |b+c| + |c+a|

これは Hlawka's ineequality かな?


(6) [2012 大阪教育大]、[10] P.125
x,y>0 かつ (x^6)(y^2) - (x^5)(y^3) + (x^5)(y^5) - (x^4)(y^6) ≧ 4 のとき、x^3+y^2≧3

どうやって、こういう変な条件を出したのか分からないから、類題を作りにくい。


(7) [2013 北海道大]、[10] P.126
a,b,c,x,y>0 に対して、ax^(a+b+c) + by^(a+b+c) + c ≧ (a+b+c)(x^a)(y^b)

⇒ a,b,c,x,y,z>0 に対して、ax^(a+b+c) + by^(a+b+c) + cz^(a+b+c) ≧ (a+b+c)(x^a)(y^b)(z^c)

weighted-AM-GMだけど、入試問題で出されると答案書くのはシンドイな。
0389132人目の素数さん
垢版 |
2017/08/05(土) 22:22:51.97ID:BdLSvd9B
別にこのスレの参加者ではないが
面白い問題を見つけたので


平面上にA(p,q),B(r,s),C(t,u)とD(v,w)があるとき
(Dが△ABCの内部および周上)
⇔ ∃k, ∀(x,y)>0, (x^v)(y^w)≦k((x^p)(y^q)+(x^r)(y^s)+(x^t)(y^u)

出典:近大数コン2009-A4
0401132人目の素数さん
垢版 |
2017/08/06(日) 09:42:51.79ID:toVHuNxr
>>388-389
指数祭りかな?
自作問題でおじゃるが、簡単すぎた。

定数 a>0 に対して b = a^a とおくとき、a^a、a^b、b^a、b^b の大小を比較せよ。


   (^⌒⌒^)
    | i i i i i|    不等式、作るよ!
    | i i i i i|    
   (;`・ω・)っ-O・゚・⌒) 
   /  つ━ゝ,.゚__.,ノ))
        _l从从从从l_
  | ̄ ̄ ̄ ̄ ̄ ̄ ̄ ̄ ̄ ̄|
0402◆2VB8wsVUoo
垢版 |
2017/08/06(日) 10:15:43.68ID:+CYdGQny
☆☆☆馬鹿板は数学徒の脳を腐らせる悪い板であり、そやし廃止してナシにすべき。☆☆☆

0403132人目の素数さん
垢版 |
2017/08/06(日) 12:55:32.43ID:pqWLs7wT
(1)
√(x+a) = A、√(x+b)= B、√(x+c)= C とおくと
(左辺)=(AA-BB)C +(BB-CC)A +(CC-AA)B =(A-B)(B-C)(C-A),
ヤパーリ 要る…


(2)
(2+aa)(2+bb)(2+cc)≧(2√2)(a+b)(c+c)(c+a)≧{(16√2)/9}(a+b+c)(ab+bc+ca),
等号は a=b=c=√2.


(3)
 a = A^(3/2)、b = B^(3/2)、c = C^(3/2)とおく。
(左辺)=(ABC)^3 + A^3 + B^3 + C^3 +1 +1
  ≧ A^3 + B^3 + C^3 + 3ABC
  = AB(A+B)+ BC(B+C)+ CA(C+A)+ F_1(A,B,C)  ← Schur(n=1)
  ≧ 2{(AB)^(3/2)+(BC)^(3/2)+(CA)^(3/2)}
  = 2(ab+bc+ca),
0414132人目の素数さん
垢版 |
2017/08/07(月) 14:18:42.38ID:8+FZkWXB
[不等式スレ 第7章 984] 出典 「平成24年 第1回 東大入試プレ(文科)」

> 実数 a,b,c,d が a+b+c+d=0, a^2+b^2+c^2+d^2=100 をみたすとき、
> a^3+b^3+c^3+d^3 のとりうる値の範囲を求めよ。

> (-1000/√3, 1000/√3)に一票

エレガントな解法か、エロイ解法あるかな?
0415132人目の素数さん
垢版 |
2017/08/07(月) 22:48:29.46ID:EtB15xZg
>>414
普通にやっただけだからつまらないと思うけど
EV-theorem から a=b=c のときに最大・最小となるのは明らか。これを念頭に変形する

d=-(a+b+c) を第 2 式に代入して (a+b)^2+(b+c)^2+(c+a)^2=100
よって |(a+b)(b+c)(c+a)|<=(100/3)^(3/2)
|a^3+b^3+c^3+d^3|
=|3(a+b)(b+c)(c+a)|
<=1000/sqrt(3)
一方 d=a とすると c=-(2a+b), (a+b)^2+2a^2=50 (よって-5<=a<=5) から
与式 = -6*a*(b+a)^2 = -6a(50-2a^2)
これは [-1000/sqrt(3), 1000/sqrt(3)] の任意の値を取りうる
0416132人目の素数さん
垢版 |
2017/08/08(火) 06:26:42.69ID:0ekMhM3z
>>414-415
「東大入試プレ」で検索したが出てこない
 ↓
そもそも東大入試プレは何か検索すると、代ゼミの模試らしい
 ↓
「東大入試プレ 代ゼミ」で検索すると、かなり近づいてきた気がする
https://www.yozemi.ac.jp/news/osirase/1277627_3435.html
 ↓
左上のweb構成を見て、さらに検索し、目的の物を発見
https://www.yozemi.ac.jp/news/osirase/1285660_3435.html


その模範解答では、p+q=x、pq=y とおいて、x, y の関数として考えているらしい。
出典情報は大事だね。 まさか見つかるとは思っても見なかった。
0417132人目の素数さん
垢版 |
2017/08/09(水) 08:03:24.35ID:A2I5YGTu
いつもと違う出題形式。 いろんな解法を考えていて、おかしくなったでござる。

『実数 a, b>0 が ab ≧ a+b+1 をみたすとき、ab の最小値を求めよ。』
について、以下の解法(a)、(b)、(c)を考える。
(a)、(b)のどこがおかしいのか?

(a)
ab ≧ a+b+1 ≧ 3*(a*b*1)^(1/3)、等号はa=b=1 かつab=a+b+1
∴ (ab)^3 ≧ 27ab
ab>0で割って、(ab)^2 ≧ 27
ab>0だから、ab ≧ 3√3
等号成立条件をみたすa, bがないから、ab > 3√3

(b)
ab ≧ a+b+1 ≧ 2√(ab) + 1、等号はa=b かつab=a+b+1
∴ab-1 ≧ 2√(ab)
∴(ab-1)^2 ≧ 4ab
∴(ab)^2 - 6ab - 1 ≧ 0
ab>0だから、0 < ab ≦3-2√2 または 3+2√2 ≦ab

(c)
a+b ≧ 2√(ab) ≧ 2√(a+b+1)、等号はa=b かつ ab=a+b+1
∴ (a+b)^2 ≧4(a+b+1)
∴ (a+b)^2 - 4(a+b) - 4 ≧0
∴ a+b>0 だから、a+b ≧ 2+2√2
∴ ab ≧ a+b+1 ≧ 3+2√2
abの最小値は、3+2√2 (a=b=1+√2)
0418132人目の素数さん
垢版 |
2017/08/09(水) 09:43:16.84ID:DWUU74oj
>>417
(a)
間違ってない
ただ等号が成立しない雑な不等式を用いてるから最後の結論もいい加減になっただけ
ab>3sqrt3 を満たすとは言ってるけどそのすべての範囲を取りうるとは言っていない

(b)
条件 ab>=1 を加えればいい
0419◆2VB8wsVUoo
垢版 |
2017/08/09(水) 10:29:08.78ID:WvFggA1P
★★★馬鹿板は悪い習慣であり、大脳が劣化します。なので早く止めましょう。★★★

0420132人目の素数さん
垢版 |
2017/08/09(水) 13:30:53.39ID:A2I5YGTu
>>418
ありがとう。
脊髄反射でAM-GMを使って (a) のやり方でやって、アレレとなった。
結局、真面目に領域図示で片付けたんだが…。
0432132人目の素数さん
垢版 |
2017/08/09(水) 14:30:22.92ID:vWdGLnQX
>>388
(3)平方和で表わした。

(左辺)-(右辺) ={(abc)^2 -3GG +2}+{3(a+b+c -3G)GG + F_1(a,b,c)}/(a+b+c),

ここで、G =(abc)^(1/3)

(abc)^2 -3GG +2 = G^6 -3GG +2 = (GG+2)(GG-1)^2,

(a+b+c)-3G =(a'+b'+c'){(a'-b')^2+(b'-c')^2+(c'-a')^2}/2, a'=a^(1/3), b'=b^(1/3), c'=c^(1/3),

F_1(a,b,c) = a(a-b)(a-c) + b(b-c)(b-a) + c(c-a)(c-b)
 = {ab(aa-bb)^2 + bc(bb-cc)^2 + ca(cc-aa)^2}/{(a+b)(b+c)(c+a)}

(4) (i)

OB方向をz軸とし、
OAの天頂角を ∠AOB=α
OCの天頂角を ∠BOC=γ
とする。
cosβ = cos(∠COA) =(OC・OA)= cosα cosγ + sinα sinγ cosφ (φは方位角の差、0<φ<π)
∴ cos(α+γ)< cosβ < cos(α-γ),
∴ α+γ > β > |α-γ|
0443132人目の素数さん
垢版 |
2017/08/09(水) 14:51:36.23ID:vWdGLnQX
>>417
(d)
a,b>0 ゆえ
(√ab -1)^2 - 2 = ab -2√(ab) -1
 = ab -(a+b+1) +(√a-√b)^2
 ≧ 0,
∴ √ab ≧ 1+√2,
0444◆2VB8wsVUoo
垢版 |
2017/08/09(水) 15:28:26.39ID:WvFggA1P
★★★馬鹿板は悪い習慣であり、大脳が劣化します。なので早く止めましょう。★★★

0445132人目の素数さん
垢版 |
2017/08/09(水) 15:58:47.69ID:QFWbMnD6
(3)
a,b,cは任意の実数でよい
L-R=(a^2-1)(b^2-1)(c^2-1)+(ab-1)^2+(bc-1)^2+(ca-1)^2
よって絶対値が 1 以下のものが奇数個あるときのみ示せば十分
それを c とすると
(a^2-1)(b^2-1)(c^2-1)+(ab-1)^2 >= -(a^2-1)(b^2-1)+(ab-1)^2 = (a-b)^2 >= 0
0446◆2VB8wsVUoo
垢版 |
2017/08/09(水) 15:59:21.88ID:WvFggA1P
★★★馬鹿板は悪い習慣であり、大脳が劣化します。なので早く止めましょう。★★★

0449132人目の素数さん
垢版 |
2017/08/09(水) 17:01:42.00ID:A2I5YGTu
a, b, c >0 かつ abc=1 のとき、

(1) [memoには 2004 JMO とあるが、全然違っていた…]
 c/(1+a) + b/(1+b) + a/(1+c) ≧ 3/2

(2) [memoには 1998 Ukraina とあるが、もう自信がない]
 (1+ab)/(1+a) + (1+bc)/(1+b) + (1+ca)/(1+c) ≧ 3

(3) [疑問]
 1/(1+a) + 1/(1+b) + 1/(1+c) ≧?
 bc/(1+a) + ca/(1+b) + ab/(1+c) ≧?

(4) [1998 IMO shortlist.A3]
 a^3/{(1+b)(1+c)} + b^3/{(1+c)(1+a)} + c^3/{(1+a)(1+b)} ≧ 3/4

-----------------------------------------------------

TeXで編集する際に、問題順を入れ替えたりしているうちに、
問題番号と出典番号がずれて、もはや修正のしようがない。

確認したくても、リンク先が消えているし。
https://mks.mff.cuni.cz/kalva/index2.html


|  |             
| ‖           ノノノノ -__
|| ‖           (゚∈゚ )  ─_____ ___
|∧ 从ノ      (ミ_ (⌒\ヽ _ ___
( (≡ ̄ ̄ ̄ ̄三\⌒ノ ノ )
|(つWつ  ̄ ̄\  ⌒彡)   ノ  =_
| \つ つ    \,___,ノノ
|  |  )        / / ≡=
|  |          / ノ      __________
|  |         /ノ _─ (´⌒(´
|  |       ミ/= (´⌒(´⌒;;
| ''''""'''"'''"""''"""'''''"'"''''""''"''''"""''"'''""''"''"'''"''()
|  / ̄ ̄ ̄ ̄ ̄ ̄ ̄ ̄ ̄ ̄ ̄ ̄ ̄ ̄ ̄ ̄ ̄ ̄
0450132人目の素数さん
垢版 |
2017/08/09(水) 17:34:49.15ID:vWdGLnQX
>>388
(5) Hlawka の不等式 にござりまする。

(左辺)*(左辺 - 右辺)= Sq + Trig,

Sq = |a|^2 + |b|^2 + |c|^2 + |a+b+c|^2 - |a+b|^2 - |b+c|^2 - |c+a|^2,

Trig = (|b|+|c|-|b+c|) (|a|-|b+c|+|a+b+c|)
 + (|c|+|a|-|c+a|) (|b|-|c+a|+|a+b+c|)
+ (|a|+|b|-|a+b|) (|c|-|a+b|+|a+b+c|).
式の変形とはいえ、うまいものと感心するばかり。

Trig ≧0 は△不等式から出るが、Sq = 0 を出すには内積計算などが要る。(← Euclid性)

文献[3] 大関「不等式への招待」 p.33-34 例題8. >>2
0452132人目の素数さん
垢版 |
2017/08/09(水) 17:49:46.39ID:A2I5YGTu
>>449 に付け足し。

a, b, c >0 かつ abc=1 のとき、

(4) [出典不明、元問題は"3乗和≧2乗和"を一般化した]
 自然数nに対して、a^n + b^n + c^n ≧ a^(n-1) + b^(n-1) + c^(n-1)

(5) [出典不明]
 b/a + c/b + a/b ≧ a+b+c ≧ √a + √b + √c
 b/a + c/b + a/b ≧ 1/a + 1/b + 1/c ≧ √a + √b + √c

(6) [2016 東北大]
 a^2 + b^2 + c^2 ≧ 1/a + 1/b + 1/c

(7) [疑問]
 a^n + b^n + c^n ≧ b/a + c/b + a/b をみたす最小の n∈N はあるかな?

(8) [参考までに、これも出典のmemoがなくて困るが…]
 a^3 + b^3 + c^3 + (ab)^3 + (bc)^3 + (ca)^3 ≧ b/a + c/b + a/b

--------------------------------------------------------------

同じ条件の不等式を整理していると、この問題と あの問題は繋がるのでは?
などと気になりはじめると、整理どころではなくなる。そうして未整理の不等式が貯まっていく。

(5)の2つの不等式の中辺の大小は定まらない。(過去スレでやったような希ガス…)
abc=1 に注意して、(a+b+c)-(ab+bc+ca) = (a-1)(b-1)(c-1)
a, b, cと1の大小で、正にも、0にも、負にもなる。
0453132人目の素数さん
垢版 |
2017/08/09(水) 17:50:44.87ID:A2I5YGTu
>>452
(8)の訂正。右辺は2倍ですた。
 a^3 + b^3 + c^3 + (ab)^3 + (bc)^3 + (ca)^3 ≧ 2(b/a + c/b + a/b)
0454◆2VB8wsVUoo
垢版 |
2017/08/09(水) 18:14:30.50ID:WvFggA1P
★★★馬鹿板は悪い習慣であり、大脳が劣化します。なので早く止めましょう。★★★

0455132人目の素数さん
垢版 |
2017/08/09(水) 18:51:30.33ID:sOQtPSi2
>>449
(1), (2) a=y/x … とおくだけ
(3)
Σ1/(1+a) = 1 + (a+b+c+1)/(ab+bc+ca+a+b+c+1) -> 1 (c=1/(ab), a->inf, b->inf)
Σbc/(1+a) = Σ1/(a+a^2) >= Σ(-3/4log(a)+1/2) = 3/4
(4) 相加相乗で終わり
0456132人目の素数さん
垢版 |
2017/08/09(水) 19:19:25.80ID:vWdGLnQX
>>388 (2)
 (aa+2)(bb+2)(cc+2) ≧ 3(a+b+c)^2

Asia-Pacific MO-2004改
文献 [9] 佐藤(訳)、問題3.85改

(左辺)=(abc)^2 + 2(ab)^2 +2(bc)^2 +2(ca)^2 +4(aa+bb+cc) +8
 =(abc)^2 +2(ab-1)^2 +2(bc-1)^2 +2(ca-1)^2 +3(a+b+c)^2 -2(ab+bc+ca) +2
 ={(abc)^2 +aa +bb +cc +2 -2(ab+bc+ca)}+2(ab-1)^2 +2(bc-1)^2 +2(ca-1)^2 +3(a+b+c)^2
 ≧ 3(a+b+c)^2,

※ (abc)^2 +aa +bb +cc +2 -2(ab+bc+ca)≧ 0
 は >>388 (3)または練習問題1.90(i)を使う。


>>449 (4)
文献 [9] 佐藤(訳)、演習問題 1.120
0467132人目の素数さん
垢版 |
2017/08/09(水) 22:45:59.22ID:A2I5YGTu
不等式が少しだけ載っているというタレコミがあったので、事情徴収(立ち読み)してきた。
容疑者 : 佐久間一浩、『高校数学と大学数学の接点』、PP.18-30

(1) PP.18-24
三角形の辺長を a, b, c、面積をSとするとき、a^2 + b^2 + c^2 ≧ (4√3)S.

(2) PP.25-30
R ≧ 2r (球殻不等式)

(1)に対して、8通りの証明を与えていた。
(2)は d^2 = R^2 - 2Rr (茶ップル-オイラーの定理)を証明して片付けていた。
ここで d は外心と内心の距離。



   ∧,,∧    
   (`・ω・´) 8通りの証明だと? 詳しく聞こうか?
   (    )
 ̄ ̄Φ口U ̄ ̄\
   _ _.        \
_(    ) ← 佐久間\
 ̄┏┳┓)
0468132人目の素数さん
垢版 |
2017/08/09(水) 22:47:06.98ID:A2I5YGTu
>>467
> 三角形の辺長を a, b, c、面積をSとするとき、a^2 + b^2 + c^2 ≧ (4√3)S.

(証明1)
ヘロンの公式を使って a, b, c だけの式にして、(左辺)^2 - (右辺)^2

(証明2)
面積公式と余弦定理を使って a, b, c だけの式にして、(左辺)^2 - (右辺)^2

(証明3)
b+c-a=A, c+a-b=B, a+b-c=C とおいて、AM-GM とヘロンの公式。

(証明4)
a^2 + b^2 + c^2 ≧ ab+bc+ca の右辺に正弦定理を用いてから、凸不等式。

(証明5)
a^2 + b^2 + c^2 ≧ (4√3)S + (1/2){(a-b)^2 + (b-c)^2 + (c-a)^2} を証明。

(証明6)
a^2 + b^2 + c^2 ≧ (4√3)S + (a-b)^2 + (b-c)^2 + (c-a)^2 を証明。

(証明7)
証明6の不等式を三角関数で証明。

(証明8)
座標平面上に、頂点を A(a/2,0)、B(-a/2,0)、C(s,t)、t>0 とおいて計算。

---------------------------------------------------------------------

[1] そもそもヘロンの公式は、面積公式と余弦定理から三角関数を消去して得られるものだから、
  証明1と証明2は全く同じものである。証明6と証明7も一緒。つまり6通りの証明ですな。

[2] この不等式には、オノとかフランダースとか、なんか名前はついていないのかな?

[3] 他に証明は無いのかな。証明3と実質同じだが、Ravi変換くらいしか思いつかない。

ヘロンの公式を行列式で表すと、S = (√D)/4。ここでDは以下の行列式。
|0 1   1   1 |
|1 0 a^2 b^2|
|1 a^2 0 c^2|
|1 b^2 c^2 0 |
0469132人目の素数さん
垢版 |
2017/08/09(水) 22:48:59.09ID:DWUU74oj
>>388
>>456
相当な量の改良問題があった

for reals
[1] (a^2+1)(b^2+1)(c^2+1) >= (1+a+b)(1+b+c)(1+c+a)
[2] ((a^2+3)(b^2+3)(c^2+3))^2 >= 512(a+b)(b+c)(c+a)

for nonnegarives
[3] (a^2+2)(b^2+2)(c^2+2) >= 3(a+b+c)^2+(abc-1)^2
[4] (x^2+2)(y^2+2)(z^2+2) >= 4(x^2+y^2+z^2)+5(xy+yz+zx)+(xyz-1)^2
[5] (a^2+2)(b^2+2)(c^2+2) >= 4(a^2+b^2+c^2)+5(ab+bc+ca)+(abc(a-1)^2(b-1)^2(c-1)^2)^(1/3)

AOPS
[1], [2] : c6h588096p3481394
[3] : c6h4830p15309
[4], [5] : c6h581954p3438879

他にもいろいろ
0471132人目の素数さん
垢版 |
2017/08/10(木) 00:03:36.94ID:ZcMNVdrv
[出典不明]
実数 a,b,c,x,y,z が ax-2by+cz=0 かつ ac > b^2 > 0 をみたすとき、y^2 ≧ xz を示せ。

こういう掴みどころのない問題は、改造や類題を作りにくいので困る。 ('A`)ヴォエァ!
0472132人目の素数さん
垢版 |
2017/08/10(木) 01:44:43.71ID:DPXWgKrx
>>471
xz≦0 のときは明らか。
xz>0 のとき
4{bbyy -(ax)(cz)}≧(2by)^2 -(ax+cz)^2 = -(ac-2by+cz)(ac+2by+cz)= 0,
∴ yy ≧(ac/bb)xz ≧ xz,
0473132人目の素数さん
垢版 |
2017/08/10(木) 02:37:02.72ID:DPXWgKrx
>>467
(2)
 △の3辺を切る円はその内接円より大きい、を認めよう。
 △の各辺の中点を通る円を考える。
 この円は半径R/2であるが、△の3辺を切る。
 R/2 ≧ r
 (清水多門氏による)

文献[3]、p.7-8 例題4 >>2
0484132人目の素数さん
垢版 |
2017/08/10(木) 02:47:00.98ID:ZcMNVdrv
>>414
> 実数 a,b,c,d が a+b+c+d=0, a^2+b^2+c^2+d^2=100 をみたすとき、
> a^3+b^3+c^3+d^3 のとりうる値の範囲を求めよ。

追加問題 : 同じ条件の下で、aのとりうる値の範囲を求めよ。


     |
 \  __  /
 _ (m) _ピコーン
    |ミ|
 /___\ 
 ./  ≧ \ 
 |::::  \ ./ | 
 |::::: (● (● | < 改造せずにはいられない!
 ヽ::::... .ワ.....ノ   (閃いたが、簡単過ぎる…)
   人つゝ 人,,         
  Yノ人 ノ ノノゞ⌒〜ゞ    
 ノ /ミ|\、    ノノ ( 彡
`⌒  .U~U`ヾ    丿
         ⌒〜⌒
0486132人目の素数さん
垢版 |
2017/08/10(木) 02:58:19.02ID:DPXWgKrx
>>449 (4)
チェビシェフにより
(左辺)≧ a/{(1+b)(1+c)}+ b/{(1+c)(1+a)}+ c/{(1+a)(1+b)}
 ={a(1+a)+ b(1+b)+ c(1+c)}/{(1+a)(1+b)(1+c)}
 ≧(s+t)/(1+s+t+u),
 ≧ 3/4,
∵題意より u=abc=1 ゆえ s+t≧3{u^(1/3)+u^(2/3)}= 6,
0487132人目の素数さん
垢版 |
2017/08/10(木) 03:13:12.22ID:ZcMNVdrv
>>414
> 実数 a,b,c,d が a+b+c+d=0, a^2+b^2+c^2+d^2=100 をみたすとき、
> a^3+b^3+c^3+d^3 のとりうる値の範囲を求めよ。
>
> 追加問題 : 同じ条件の下で、aのとりうる値の範囲を求めよ。

さらに追加 : 同じ条件の下で、ab+bc+cd+da のとりうる値の範囲を求めよ。


           /⌒\ っ   /\
          /'⌒'ヽ \ っ/\  |
          (●.●) )/   |: | 
           >冊/  ./     |: /
         /⌒   ミミ \   〆
         /   / |::|λ|   
         |√7ミ   |::|  ト、  
         |:/    V_ハ   
        /| i         | 
         и .i      N 
          λヘ、| i .NV 
            V\W  
     |
 \  __  /
 _ (m) _ピコーン
    |ミ|
 /___\ 
 ./  ≧ \ 
 |::::  \ ./ | 
 |::::: (● (● | < なんか降りてきた!
 ヽ::::... .ワ.....ノ    今夜は冴えてるぜ!
   人つゝ 人,,         
  Yノ人 ノ ノノゞ⌒〜ゞ    
 ノ /ミ|\、    ノノ ( 彡
`⌒  .U~U`ヾ    丿
         ⌒〜⌒
0500132人目の素数さん
垢版 |
2017/08/10(木) 13:59:34.75ID:DPXWgKrx
>>484
a=-(b+c+d)を代入して
aa = (-b-c-d)^2 ≦ 3(bb+cc+dd)= 3(100-aa),
aa ≦ 75,
|a| ≦ 5√3,

>>487
(a+c)(b+d)= -(a+c)^2 = -(b+d)^2 ≧ -(aa+cc) -(bb+dd) = -100,
-100 ≦ (a+c)(b+d)≦ 0,
等号成立は(a,b,c,d)=(5,-5,5,-5)(5,5,-5,-5)など。
0514◆2VB8wsVUoo
垢版 |
2017/08/10(木) 21:57:11.66ID:JHmEReZW
★★★馬鹿板は悪い習慣であり、大脳が劣化します。なので早く止めましょう。★★★

0526132人目の素数さん
垢版 |
2017/08/11(金) 00:32:25.04ID:UlqqGaeP
ネタギレだな
興奮する問題が無い
0528◆2VB8wsVUoo
垢版 |
2017/08/11(金) 00:58:40.57ID:ToUPXODc
♪♪♪もう良い子は寝る時間です。そやし馬鹿板は止めて、また明日にしましょう。♪♪♪

ケケケ¥
0539132人目の素数さん
垢版 |
2017/08/11(金) 12:57:10.89ID:OXujv9yn
>>467 (1)を改造...

三角形の辺長を a,b,c、面積をSとするとき、(1/3)(a+b+c)^2 ≧ (4√3)S.

(証明3)
b+c-a=A, c+a-b=B, a+b-c=C とおく。
(1/3)(a+b+c)^2
=(1/3)(A+B+C)^2
≧ √{3(A+B+C)ABC} (← AM-GM)
=(4√3)S,


三角形の辺長を a,b,c、面積をSとするとき、ab+bc+ca ≧ (4√3)S.

(証明6)
b+c-a=A, c+a-b=B, a+b-c=C とおく。
ab+bc+ca = aa+bb+cc -{(a-b)^2 +(b-c)^2 +(c-a)^2}/2
≧ aa+bb+cc -(a-b)^2 -(b-c)^2 -(c-a)^2
= AB+BC+CA
≧ √{3(A+B+C)ABC}
=(4√3)S,
0540◆2VB8wsVUoo
垢版 |
2017/08/11(金) 12:59:40.46ID:ToUPXODc
★★★馬鹿板は悪い習慣であり、大脳が劣化します。なので早く止めましょう。★★★

0541132人目の素数さん
垢版 |
2017/08/11(金) 16:26:36.35ID:XzY0B0Bq
a, b, c >0 かつ abc=1 のとき、

(1) [AYIN 2012.09]
(a+b)/(ab+a+b) + (b+c)/(bc+b+c) + (c+a)/(ca+c+a) ≧ 2

(2) [1997 Romania]
(a^3+b^3)/(ab+a^2+b^2) + (b^3+c^3)/(bc+b^2+c^2) + (c^3+a^3)/(ca+c^2+a^2) ≧ 2

(3) [1996 IMO shortlist.A1]
ab/(ab+a^5+b^5) + bc/(bc+b^5+c^5) + ca/(ca+c^5+a^5) ≦ 1

----------------------------------------------------

[1] (3)だけ向きが逆。もしかして (1)(2)(3) すべて最大値と最小値があるかな?

[2] 分母が ab+a^n+b^n のタイプで、他に類題ないかな?


     /⌒\ っ   /\
    /'⌒'ヽ \ っ/\  |
    (●.●) )/   |: | 
     >冊/  ./     |: /
   /⌒   ミミ \   〆
   /   / |::|λ|    |
   |√7ミ   |::|  ト、   |
   |:/    V_ハ   |
  /| i         | ∧|∧
   и .i      N /  ヽ) きりがないでござる…
    λヘ、| i .NV  |   | |
      V\W   ( 、 ∪
              || |
              ∪∪
0542132人目の素数さん
垢版 |
2017/08/11(金) 16:46:26.28ID:XzY0B0Bq
>>467>>539
さらに改造。というか、コレクションに纏め済みでござった。

三角形の辺長 a, b, c、面積 S、外接円の半径 R、内接円の半径 r に対して、

9R^2 ≧ a^2 + b^2 + c^2 ≧ (1/3)(a+b+c)^2 ≧ ab+bc+ca ≧ 3(abc)^(2/3) ≧ (4√3)S ≧ 36r.


     凵@    ○   ∇ 、___,、´`゙;~、  ';冫 ☆
           ┏  ━ゝヽ''/  ≧ \━〆A!゚━━┓。
 ╋┓"〓┃  < ゝ\',冫。' |::::  \ ./ |゛△│´'´,.ゝ'┃.      ●┃ ┃┃
 ┃┃_.━┛ヤ━━━━━━|::::: (● (● |━━━━━━━━━  ━┛ ・ ・
        ∇  ┠─Σ-  ヽ::::... .ワ.....ノ  冫 そ',´; ┨'゚,。
           .。冫▽ <   ⊂     ./⊃     乙  ≧   ▽
         。 ┃   Σ   (⌒ゞ ,l, 、''  │   て く
           ┠─ム┼   ゝ,,ノ ノゝ. 、,, .┼ ァ Ζ┨ ミo''`
         。、゚`。、   i/   レ' o。了 、'' ×  个o
        ○  ┃   `、,~´+√ ▽   ',!ヽ.◇    o┃
            ┗〆━┷ Z,.' /┷━''o ヾo┷+\━┛,゛;
       ヾ   凵@              '、´    ∇
0543132人目の素数さん
垢版 |
2017/08/11(金) 17:08:13.18ID:XzY0B0Bq
>>467 >>539 >>542
さらに行けそうだぜ! ヒャッハー!
http://forumgeom.fau.edu/FG2005volume5/FG200519index.html

9abc/(a+b+c) ≧ (4√3)S が成り立つらしい (証明は未だ読んでいない)

AM-GMから直ちに >>542 とドッキングさせられるぜ! ヒャッハー!

9R^2 ≧ a^2 + b^2 + c^2 ≧ (1/3)(a+b+c)^2 ≧ ab+bc+ca ≧ 3(abc)^(2/3) ≧ 9abc/(a+b+c)≧ (4√3)S ≧ 36r.



     _  ())二) )) 、,r:ニヽ  いいぞ ベイべー!
 @ニ===)二二ニニ)('A` ))  不等式を収集し証明する奴は 不等式ヲタだ!!
     ^ ̄" フ\''|ノ=ノ-(  )   不等式を改造し拡張する奴は よく訓練された不等式ヲタだ!!
         _/  \_   L L   ホント不等式はハァハァするぜ! フゥハハハーハァー
0554132人目の素数さん
垢版 |
2017/08/11(金) 18:20:48.72ID:OXujv9yn
>>467 (1)>>539 を再改造…
>>541
(2)
(aa-ab+bb)/(aa+ab+bb)≧ 1/3,など。
(左辺)≧ 2(a+b+c)/3 ≧ 2(abc)^(2/3)= 2,


(3)
ab +a^5 +b^5 = aabbc +a^5 +b^5 ≧ aabb(a+b+c)= ab(a+b+c)/c,
IMO-1996 予選
文献[9]佐藤、演習問題1.15


>>543
abc =(A+B)(B+C)(C+A)/8 ≧(A+B+C)(AB+BC+CA)/9,
∴ ab+bc+ca ≧ 9abc/(a+b+c)≧ AB+BC+CA
>>539 により成立。

きりがないでござるよ…
0555132人目の素数さん
垢版 |
2017/08/11(金) 18:50:48.93ID:XzY0B0Bq
>>554
むむむ、再改造とは 恐るべし不等式ヲタ…

List of triangle inequalities
https://en.wikipedia.org/wiki/List_of_triangle_inequalities



彳b” ,イ云”   ,.ッ          |  ィ1 |l  |       、   ,. '´
レ/   チa     rf少        [> |||| ||  |       迅  /
rf   fリイ     {ヲ        _レ-ー、|__ト-、     什 (      む
lト   {iヌ    {iヌ      _/´,.´ ,.  .., 、 フ _ヽ、  ノ糸 _,)    む
斗   弋z    弋z,.     〃_` /',ニ=ュ> lxニ∠ヽ|_ ァzソ (       む
も、  `マチtz,        { G レ‐、ゝー"´=ゝ一'‐, L     `┐
ミマ辷   ` =z.,,__      ! ,r〉 ,二_,.{,_,}二,,,..、 .}     ゝ
 ` t述シtr、         {`-”し',. '"´`ゝv, ィ/´゛ヽレ'      `つ
     `ー≧= ‐ .,,,    ト,  || ゝ ひ フ  / てソj |:|       〈 ⊂´ ̄ ̄
` 爻ミzz,,           | | . || , '´ ̄   |` ̄''` i,|       ,)r'"
   `弋≧=ー'       |  || J      ,._|    .//      /"
               ,/、.  ||   、_,,,.--、_, //
              ,.r' !、  ̄ ゝ....,,,,____,,,/,1
         ,,.. ‐'フ´   >`、「 0        C.〕、
       ,. < ``、、   /'  ,.ヘ>========< \‐- .._
0568132人目の素数さん
垢版 |
2017/08/12(土) 00:51:30.41ID:rvCA1oPA
>>389 >>515

△ABC における重心座標を考える。
 ↑D = L・↑A + m・↑B + n・↑C, L+m+n=1,

(v,w)=((Lp+mr+nt)/(L+m+n),(Lq+ms+nu)/(L+m+n))

(Dが△ABCの内部または周上) ⇔ 0 ≦ L,m,n

∴ AM-GM により
 x^v・y^w ≦{L(x^p)(y^q) + m(x^r)(y^s) + n(x^t)(y^u)}/(L+m+n)
     ≦ (x^p)(y^q) + (x^r)(y^s) + (x^t)(y^u),

(Dが△ABCの外部) ⇔ min{L,m,n}<0
 さて、どうする?
0579132人目の素数さん
垢版 |
2017/08/12(土) 03:30:21.66ID:hiSFFC3j
不等式ではなくって、等式なんだけど、
>>467の本 : 佐久間一浩、『高校数学と大学数学の接点』
を立ち読みしてきたときに見つけた問題を。

Σ[n=1 to ∞] (15n^2 - 30πn^4 + 8π^2 n^6)*e^(-πn^2) = ?


あと、名前の付いた等式を一つ。(只の式変形で出るので面白くはないが…)

ヒルツェブルフの等式 : x/ tanh x = 2x/(e^(2x)-1) + x
0580132人目の素数さん
垢版 |
2017/08/12(土) 11:28:19.55ID:rvCA1oPA
>>388 (4)

(i) >>432

(ii) OA=OB=OC とし、Oから平面ABCに垂線OHを下し、z軸とする。
  A,B,C の天頂角をθとおくと、OH =|OA|・cosθ,etc.

 2平面 OAH と OBH のなす角(二面角)を ∠AHB = φとおく。
 cos(∠AOB)=(OA・OB)/|OA||OB|=(cosθ)^2 +(sinθ)^2 cosφ ≧ cosφ,
∴ ∠AOB ≦ φ = ∠AHB,
循環的にたす。
0582132人目の素数さん
垢版 |
2017/08/13(日) 16:43:33.64ID:/or+kDcE
>>541 (1)

(a+b)/(ab+a+b) = (a+b)c/{1+(a+b)c}= z/(1+z),

通分して
(1+x)(1+y)z +(1+x)y(1+z)+ x(1+y)(1+z)- 2(1+x)(1+y)(1+z)
 = -2 -(x+y+z) +xyz,
 = -2 -2(ab+bc+ca)+ abc(a+b)(b+c)(c+a)
={1 - (abc)^2}+(ab+bc+ca-3)+(ab+bc+ca){abc(a+b+c) -3}
 ≧ 0,
0583132人目の素数さん
垢版 |
2017/08/14(月) 03:30:28.48ID:DhVyRLdl
>>449 >>455

(2)
(1+ab)/(1+a)= (1+c)/{c(1+a)},etc.
 AM-GM する。

>>455 とほとんど同じだ....

(3)
 1/(1+a)+ 1/(1+b)+ 1/(1+c)
 ≧ 1/(1+a+ab)+ 1/(1+b+bc)+ 1/(1+c+ca)
 = x/(x+y+z)+ y/(y+z+x)+ z/(z+x+y)
 = 1,

 bc/(1+a) + ca/(1+b) + ab/(1+c) ≧ 3/2,
通分して
 bc(1+b)(1+c)+ ca(1+c)(1+a)+ ab(1+a)(1+b)-(3/2)(1+a)(1+b)(1+c)
 = t +(st-3u)+(tt-2su)-(3/2)(1+s+t+u)
 ={(s-3)t + s(t-3)}/6 +(2s+3)(u-1)/2 + 2(st-9u)/3 +(tt-3su)
 ={(s-3)t + s(t-3)}/6 +(2s+3)(u-1)/2
 ≧0,    (← s≧3、t≧3、u=1)
0584132人目の素数さん
垢版 |
2017/08/14(月) 14:19:59.12ID:2wTFMFcz
>>543-544
> 9R^2 ≧ a^2 + b^2 + c^2 ≧ (1/3)(a+b+c)^2 ≧ ab+bc+ca ≧ 3(abc)^(2/3) ≧ 9abc/(a+b+c)≧ (4√3)S ≧ 36r.

書き直すと
(√3)R ≧ (a^2+b^2+c^2)/3 ≧ AM ≧ √{(ab+bc+ca)/3} ≧ GM ≧ √|3abc/(a+b+c)} ≧ 2√(S/√3) ≧ (2√3)r.

>>544より、√|3abc/(a+b+c)} ≧ √|(AB+BC+CA)/3}.

ところで、(ab+bc+ca)^2 - 3abc(a+b+c) ≧ 0 より、√|3abc/(a+b+c)} ≧ HM.

そこで気になるのは、2√(S/√3)、HM、√|(AB+BC+CA)/3} の大小だけど、定まるかな?
 
    /⌒ヽ
  /⌒  ・ >
  E ̄U) ε | きりがないでござる
  E ̄∩) ・ >
゛゛゛゛゛゛゛゛゛゛゛゛゛゛゛゛゛゛゛゛゛゛
0585132人目の素数さん
垢版 |
2017/08/14(月) 16:40:29.61ID:2wTFMFcz
数学文化という雑誌に不等式の特集があるというタレ込みがあったので買ってきた。未だ目を通していない。
0586132人目の素数さん
垢版 |
2017/08/14(月) 21:52:11.78ID:DhVyRLdl
>>449
(3)下
チェビシェフで
(左辺)= 1/{a(1+a)}+ 1/{b(1+b)}+ 1/{c(1+c)}
   ≧ 1/{a(1+b)}+ 1/{b(1+c)}+ 1/{c(1+a)}
よって、次の問題に帰着する。

〔問題3.93〕
 1/{a(1+b)}+ 1/{b(1+c)}+ 1/{c(1+a)}≧ 3/(1+abc),

 バルカンMO-2006
 文献[9] 佐藤(訳)、問題3.93

左辺に 1+abc を掛ける。
 (1+abc)/{a(1+b)}= (1+a)/{a(1+b)}-1 + b(1+c)/(1+b),etc.
巡回的に AM-GM すると
(1+abc)(左辺)≧ 3(1/G -1 +G)
 = 3(1-G+GG)/G
 = 3(1+GGG)/{G(1+G)}.
∴ (左辺)≧ 3/{G(1+G)},
ここに G=(abc)^(1/3)
0587132人目の素数さん
垢版 |
2017/08/15(火) 00:00:45.18ID:CDzXTDus
>>584

(AB+BC+CA)/3 ≧ √{(A+B+C)ABC/3} =(4/√3)S, >>554
(HM)^2 ≧(4/√3)S
にて御座候。

HM と √{(AB+BC+CA)/3}の大小は不定と思われ...

き、きりがねぇ。。。
0589132人目の素数さん
垢版 |
2017/08/15(火) 11:56:07.50ID:MRdTx6vq
>>584-585より、
(√3)R ≧ (a^2+b^2+c^2)/3 ≧ AM ≧ √{(ab+bc+ca)/3} ≧ GM ≧ √|3abc/(a+b+c)}≧ HM ≧ 2√(S/√3) ≧ (2√3)r

ところで、三角形の辺長a,b,cに対して、2(ab+bc+ca) > (1/2)(a+b+c)^2 > a^2+b^2+c^2 だから、
{√2(ab+bc+ca)}/3 > (a+b+c)/(3√2) > (a^2+b^2+c^2)/3

合体させて、
{√2(ab+bc+ca)}/3 > (a+b+c)/(3√2) > (a^2+b^2+c^2)/3 ≧ AM ≧ √{(ab+bc+ca)/3} ≧ GM ≧ √|3abc/(a+b+c)}≧ HM ≧ 2√(S/√3) ≧ (2√3)r

さて、(√3)R はどこに入るのだろう?



    ('A`)  出口が見えないでござる
    ノ ノ)_
0590132人目の素数さん
垢版 |
2017/08/15(火) 12:37:59.27ID:MRdTx6vq
>>589
> >>584-585より、
> (√3)R ≧ (a^2+b^2+c^2)/3 ≧ AM ≧ √{(ab+bc+ca)/3} ≧ GM ≧ √|3abc/(a+b+c)}≧ HM ≧ 2√(S/√3) ≧ (2√3)r
>
> ところで、三角形の辺長a,b,cに対して、2(ab+bc+ca) > (1/2)(a+b+c)^2 > a^2+b^2+c^2 だから、

GMの左側から合体させたら、
√{(ab+bc+ca)/3} > (a+b+c)/(2√3) > √|(a^2+b^2+c^2)/6} ≧ GM/(√2)
√{(ab+bc+ca)/3} ≧ GM ≧ √|3abc/(a+b+c)}≧ HM ≧ 2√(S/√3) ≧ (2√3)r

この2つは合体は無理そうかな。上側はGMより小さくなってるようだし…
0591132人目の素数さん
垢版 |
2017/08/15(火) 13:03:49.64ID:CDzXTDus
>>589

{√2(ab+bc+ca)/3}>(a+b+c)/√6 > √{(aa+bb+cc)/3}≧ AM

>>590

正△でも等号不成立なので、無理そうでござる。
0592132人目の素数さん
垢版 |
2017/08/16(水) 07:17:47.30ID:QnvYtidY
>>588
(HM)^2 ≧(4/√3)S
⇔ (3√3)(abc)^2 ≧ 4(ab+bc+ca)^2 √{s(s-a)(s-b)(s-c)}
⇔ (3√3) sin A sin B sin C ≧ 2 (sin A sin B + sin B sin C + sin C sin A)^2

この証明は難しいのでは?
a, b, c で表しても、sin で表してもややこしい。
レムスで削り落としても、まだ複雑な形でござる…

Lehmu's inequality : abc ≧ (s-a)(s-b)(s-c)

           , /    ,
        ,   / /   ,   /  ,
          / '^メ-' ─/- 、   / ,
       ∠r  _,゛_ /  , ヽ/__/ モウ ダメポ…
        ''ヽ'_・.ノ` ' r/、 ヘ /‐’
       ./ " j 厂゙j | レ_`> j__ /
        '  .:‘::'ニ‘.:‐'´─゙.:´一’ 
0593132人目の素数さん
垢版 |
2017/08/16(水) 08:03:22.63ID:QnvYtidY
>>592
左辺の係数間違ごうとる

(HM)^2 ≧(4/√3)S
⇔ (9√3)(abc)^2 ≧ 4(ab+bc+ca)^2 √{s(s-a)(s-b)(s-c)}
⇔ (9√3) sin A sin B sin C ≧ 2 (sin A sin B + sin B sin C + sin C sin A)^2
0596132人目の素数さん
垢版 |
2017/08/18(金) 01:02:25.04ID:90S02hzN
>>588-595

HM^2 と (4/√3)S の大小
1辺だけが短い楔状△の場合は不成立のようでござる。

手間取らせて、すまぬ。
0597132人目の素数さん
垢版 |
2017/08/18(金) 11:06:43.92ID:WHydeLcz
>>596
さんくす。(a,b,c)=(1/3,1,1)で、HMの方が小さくなりますね。

[疑問] HM ≧ (2/√3)r は成り立つか?

b=c=1、0<a<2でWolfram先生にグラフを書かせたら、0以上っぽいので、基本対称式で表すと、
(HM^2 - 12r)/3 の分子
= 3su^2+s^3t^2-4st^3+8t^2u
= (s^2-3t)st^2 - (t^2-3su)u
= 正 - 正
で、この方法では失敗でござった。
0599132人目の素数さん
垢版 |
2017/08/18(金) 17:50:32.42ID:WHydeLcz
>>449>>583>>586
> a, b, c >0 かつ abc=1 のとき、bc/(1+a) + ca/(1+b) + ab/(1+c) ≧ 3/2

>>586
>  1/{a(1+b)}+ 1/{b(1+c)}+ 1/{c(1+a)}≧ 3/(1+abc),
>
>  バルカンMO-2006、文献[9] 佐藤(訳)、問題3.93


似たような不等式を見つけた。
[IMO 1995 第2問] http://www.cs.cornell.edu/~asdas/imo/imo/imo95.html

1/(a^3*(b+c)) + 1/(b^3*(c+a)) + 1/(c^3*(a+b)) ≧ 3/2.
0600132人目の素数さん
垢版 |
2017/08/18(金) 18:07:26.70ID:WHydeLcz
>>449>>455>>583
a, b, c >0 かつ abc=1 のとき、3 > 1/(1+a)+ 1/(1+b)+ 1/(1+c) > 1

上限を厳しく評価するには、どういう考え方でやればいいんでせうか?
0601132人目の素数さん
垢版 |
2017/08/18(金) 22:17:20.07ID:/k+bKW+I
>>600
f(x)=1/(1+e^x)
x+y+z=0 なる実数 x, y, z に対して f(x)+f(y)+f(z) の上限を調べればよい
f は x<=0 で狭義凸だから LCF から y=x, z=-2x のときの上限を調べれよばよい
sup 2f(x)+f(-2x) = 2
よって上限は 2
0602132人目の素数さん
垢版 |
2017/08/19(土) 03:06:30.81ID:HQ7H9Ohy
>>599
x=1/a、y=1/b、z=1/c とおくと、xyz=1
S = xx/(y+z)+ yy/(z+x)+ zz/(x+y)
≧ (x+y+z)^2 /{(y+z)+(z+x)+(x+y)}  (←コーシー)
=(x+y+z)/2
≧3/2,
 文献[9] 佐藤(訳)例1.4.9

>>600
 a,b,… のうち最小のものをmとおきます。(m≦1)
 1より大きい2要素 p,q があったときは
 (p, q)→(m, pq/m)と置き換えてみます。
 このとき相乗平均は変わらず、
 (m + pq/m)-(p+q)= (p-m)(q-m)/m ≧ 0 ゆえ左辺は
 1/(1+m)+ 1/(1+pq/m)- 1/(1+p)- 1/(1+q)
 = (2+m+pq/m)/{(1+m)(1+pq/m)}-(2+p+q)/{(1+p)(1+q)}
 =(2+m+pq/m)/{1+(m+pq/m)+pq}-(2+p+q)/{1+(p+q)+pq}
 = -(pq-1)/{1+(m+pq/m)+pq}+(pq-1)/{1+(p+q)+pq}
 ≧0
増大します。
0606132人目の素数さん
垢版 |
2017/08/19(土) 05:22:53.63ID:Q+nr/ATk
Arithmetic Compensation Theorem (AC-Theorem)
Equal Variable Theorem (EV-Theorem)
Half Convex Function Theorem (HCF-Theorem)
Left Concave Function Theorem (LCF-Theorem)
Right Convex Function theorem (RCF-Theorem)
Left Convex-Right Concave Function Theorem (LCRCF-Theorem)
Single Inflection Point Theorem (SIP-Theorem)
Strong Mixing Variables Theorem (SMV-Theorem)

GC-Theorem (文献[8] 安藤 P.197)は何の略?
0608132人目の素数さん
垢版 |
2017/08/19(土) 13:16:25.17ID:HQ7H9Ohy
>>597 >>598

s^3 -4st +9u = a(a-b)(a-c)+ b(b-c)(b-a)+ c(c-a)(c-b),
tt-3su = bc(a-b)(a-c)+ ca(b-c)(b-a)+ ab(c-a)(c-b),
より
(s^3 -4st +9u)tt - (tt-3su)u = P(a-b)(a-c)+ Q(b-c)(b-a)+ R(c-a)(c-b),
ここに
P=a(tt-bbcc),Q=b(tt-ccaa),R=c(tt-aabb),
P,Q,R≧0 かつ(P,Q,R)(a,b,c)は同順序なので Schurの拡張で成立..
0609132人目の素数さん
垢版 |
2017/08/19(土) 14:37:54.57ID:Q+nr/ATk
>>608
Schurの拡張について詳しく教えてください。

f : R→(0,∞) が単調増加 or 単調減少のとき、a, b, c∈R に対して、
f(a)(a-b)(a-c) + f(b)(b-c)(b-a) + f(c)(c-a)(c-b) ≧0

というのは知っているけど、この場合は f(x) が f(a,b,c)の3変数関数で、
同順序ならokってのが、ピンと来ない…
0610132人目の素数さん
垢版 |
2017/08/19(土) 15:39:17.29ID:Q+nr/ATk
>>600-602
> a,b,c>0, abc=1のとき、1 < 1/(1+a)+ 1/(1+b)+ 1/(1+c) < 2

>>583の真似をして上限を出してみたなり。 ( ゚∀゚) ウヒョッ!

1/(1+a)+ 1/(1+b)+ 1/(1+c)
= 1 + (1-ab)/(1+a+b+ab) + 1/(1+c)
< 1 + 1/(1+ab) + 1/(1+c)
= 1 + c/(1+c) + 1/(1+c)
= 2
0612132人目の素数さん
垢版 |
2017/08/19(土) 16:34:50.44ID:Q+nr/ATk
つまり、不等式を証明するだけなら、そのやり方でよいが、上限、下限であることを言うには、
a, b → +0 や a.,b → ∞ を調べて、限界値であることを確認しろってことかな?
0613132人目の素数さん
垢版 |
2017/08/19(土) 17:26:48.38ID:Qk9aUlzH
うん
でもその解き方でもa,b->0考えれば最適であることは言えるかもね
0614132人目の素数さん
垢版 |
2017/08/19(土) 17:44:45.38ID:Q+nr/ATk
>>449
(3)下を、Jensen + AMGM で。

f(x) = 1/(a+a^2) は下に凸だから、
左辺
= f(a) + f(b) + f(c)
≧ 3*f( (a+b+c)/3 )
≧ 3*f( (abc)^(1/3) )
= 3*f(1)
= 3/2
0615132人目の素数さん
垢版 |
2017/08/19(土) 18:43:45.36ID:Qk9aUlzH
>>614
f は単調増加じゃないから f( (a+b+c)/3 ) >= 3*f( (abc)^(1/3) ) は成り立たない
むしろ逆の不等号が成り立つ
0617132人目の素数さん
垢版 |
2017/08/19(土) 20:33:36.53ID:C7tE2SmP
不等式を極めるとなんかいいことがある?
0628132人目の素数さん
垢版 |
2017/08/19(土) 22:28:51.70ID:HQ7H9Ohy
>>597 >>598

a,b,c が△の辺長の場合は Ravi変換で簡単でござるよ。 >>594
b+c-a=A, c+a-b=B, a+b-c=C, a+b+c=A+B+C.

HM = 3abc/(ab+bc+ca)
=(3/2)(A+B)(B+C)(C+A)/{(A+B+C)^2 +(AB+BC+CA)}
≧(4/3)(A+B+C)(AB+BC+CA)/{(4/3)(A+B+C)^2}
=(AB+BC+CA)/(A+B+C)
≧(4√3)S/(a+b+c)
=(2√3)r,

したがって a,b,c>0 で成立するかがミソのようでござる… >>608
0640132人目の素数さん
垢版 |
2017/08/20(日) 11:40:27.25ID:XEX21MRP
疑問でござる。

(1)
a, b, c >0 の相乗平均を G とおくとき、a/(b+G) + b/(c+G) + c/(a+G) ≧ 3/2 は成り立つか?

(2)
上式で、右辺の定数をGを含む式に変えられないか? たとえば、3/(1+G) みたいな感じで。

(3)
a, b, c >0、s = a+b+c、t = ab+bc+ca、u = abc に対して、s^3u - t^3 ≧0 は成り立つか?
0652132人目の素数さん
垢版 |
2017/08/20(日) 18:47:22.79ID:XEX21MRP
>>628
ようやく理解。ところでRavi変換は (b+c-a)/2 = x、… なのでは?

基本対称式を使って、力任せに証明してみた。
a, b, c の基本対称式を s, t, u とおくと、
HM^2 - (2√3*r)^2 = 3{3s(st-u)^2 - 4u(s^2+t)^2}/{s(s^2+t)^2}
分子 = u(s^2t+3su-4t^2) + s^2(st^2-4s^2u+3tu) + 2s^2t(st-9u) ≧0

週末が始まったと思ったら、もう終わっていたでござる… ('A`)
0654132人目の素数さん
垢版 |
2017/08/20(日) 18:56:12.70ID:XEX21MRP
>>652
何度もすまぬ。
Ravi変換 (b+c-a)/2 = x、…をしてから、x, y, z の基本対称式 s, t, u を使ったのでござった。
0665132人目の素数さん
垢版 |
2017/08/20(日) 22:50:23.44ID:mA3fdDEU
>>609
〔Schur 不等式の拡張〕
P,Q,R≧0 かつ(P,Q,R)(a,b,c)が同順または逆順ならば
 P(a-b)(a-c)+ Q(b-c)(b-a)+ R(c-a)(c-b)≧ 0.

(略証)
bはa,cの中間にあるとしてよい。
 (a-b)(b-c)≧ 0
題意より、P,Q,R≧0 かつ QはP,Rの中間にあるから、
 P-Q+R ≧0
これらより、
 P(a-b)(a-c)+ Q(b-c)(b-a)+ R(c-a)(c-b)
= P(a-b)^2 +(P-Q+R)(a-b)(b-c)+ R(b-c)^2
≧ 0, (終)

いろいろな拡張があり、まとめて Vornicu−Schur 不等式と云うらしい。

詳しくは、ニコニコ大百科の「シューアの不等式」の項を参照


>>640
(1) >>449(1)と同じでつ。
(2)同次式ゆえ、定数でつ。


>>654
それなら、>>652>>628 と同じでつね。
0667132人目の素数さん
垢版 |
2017/08/20(日) 23:03:56.11ID:mA3fdDEU
>>617
専門バカになるでござる。

(ただし、専門を持たぬ只のバカよりは、すこーしマシである。)
0668132人目の素数さん
垢版 |
2017/08/21(月) 09:09:30.43ID:QiJqP8rB
>>628

a,b,c が△の辺長でない場合も簡単でござるよ。

A+B=2c≧0,B+C=2a≧0,C+A=2b≧0,
∴ A,B,Cのうち負となるのは1つだけ。
∴ HM^2 ≧ 0 ≧ 3ABC/(A+B+C),
0669132人目の素数さん
垢版 |
2017/08/21(月) 17:53:20.03ID:8ztbkIZ8
a, b, c >0 かつ abc≧1 のとき、

(1) [2004 ウクライナ、 文献 [9] 佐藤(訳)P.139]
 a^3 + b^3 + c^3 ≧ ab+bc+ca

(2) [2006.3 エレ解、一松信]
 a^2b + b^2c + c^2a ≧ ab+bc+ca

(3) [疑問]
上の左辺 a^3 + b^3 + c^3 と a^2b + b^2c + c^2a の大小は定まるのか?
巡回不等式に有効な手段って何? 真ん中の数を固定して場合分けくらいかな?
0670132人目の素数さん
垢版 |
2017/08/21(月) 22:19:47.87ID:QiJqP8rB
>>669
(abc)^(1/3) = G とおき、AM-GM する。

(1)
 a^3+a^3+b^3 -3aab = (2a+b)(a-b)^2 ≧ 0
 ゆえ、(2)に帰着する。


(2) aab+aab+bbc ≧ 3aG,
 巡回的にたす。


(3) Muirheadの不等式
0671132人目の素数さん
垢版 |
2017/08/21(月) 22:26:23.89ID:QiJqP8rB
>>669
 >>670 の訂正

(2) aab + aab + bbc ≧ 3abG
でござった。

(3) 非対称のときは微妙な場合もあるが、この場合は成立つでござる。
0673132人目の素数さん
垢版 |
2017/08/22(火) 00:50:18.83ID:fGEhoquB
>>2 安藤 [8] に著者のHPのリンクを追加 (まとめwikiは更新済み)
http://www.math.s.chiba-u.ac.jp/~ando/ (著者のページに正誤表+補遺)

Muirhead's inequality は難しくて、>>1のまとめwikiを見たけど挫折。
その後、>>2 安藤 [8] PP.11-14を読んで、なんとか証明は辿れたけど、
簡単な例を作るなどで練習していないから、全く使いこなせない。 ← 今ココ

今が勉強するときなのかもしれないなあ。

  ( ゚д゚ ) ガタッ
  .r   ヾ
__l_l / ̄ ̄ ̄/_
  \/    /
0674132人目の素数さん
垢版 |
2017/08/22(火) 00:57:31.85ID:fGEhoquB
古いmemoを見つけたので、紛失する前に書き込んでおく。
証明は簡単だけど、見た目がよかったので。

〔出典不明〕
A(a,b) = (a+b)/2、G(a,b) = √(ab)、A(a,b,c) = (a+b+c)/3 などと書くことにする。
正の数 a, b, c, d に対して、
A(a,b,c,d) ≧ G(A(a,b,c),A(b,c,d),A(c,d,a),A(d,a,b)) ≧ G(A(a,b).A(a,c).A(a,d).A(b,c).A(b,d).A(c,d).) ≧ G(a,b,c,d)
0675132人目の素数さん
垢版 |
2017/08/22(火) 13:46:51.51ID:yCSUoaY7
>>674
[第6章.151-159]の辺りにござる。

G(A(a,b,c), A(b,c,d), A(c,d,a), A(d,a,b))^4
= (a+b+c)(b+c+d)(c+d+a)(d+a+b)/81
= (sst -su +v)/81,

G(A(a,b), A(a,c), A(a,d), A(b,c), A(b,d), A(c,d))^6
= (a+b)(a+c)(a+d)(b+c)(b+d)(c+d)/64
= (stu -ssv -uu)/64,

A(ab, ac, ad, bc, bd, cd)
= (ab+ac+ad+bc+bd+cd)/6
= t/6,

A(abc, bcd, cda, dab)
= (abc+bcd+cda+dab)/4
= u/4,
0676132人目の素数さん
垢版 |
2017/08/22(火) 15:23:36.14ID:fGEhoquB
>>669(3)
(a^2, b^2, c^2) と (a,b,c) は大小の順が同じだから、
『同順序積の和 ≧ 乱順序積の和 ≧ 逆順除籍の和』 で、
a^3 + b^3 + c^3 ≧ a^2b + b^2c + c^2a
で問題ない蟹?
0677132人目の素数さん
垢版 |
2017/08/22(火) 18:38:27.52ID:fGEhoquB
(1) [1999 Russia]
a, b, c >0 に対して、1 + 3/(ab+bc+ca) ≧ 6/(a+b+c)

(2) [1999 Russia]
a, b, c >0、abc=1 に対して、1 + 3/(a+b+c) ≧ 6/(ab+bc+ca)

(3) [不明]
a, b, c >0、abc=1 に対して、2/(a+b+c) + 1/3 ≧ 3/(ab+bc+ca)
0678132人目の素数さん
垢版 |
2017/08/22(火) 18:49:45.42ID:fGEhoquB
(1) [出典不明]
a, b, c, d >0、abcd=1 とする。
1/(1+ab+bc+ca) + 1/(1+bc+cd+db) + 1/(1+cd+da+ac) + 1/(1+da+ab+bd) ≦ 1

[疑問]
1/(1+ab+bc+cd) + 1/(1+bc+cd+da) + 1/(1+cd+da+ab) + 1/(1+da+ab+bc) だと、どうなるのだろう?
0679132人目の素数さん
垢版 |
2017/08/22(火) 18:56:05.03ID:fGEhoquB
以下、a, b, c >0、abc=1 とする。いずれも出典不明

(1)
(a+b)(b+c)(c+a) + 7 ≧ 5(a+b+c)

(2)
(a+b+c)/3 ≧ {(a^2+b^2+c^2)/3}^(1/5)

(3)
(a-1)/b + (b-1)/c + (c-1)/a ≧ 0

(4)
(a-1)/(b+c) + (b-1)/(c+a) + (c-1)/(a+b) ≧ 0

(5)
(a/c)^2 + (b/a)^2 + (c/b)^2 ≧ 2(a-b)(b-c)(c-a) + 3
-----------------------------------

未整理のmemoの中で abc=1、abcd=1 のタイプは片付いたかも…。

          r〜〜〜〜〜〜〜〜〜
   __    _ノ きりがないでござる・・・
  /__  `ヽ_ ⌒ヽ〜〜〜〜〜〜〜〜〜
  |〈___ノf レ1(
 ,L| しL.し'゙"
 "`  "′
0680132人目の素数さん
垢版 |
2017/08/22(火) 19:09:30.23ID:fGEhoquB
[おまけ]
友愛数みたいな関係でござるな。

(1)
a, b, c >0、a+b+c=3 のとき、a^2 + b^2 + c^2 + abc ≧ 4.

(2)
a, b, c >0、a^2 + b^2 + c^2 + abc = 4 のとき、a+b+c ≦3.
0682132人目の素数さん
垢版 |
2017/08/23(水) 17:00:04.08ID:edu8Brze
>>667
(1)
1 -6/s +3/t =(1 -3/s)^2 + 9/(3t)- 9/ss ≧ 0,  (ss≧3t)

(2)
1 -6/t +3/(su)=(1 -3/t)^2 + 9/(3su)- 9/tt ≧ 0,  (tt≧3su)

(3)
a=b<c のとき不成立(a=b≧c では成立)でござる。
0683132人目の素数さん
垢版 |
2017/08/23(水) 17:04:32.35ID:edu8Brze
>>680
(1)題意より
(左辺)= s(ss-2t)/3 + u
 ={4s^3 + 3(s^3 -4st+9u) + 2(ss-3t)}/27
 ≧(4/27)s^3
 = 4,
セビリアMO-2008改
佐藤(訳)、[9] 問題3.118


(2)
題意より、0<a〜c<2、
(3-a-b-c)(3+a-b-c+bc)=(4-aa-bb-cc-abc)+ 1 -(2-b)(2-c)(b+c-1)
≧ 1 -(2-b)(2-c)(b+c-1)≧0
∵ b+c-1>0 のとき、AM-GMで(2-b)(2-c)(b+c-1)≦1
イランMO-2002、A.16

>>682 (3)
不等号が逆でござった。
0684132人目の素数さん
垢版 |
2017/08/23(水) 22:42:04.81ID:6dHoZEIo
>>679
>>681

(3)
a=y/x, ... とおくだけ

(5)
x=b/a, ..., f(x, y, z)=LHS-RHSとおくと
f(x, y, z) - f(t, t, t) = 3/4 * (x^2+y^2+z^2-xy-yz-zx) >= 0 where t = (x+y+z)/3
よって x = y = z = 1 のとき示せばよいがこれは明らか
0686132人目の素数さん
垢版 |
2017/08/24(木) 00:19:32.68ID:9N+3FV4m
>>677
(1)
1 -6/s +3/t =(1 -3/s)^2 + 9/(3t)- 9/ss ≧ 0,  (ss≧3t)

(2)
1 -6/t +3/(su)=(1 -3/t)^2 + 9/(3su)- 9/tt ≧ 0,  (tt≧3su)

(3)
成り立ったでござる。死んでお詫びを…(AA略
0687132人目の素数さん
垢版 |
2017/08/24(木) 01:23:07.53ID:9N+3FV4m
>>679

(2)
ss =(aa+bb+cc)+ t + t,
s^6 ≧{(aa+bb+cc) +t +t}^3
 ≧ 27(aa+bb+cc)tt
 ≧ 81(aa+bb+cc)su,
∴(s/3)^5 ≧{(aa+bb+cc)/3}u,

〔補題196〕
 (8/27)(a+b+c)^5 ≧ (a+b)(b+c)(c+a)(a+b+c)^2 ≧ 24abc(aa+bb+cc),
を使う。(じゅー)


(4)
チェビシェフで,
箔ッ順序積 ≧ 迫随�マ,
(左辺)≧(1/3)(a+b+c-3){1/(b+c)+1/(c+a)+1/(a+b)}≧0,
0688132人目の素数さん
垢版 |
2017/08/24(木) 03:22:45.56ID:rYRHhAcs
>>687
> 〔補題196〕
>  (8/27)(a+b+c)^5 ≧ (a+b)(b+c)(c+a)(a+b+c)^2 ≧ 24abc(aa+bb+cc),

左側はアッサリ、右側はサッパリ…。

8(a+b+c)^3 - 27(a+b)(b+c)(c+a) = 3(s^3-4st+9u) + 5s(s^2-3t) ≧ 0

(a+b)(b+c)(c+a)(a+b+c)^2 - 24abc(aa+bb+cc) = s^3t - 25s^2u +48tu

--------------------------------------------------

ついでに、過去ログ漁っていて出てきたやつですが、すっきりした証明ができませぬ。
[第6章.908]
a,b,c>0のとき、{(a+b+c)(ab+bc+ca)}^2≧27abc(a^3+b^3+c^3),

{(a+b+c)(ab+bc+ca)}^2 - 27abc(a^3+b^3+c^3)
= s^2t^2 - 27s^3u + 81stu - 81u^2

次数が上がると、s, t, u の不等式のどれを組み合わせるか難しくなる。
0689132人目の素数さん
垢版 |
2017/08/24(木) 10:30:59.65ID:9N+3FV4m
>>688

〔補題196〕の略証
チョト難しいのでSchurの拡張で。
bはa、cの間にあるとする。
(左辺)-(右辺)= P(a-b)(a-c)+ Q(b-c)(b-a)+ R(c-a)(c-b)
 = P(a-b)^2 +(P-Q+R)(a-b)(b-c)+ R(b-c)^2,
P =(b+c)(b+c-a)^2 + 2(a+b+c)(b-c)^2 ≧ 0,
P-Q+R = 2b{(a+c)^2 -6ac+3bb}= 2b{(a+c-3m)^2+3(bb-mm)}≧ 0,
R =(a+b)(a+b-c)^2 + 2(a+b+c)(a-b)^2 ≧0,
ここに、m = min{a,c}、ac=m(a+c-m)

-----------------------------ーーーーーーーーーーーーーーーーーーー--------------------
[第6章.908]の略証

S = aaa+bbb+ccc, T =(ab)^3+(bc)^3+(ca)^3,
p = aab+bbc+cca, q = abb+bcc+caa, u=abc とおく。
pq = T+uS+3uu ≧ 3u(3ST)^(1/3)≧ 3u√(3Su)より、
(左辺)={(a+b+c)(aa+bb+cc)}^2 =(S+p+q)^2 ≧ 9(Spq)^(2/3)≧ 27Su,

Casphy!-不等式2-177 じゅー
0690132人目の素数さん
垢版 |
2017/08/25(金) 00:31:48.00ID:oetrvUQn
>>677 (3)
 st +6Gt -9GGs ≧ 0,

>>679 (1)
st +6u -5GGs ≧ 0,

の特効薬は無いでござるか?(G=(abc)^(1/3))

3次方程式
 X^3 -sX^2 +tX -u=0
の判別式は
 27竸2 = 27{(a-b)(b-c)(c-a)}^2
= 4(ss-3t)^3 - (2s^3 -9st +27u)^2
 =(st-9u)^2 -4(t^3 +s^3・u +27uu -9stu)
 =(st+9u)^2 -4(t^3 +s^3・u +27uu)
 =(st+6GGs +6Gt +9u)^2 -4(t +Gs +3GG)^3,
3つの実根 a,b,c をもつときは
 st+6GGs+6Gt+9u ≧ 2(t+sG+3GG)^(3/2),
と思われるが、さて…
0691132人目の素数さん
垢版 |
2017/08/25(金) 01:15:11.00ID:3FtU8w0T
>>679
>>690
f(a, b, c)=LHS-RHS, a>=c>=b とすると
f(a, b, c)- f(a, t, t) = 1/4 *(2a-b-c) >= 0 where t = (b+c)/2
f(a, b, c) - f(ab, c, 1) = (a-1)(1-b)(c^2+abc+ab+bc+ca+c-5) >=0
よって a = b = c = 1 のとき調べればよいが明らか
0693132人目の素数さん
垢版 |
2017/08/25(金) 04:26:49.66ID:Yhp4f37o
>>690
f(X) = X^3 -sX^2 +tX -u
f'(X) = 3X^2 - 2sX + t

AN-GMより f'(X)の判別式 D/4 = s^2-3t ≧0
f'(X)=0の解α,βは、α+β, αβ>0 より、α,β>0
また f(0)=-u<0

グラフを考えると、f(X)=0が正の解a, b, cをもつ条件は f(α)f(β)≦0
f(α)f(β) = -(s^2t^2 - 4s^3u +18stu - 4t^3 -27u^2) ≦0
∴ s^2t^2 - 4s^3u +18stu - 4t^3 -27u^2 ≧0

残念無念…
s, t, u に関する既知の不等式が出てきただけでござった。
s^2t^2 - 4s^3u +18stu - 4t^3 -27u^2 = {(a-b)(b-c)(c-a)}^2

  ('A`) ,..;:〜''"
 ノ( ヘヘ ,,.、;;:〜'''
0694132人目の素数さん
垢版 |
2017/08/25(金) 17:27:20.09ID:oetrvUQn
>>677 (3) が成立つとする。
 2/s + 1/3 ≧ 3/t,
または
 t ≧ 9s/(s+6),
一方、9ss -(s+6)(5s-6)= 4(s-3)^2 ≧ 0 より
 9s/(s+6)≧(5s-6)/s,
したがって
 t ≧(5s-6)/s,
または
 st + 6 ≧ 5s  >>679(1)

それぢゃ、>>677(3)はどうするか?
0695132人目の素数さん
垢版 |
2017/08/25(金) 19:30:02.99ID:Yhp4f37o
Schur's inequality を対称性を崩さずに証明するときの以下の変形は、どうやって思いつくんでしょうか?

F_1 = xy(x+y)(x-y)^2/{(y+z)(z+x)} + yz(y+z)(y-z)^2/{(z+x)(x+y)} + zx(z+x)(z-x)^2/{(x+y)(y+z)}

F_2 = {(x+y-z)2(x-y)^2 + (y+z-x)2(y-z)^2 + (z+x-y)2(z-x)^2 }/2
0697132人目の素数さん
垢版 |
2017/08/26(土) 01:33:14.23ID:MEky4IFO
[疑問1]
Schur's inequality を対称性を崩さずに証明できるのは、n=0,1,2,3 以外には知られていないのかな?
検索の仕方が下手なのか見当たらんでござる。

[疑問2]
>>677のように、同次でない不等式の証明で、お決まりのテクニックって何じゃらほい?
条件式を使って無理やり同時にして、基本対称式の不等式を利用するくらいしか思いつかないけど、
この問題では、条件式を使っても3乗根が現れて大変だし…
0698132人目の素数さん
垢版 |
2017/08/26(土) 02:00:02.17ID:a5WQhO5r
>>695 >>697 [1]
拙者にも分かりませぬ。
F_(n+3)=(x+y+z)F_(n+2)-(xy+yz+zx)F_(n+1)+ xyz F_n
では対称性は崩れませぬが、うまく証明できるのか疑問だし。
0699132人目の素数さん
垢版 |
2017/08/26(土) 02:32:02.40ID:MEky4IFO
>>698
> F_(n+3)=(x+y+z)F_(n+2)-(xy+yz+zx)F_(n+1)+ xyz F_n

ちょうど今、その等式を導いたとこでござる。
それから F_1 を対称性を保つように変形中に次式が出てきて、Wolfram先生に確認してもらった。

F_1
= (1/2){(x^2+y^2-z^2)(x-y)^2 + (y^2+z^2-x^2)(y-z)^2 + (z^2+x^2-y^2)(z-x)^2}
= (1/2){(x+y-z)^2(x-y)^2 + (y+z-x)^2(y-z)^2 + (z+x-y)^2(z-x)^2}

しかし、ここから (結果を知らずに) 次式に変形する方法が思いつかない。

F_1 = xy(x+y)(x-y)^2/{(y+z)(z+x)} + yz(y+z)(y-z)^2/{(z+x)(x+y)} + zx(z+x)(z-x)^2/{(x+y)(y+z)}
0700132人目の素数さん
垢版 |
2017/08/26(土) 15:31:34.41ID:a5WQhO5r
>>698
3F_2 = 2(x+y+z)F_1 +{(x+y+z)^2 -4(xy+yz+zx)}F_0,
を使うと
F_3 =(xx+yy+zz)F_1 - 2xyzF_0
となるが、その後が…

700げとー
0701132人目の素数さん
垢版 |
2017/08/26(土) 16:54:34.17ID:a5WQhO5r
>>700

P=p(z-y), Q=q(x-z), R=r(y-x), p+q+r=0 のとき
 P(x-y)(x-z)+ Q(y-z)(y-x)+ R(z-x)(z-y)
 =(p+q+r)
 = 0,
ここに=(x-y)(y-z)(z-x),

例 p=z-y,q=x-z,r=y-x のとき P=pp、Q=qq、R=rr.
0702132人目の素数さん
垢版 |
2017/08/27(日) 00:28:26.43ID:NetfQ0ow
>>677 (3) >>690 >>694

・t≧9 のときは明らか。

・3≦t≦9 のとき。

24tt -(9-t)(t^3 +9u)= t(t-3)^3 + 3(9-t)(t-3)≧ 0,

(9-t)/3t ≦ 8t/(t^3 +9u),

(左辺)-(右辺)=(2/s + 1/3)- 3/t
 = 2/s - (9-t)/3t
 ≧ 2/s -8t/(s^3 +9u)
 = 2(s^3 -4st +9u)/{s(s^3 +9u)}
 = 2F_1(x,y,z)/{s(s^3 +9u)}
 ≧ 0,
0703132人目の素数さん
垢版 |
2017/08/27(日) 00:47:52.80ID:NetfQ0ow
>>677 (3) >>690 >>694

・t≧9 のときは明らか。

・3≦t≦9 のとき。

24tt -(9-t)(t^3 +9uu)= t(t-3)^3 +3(9-t)(t-3)≧ 0,

(9-t)/3t ≦ 8t/(t^3 +9uu),

(左辺)-(右辺)=(2/s + 1/3)- 3/t
 = 2/(su) - (9-t)/3t
 ≧ 2/(su) -8t/(t^3 +9uu)
 = 2(t^3 -4stu +9uu)/{su(t^3 +9uu)}
 = 2uu・F_1(1/x,1/y,1/z)/{s(t^3 +9uu)}
 ≧ 0,
0704132人目の素数さん
垢版 |
2017/08/27(日) 01:08:20.58ID:NetfQ0ow
>>679 (1) >>690

・t≧5 のときは明らか。

・3≦t≦5 のとき、

24t -(5-t)(t^3 +9uu)=(t-3)^4 +7(t-3)^3 +9(t-3)^2 +6(t-3)≧0,

5-t ≦ 24t/(t^3 +9uu),

(左辺)-(右辺)= 6 -(5-t)s
 ≧ 6 -24st/(t^3 +9uu)
 = 6(t^3 -24stu +9uu)/(t^3 +9uu)
 = 6u^3・F_1(1/x,1/y,1/z)/(t^3 +9uu)
 ≧ 0,
0706132人目の素数さん
垢版 |
2017/08/27(日) 10:23:54.77ID:NetfQ0ow
>>703 >>704
 
t^3 -4stu +9uu = u^3・F_1(1/x,1/y,1/z)= uu・F_{-2}(x,y,z)

={(z^5)(xx-yy)^2 + (x^5)(yy-zz)^2 +(y^5)(zz-xx)^2}/{(x+y)(y+z)(z+x)}

≧0

を使いますた。
0707132人目の素数さん
垢版 |
2017/08/27(日) 16:11:26.92ID:NetfQ0ow
>>677

佐藤(訳):文献[9]、演習問題1.86

u=1 のときは(s,t)を入れ換えても成り立つ。(duality)
0708132人目の素数さん
垢版 |
2017/08/27(日) 16:26:59.89ID:NetfQ0ow
>>388 (5) >>450

〔Hlawkaの不等式〕を拡張…
r≧1 のとき
 K(r){|a|^r +|b|^r +|c|^r +|a+b+c|^r}≧|a+b|^r +|b+c|^r +|c+a|^r,

ここに K(r)は
 1≦r≦2 のとき、K(r)=(2^r)/{1+3^(r-1)},
 2≦r のとき、K(r)= 2^(r-2),

 kurims 講究録-1136-11 p.90-95 (2000) Theorem 2


>>449 (2)
 佐藤(訳):文献[9]、演習問題1.43、問題3.67
 (1+ab)(1+a)= ab(1+c)/(1+a), など。
 AM-GMする。


>>453
 佐藤(訳):文献[9]、演習問題1.61
 x^3 +x^3 +y^3 ≧ 3xxy, (AM-GM)より
 x^3 +y^3 +z^3 ≧ xxy + yyz + zzx,
 (x,y,z)=(a,b,c)と(x,y,z)=(ab,bc,ca)をたす。
0709132人目の素数さん
垢版 |
2017/08/27(日) 20:32:51.97ID:u/VQjdir
>>689
> 〔補題196〕の略証
> (左辺)-(右辺)= P(a-b)(a-c)+ Q(b-c)(b-a)+ R(c-a)(c-b)

この形に変形するのって、ものすごく大変なんじゃないん?
0710132人目の素数さん
垢版 |
2017/08/27(日) 23:16:42.54ID:NetfQ0ow
>>709
その通り。
(a,b,c)=(1,1,1)以外に(1,1,2)(1,2,1)(2,1,1)でも等号が成立するから、チョト難しい。
他に使えそうな方法は無いか?
0711132人目の素数さん
垢版 |
2017/08/28(月) 00:00:38.32ID:4VsD2YTN
>>708
 解答も訂正。

>>453
チェビシェフ(または AM-GM)で
 a^3 +b^3 +c^3 ≧ aab + bbc + cca = (a/c + b/a + c/b)u,
 (ab)^3 +(bc)^3 +(ca)^3 ≧ ab(bc)^2 + bc(ca)^2 + ca(ab)^2 = (b/a + c/b + a/c)uu,
辺々たす。
0712132人目の素数さん
垢版 |
2017/08/28(月) 01:54:30.17ID:4VsD2YTN
>>679 (5)

a/c=y, b/a=z, c/b=x とおくと xyz=1.
(a-b)(b-c)(c-a)/abc =(a/b +b/c +c/a)-(c/b +a/c +b/a)=(xy+yz+zx)-(x-y-z),

(左辺)-(右辺)=(xx+yy+zz)-2(xy+yz+zx)+2(x-y-z)-3
=(x+y+z)^2 -4(xy+yz+zx)+2s -3
={F_1(x,y,z) -9xyz}/s +2s -3
= F_1(x,y,z)+(2s+3)(s-3)/s
≧0,   (s=x+y+z≧3)
0713132人目の素数さん
垢版 |
2017/08/28(月) 03:43:27.12ID:Xt3/xWpv
(1) a, b, c>0 に対して、(a+b+c)^5 ≧ 81abc(a^2+b^2+c^2)
(2) a, b, c>0 に対して、(a+b+c)^6 ≧ 27(a^2+b^2+c^2)(ab+bc+ca)^2

AOPS:https://artofproblemsolving.com/community/c6h1282022p6753168

[疑問1]
(1)の証明について、

(a+b+c)^3 - 3(a+b)(b+c)(c+a) = s^3 - 3(st-u) = s(s^2-3t) + 3u >0
∴ (a+b+c)^3 > 3(a+b)(b+c)(c+a) ---(A)

>>687 〔補題196〕 の右側
(a+b)(b+c)(c+a)(a+b+c)^2 ≧ 24abc(a^2+b^2+c^2) ---(B)

(A),(B)から、
(a+b+c)^3 *(a+b+c)^2 > 3(a+b)(b+c)(c+a)*(a+b+c)^2 ≧ 3*24abc(a^2+b^2+c^2)

等号が成り立たなくなるが、実際は例えば、a=b=c のときに等号が成り立つ。
このやり方は、何か間違っているのかな?

A≧B を証明するときに、途中に式を挟んで A≧C、C≧B を証明することがあるけど、
A=C かつ C=B から出した等号成立条件が、A=Bの等号成立条件と一致しないことがあるのは仕方のないことなのかな?
(具体例がすぐには出てこないけど、絶対値の入った不等式の証明とかで、なったことがある)

[疑問2]
(2)の証明が分かりませぬ…。
(1)を次のように証明して、そのコメントに、「コーラを飲んだらゲップが出るくらい明らか(嘘訳)」
と書いてあるけど、ピンときませぬ…。

(a+b+c)^6 ≧ 27(a^2+b^2+c^2)(ab+bc+ca)^2 ≧ 81abc(a^2+b^2+c^2)
0715132人目の素数さん
垢版 |
2017/08/28(月) 06:30:48.05ID:Xt3/xWpv
>>688-689
> (a+b)(b+c)(c+a)(a+b+c)^2 ≧ 24abc(aa+bb+cc)
>
> bはa、cの間にあるとする。
> (左辺)-(右辺)
> = P(a-b)(a-c)+ Q(b-c)(b-a)+ R(c-a)(c-b)
> = P(a-b)^2 +(P-Q+R)(a-b)(b-c)+ R(b-c)^2,
>
> P =(b+c)(b+c-a)^2 + 2(a+b+c)(b-c)^2 ≧ 0,
> P-Q+R = 2b{(a+c)^2 -6ac+3bb}= 2b{(a+c-3m)^2+3(bb-mm)}≧ 0,
> R =(a+b)(a+b-c)^2 + 2(a+b+c)(a-b)^2 ≧0,
> ここに、m = min{a,c}、ac=m(a+c-m)


Q = (c+a)(c+a-b)^2 + 2(a+b+c)(c-a)^2 として、P-Q+R を計算したら、

P-Q+R = 2b{(a+c)^2 -6ac+3bb} + 8(c+a){(c-a)^2 + ca}

となったけど、計算合っているか確認おねがいしますだ。
0716132人目の素数さん
垢版 |
2017/08/28(月) 06:52:45.91ID:Xt3/xWpv
>>715
ごめん。私の計算違いでした。

       ヘ))∧
      (゚ ∀゚ )
     ノ || y / ヽ 切腹しまつ
  ━(m二フ⊂[__ノ、
     (_(__ノ
0717132人目の素数さん
垢版 |
2017/08/28(月) 11:53:15.27ID:4VsD2YTN
>>712 の訂正
× (x-y-z)
○ (x+y+z)


>>713

[疑問1]
 (1)は >>679 (2)ですね。
 >>687 を参照。
 あえて難しい〔補題196〕を使う必要は無かったですね。

[疑問2]
 >>687 を参照。
 (2)と(ab+bc+ca)^2 ≧ 3abc(a+b+c) から(1)を出します。

>>714
 そうです。
0718132人目の素数さん
垢版 |
2017/08/28(月) 21:24:09.98ID:fpou6rxt
>>713
(1)
A >= 81B という不等式を示すのに A > 72B という不等式を示しても何も意味がない
より雑な不等式にしてるんだから等号が成立しなくなるのは必然

[疑問1]
A >=C, C >=B の両方の等号成立条件を合わせたものが A >= B の等号成立条件

(2)
因数分解が一番簡単

[疑問2]
uvw で右側の不等式は明らか
(おそらく AoPS での解き方はこれ)
0722132人目の素数さん
垢版 |
2017/08/29(火) 03:10:07.75ID:QmBHjFut
>>69 (1)、>>713 (1)
> a, b, c>0 に対して、(a+b+c)^5 ≧ 27(ab+bc+ca)(ab^2 + bc^2 + ca^2)
> a, b, c>0 に対して、(a+b+c)^5 ≧ 81abc(a^2 + b^2 + c^2)

改造手術の時間でござるよ。 右辺の大小は定まるのでせうか?

27(ab+bc+ca)(ab^2 + bc^2 +ca^2) = 27abc * (ab+bc+ca)(a/b + b/c + c/a)
         81abc(a^2+b^2+c^2) = 27abc * 3(a^2 + b^2 + c^2)

だから、(ab+bc+ca)(a/b + b/c + c/a) と 3(a^2 + b^2 + c^2) の大小が定まれば…。

(ab+bc+ca)(a/b + b/c + c/a) ≧ (a+b+c)^2 ≧ 3(ab+bc+ca) ≦ 3(a^2 + b^2 + c^2)

適当にやっても、うまく行かんでござる…

 ..::::::,、_,、::: ::::: ::: : 
  /ヨミ゙ヽ)-、. :: ::::
─(ノ─ヽ.ソ┴─
0723132人目の素数さん
垢版 |
2017/08/29(火) 03:22:58.97ID:QmBHjFut
a, b, c >0 の基本対称式 s, t, u で、曲者を縛るでござる。 (曲者 = a/b + b/c + c/a)

(ab+bc+ca)(a/b + b/c + c/a) ≧ (a+b+c)^2
a(a-b)^2 + b(b-c)^2 + c(c-a)^2 = s^3 - 2st - 3u(a/b + b/c + c/a) ≧ 0

∴ s(s^2-2t)/(3u) ≧ a/b + b/c + c/a ≧ s^2/t

これしか思いつきませぬ…。 他にないでござるかな?
0724132人目の素数さん
垢版 |
2017/08/29(火) 03:49:39.64ID:1JAWO9sa
>>721

A + 4H =(A/2) +(A/2)+ 4H
≧ 3(AAH)^(1/3)  (← AM-GM)
= 3{(s/3)(s/3)(3u/t)}^(1/3)
≧ 3u^(1/3)   (← ss≧3t)
= 3G      (← Sierpinski)
を使うのが簡単かと...

A + 3H > (2/3)(A+4H)≧ 2G >{5/16^(1/3)} G
0725132人目の素数さん
垢版 |
2017/08/29(火) 04:41:59.71ID:QmBHjFut
>>721>>724
出典を再発見。 (大量のブックマークの中から探すのに苦労したでござる)
https://math.stackexchange.com/questions/1806146/prove-fracxyz3-frac3-frac1x-frac1y-frac1z-geq5-sqrt3-fracxyz?noredirect=1&;lq=1

斜め読みしたけど、何をやってるのかサッパリでござる ('A`)


>>724
分かりやすい!
でも、この方法では等号がつかないですね。
0726132人目の素数さん
垢版 |
2017/08/29(火) 05:25:02.43ID:QmBHjFut
>>721>>724
ごめん、リンク先の問題をよく見たら、問題が間違っていました。

正しくは、 「a, b, c >0 に対して、AM + HM ≧ 5*GM/{16^(1/3)}」 でした。
0727132人目の素数さん
垢版 |
2017/08/29(火) 05:44:11.14ID:QmBHjFut
>>721 再掲
a, b, c >0 に対して、AM + HM ≧ 5*GM/{16^(1/3)}

>>724 の方法を真似てみたが、うまくいかなかった。

A + H
=(A/2) +(A/2)+ H
≧ 3(AAH/4)^(1/3)    …(1)
= 3{(ss/(3t))*(u/4)}^(1/3)
≧ 3{(u/4)}^(1/3)    …(2)
= 3G/{4^(1/3)}

(1)の等号は A=2H、(2)の等号は a=b=c で異なるから、
A+H > 3G/{4^(1/3)}

問題の右辺と較べたら、5/16^(1/3)} > 3/{4^(1/3)} でした。
0728132人目の素数さん
垢版 |
2017/08/29(火) 09:12:22.39ID:QmBHjFut
【問題】
xyz座標平面において、次の不等式で表される立体の体積を求めよ。
 |x+y+z| + |-x+y+z| + |x-y+z| + |x+y-z| ≦ 4

検索中に、どこかで見たことのある問題を見つけた。
しばらく検索したものの、出典は分からず…。
コレクションに入っているかと探したが、そこにもなかった。

これが、どんな立体図形になるのかも分かりませぬ ('A`)ヴォエァ!
0729132人目の素数さん
垢版 |
2017/08/29(火) 09:27:12.29ID:QmBHjFut
>>679 (1) について

問題再掲
a, b, c >0、abc=1 に対して、(a+b)(b+c)(c+a) + 7 ≧ 5(a+b+c).

解答
>>704>>706

うますぎて、思いつきませぬ。
以下のような泥臭い方法で考えていたんだけど、行き詰まったでござる。

左辺 - 右辺 の最小値を考える。
abc=1 があるので、実質2文字の関数で、一方を任意に固定して、一変数関数で考えて出せないかと。
0731132人目の素数さん
垢版 |
2017/08/29(火) 11:45:47.48ID:1JAWO9sa
>>728
|a+b|+|a-b|= 2 Max{|a|,|b|}を使うと、
(左辺)= Max{4|x|,4|y|,4|z|,2|x+y+z|,2|-x+y+z|,2|x-y+z|,2|x+y-z|}

|x|≦1
|y|≦1
|z|≦1
|x+y+z|≦2
|-x+y+z|≦2
|x-y+z|≦2
|x+y-z|≦2
の14面で囲まれた立方八面体でござる。


>>729
t^3 -4stu +9uu ≧ 0,  >>706
s = a+b+c ≦ (t^3 +9uu)/4tu
u = abc = 1
を使って sとu を消し、t=ab+bc+ca だけの関数で考えて出したのが >>704
0733132人目の素数さん
垢版 |
2017/08/29(火) 17:18:34.40ID:QmBHjFut
>>731
> t^3 -4stu +9uu ≧ 0,  >>706
> s = a+b+c ≦ (t^3 +9uu)/4tu
> u = abc = 1
> を使って sとu を消し、t=ab+bc+ca だけの関数で考えて出したのが >>704

なるほど。 u=1 だから、s か t のどちらかを消せばよいと。
そこで s を消すために、sを含む s, t, u の不等式の中から、s≦f(t) となりそうなものとして F_1 を選んだ訳でござるな。
考え方が分かってスッキリ!

するってぇと何かい? t^2 ≧ 3su を使ってもいいってことだね?

s ≦ (t^2)/(3u) = (t^2)/3 より、3≦t≦5 のとき、

(左辺)-(右辺)
= 6 - (5-t)s
≧ 6 - (5-t)*(t^2)/3
= (t-3)(t^2-2t-6)/3

-3 ≦ t^2-2t-6 ≦ 5 となって失敗したでござる。 F_1 じゃなきゃダメなのか…。
0735132人目の素数さん
垢版 |
2017/08/30(水) 01:43:40.46ID:BK+APDDw
>>733
F_1 じゃなきゃダメですね…。

マクラーレン・ホンダ:F_1ベルギーGPの決勝レポート(8/28)

マクラーレンはF_1ベルギーGP決勝で、S.バンドーンが14位、F.アロンソはリタイアだった。

両ドライバーは見事なスタートを切り、F.アロンソは1周目には10番手から7番手に浮上。

しかし、その後エンジンの不調が発生したためリタイアし、入賞を逃しますた。残念
0736132人目の素数さん
垢版 |
2017/08/30(水) 02:37:18.32ID:4Q4sm7+y
怒涛の abc=1 シリーズの際に書いたつもりが、書いてなかったようなので。

【問題】
a, b, c >0、abc=1 に対して、
1/(1+a)^3 + 1/(1+b)^3 + 1/(1+c)^3 + 5/{(1+a)(1+b)(1+c)} ≧ 1


 ∧_∧        積一定?
 ( ・ω・)=つ≡つ  ボコボコにしてやんよ!
 (っ ≡つ=つ
 /   ) ババババ
 ( / ̄∪
0739132人目の素数さん
垢版 |
2017/08/30(水) 08:34:33.56ID:4Q4sm7+y
>>732
AM-GM や Schur で証明できた場合は、等号成立条件が a=b=c になってしまうから、
証明の中で、それ以外の特殊な不等式が必要になるってことですかね?
0740132人目の素数さん
垢版 |
2017/08/30(水) 11:56:04.84ID:BK+APDDw
>>737

(a,b,c) →(1/a,1/b,1/c)としたでござるな。

a+b+c → (ab+bc+ca)/abc,
ab+bc+ca → (a+b+c)/abc,
abc → 1/abc,

>>703 の(s,t)を入れ換えて
 F_1(a,b,c)= s^3 -4st +9u ≧0,
 t ≦(s^3 +9u)/4s,
これを使えば おk >>707

>>739
そうですね。
AM-GM や Schurは(1,4,4)で等しくないので使えません。
0741132人目の素数さん
垢版 |
2017/08/30(水) 17:00:49.35ID:4Q4sm7+y
>>736
難しいので、劣化改造してみた。こちらは力任せに証明できる。

a, b>0 かつ ab=1 のとき、1/(1+a)^2 + 1/(1+b)^2 + 2/{(1+a)(1+b)} ≧1.
0742132人目の素数さん
垢版 |
2017/08/30(水) 17:18:01.56ID:4Q4sm7+y
ところで、AM + GM に関する不等式って何かあったっけ? Jacobsthal は差だし、Sierpinskiは商か。
0744132人目の素数さん
垢版 |
2017/08/31(木) 00:00:50.60ID:iQe17wVf
>>679
(4)をプチ改造。Nesbittの間に割り込んだ形ですね。

a, b, c >0、abc=1 に対して、
a/(b+c) + b/(c+a) + c/(a+b) ≧ 1/(b+c) + 1/(c+a) + 1/(a+b) ≧ 3/2
0745132人目の素数さん
垢版 |
2017/08/31(木) 00:14:37.43ID:iQe17wVf
>>744
左は(4)を変形しただけ。

右は間違っているかもしれん。
Cauchyの後にAM-GMを使ったんだけど、AM-GMの不等号が逆で、証明になっていなかった。
0746132人目の素数さん
垢版 |
2017/08/31(木) 00:17:09.96ID:iQe17wVf
結局、こうですね。

a, b, c >0、abc=1 に対して、
a/(b+c) + b/(c+a) + c/(a+b) ≧ 1/(b+c) + 1/(c+a) + 1/(a+b) > 0
0747132人目の素数さん
垢版 |
2017/08/31(木) 02:42:09.91ID:iQe17wVf
これでOK?

λを正定数、a, b>0 かつ ab=1 のとき、
1 + λ/4 ≧ 1/(1+a)^2 + 1/(1+b)^2 + (2+λ)/{(1+a)(1+b)} ≧1.
0748132人目の素数さん
垢版 |
2017/08/31(木) 02:45:27.34ID:iQe17wVf
λを正定数、a, b>0 かつ ab=1 のとき、
1 + λ/4 ≧ 1/(1+a)^2 + 1/(1+b)^2 + (2+λ)/{(1+a)(1+b)} > 1.

こうですね。
0750132人目の素数さん
垢版 |
2017/08/31(木) 07:12:05.62ID:iQe17wVf
a, b, c ≧0 かつ a+b+c=1 のとき、a*(a+b)^2*(b+c)^3*(c+a)^4 の最大値を求めよ。
0751132人目の素数さん
垢版 |
2017/08/31(木) 10:46:05.27ID:DG2IOYgq
>>750
GM-AM で
(与式)= 16・a・(a+b)^2・(b+c)^3・{(c+a)/2}^4
 ≦ 16{[a + 2(a+b)+ 3(b+c)+ 4((c+a)/2)]/(1+2+3+4)}^10
 = 16{(a+b+c)/2}^10
 = 1/64.  (← a+b+c=1)
等号は(a,b,c)=(1/2,0,1/2)
0753132人目の素数さん
垢版 |
2017/08/31(木) 22:18:05.59ID:A7wnlx0o
>>752
間違えた
a, b, c >0, abc=1
a/(b+c) + b/(c+a) + c/(a+b) >= 1/(b+c) + 1/(c+a) + 1/(a+b) + (1/2 - 4/((a+b)(b+c)(c+a)))
0754132人目の素数さん
垢版 |
2017/09/01(金) 00:01:46.44ID:3P2EPmWz
【問題A】a, b, c >0 とする。

(1)
(ab+bc+ca)^3 ≧ (a^2 + 2b^2)(b^2 + 2c^2)(c^2 + 2a^2)

(2)
(a^2 + b^2 + c^2)^3 ≧ (a+b+c)(ab+bc+ca)(a^3 + b^3 + c^3)

(3)
(a^2 + bc)(b^2 + ca)(c^2 + ab) ≧ abc(a+b)(b+c)(c+a)

(4)
3*{(ab)^2 + (bc)^2 + (ca)^2} ≧ (ab+bc+ca)(a^2 + b^2 + c^2)

(5)
(a^2 + ab + b^2)(b^2 + bc + c^2)(c^2 + ca + a^2) ≧ (ab+bc+ca)^3

(6)
(a^2 + b^2 + c^2)/(ab+bc+ca) + 8abc/(a+b)(b+c)(c+a) ≧ 2


【問題B】

(7)
a, b, c, d >0 に対して、(a+b+c-d)(b+c+d-a)(c+d+a-b)(d+a+b-c) ≦ (a+b)(b+c)(c+d)(d+a)

(8)
0 ≦ a, b, c ≦ 1 に対して、a^(bc) + b^(ca) + c^(ab) > 2


【参考】
(8)の類題 [第5章.698, 708]
a, b, c >0 に対して、a^(b+c) + b^(c+a) + c^(a+b) ≧ 1


     ___   ====
\  ./ ≧  \   ====
  \| \ ./  ::::| 
   | ●) ●) :::::|  そんな不等式で俺様がクマ――!!
   ヽ......ワ...:::::.ノ
     `つ   `つ      (´⌒(´
      ゝ_つ_`つ≡≡≡(´⌒;;;≡≡≡
               (´⌒(´⌒;;
      ズザザザ
0755132人目の素数さん
垢版 |
2017/09/01(金) 00:16:20.58ID:3P2EPmWz
【問題】
a, b, c >0 に対して、2*QM + 3*GM ≦ 5*AM。 ただし、QM = √{(a^2+b^2+c^2)/3}
0756132人目の素数さん
垢版 |
2017/09/01(金) 06:54:43.37ID:3P2EPmWz
>>388
条件 x>y が抜けとる。すみませぬ。

訂正
x>y>0 かつ (x^6)(y^2) - (x^5)(y^3) + (x^5)(y^5) - (x^4)(y^6) ≧ 4 のとき、x^3+y^2≧3.
0757132人目の素数さん
垢版 |
2017/09/01(金) 11:18:02.33ID:QpLZW4eS
>>754
(1)
aa=A,bb=B,cc=C とおいて考える。

(右辺)=(A+2B)(B+2C)(C+2A)
= 2(AAB+BBC+CCA)+ 4(ABB+BCC+CAA)+ 9ABC,

(左辺)=(ab+bc+ca)^3
= aabb(ab+3bc+3ca)+ bbcc(bc+3ca+3ab)+ ccaa(ca+3ab+3bc)+6(abc)^2
≦ AB(2A+2B+3C)+ BC(2B+2C+3A)+ CA(2C+2A+3B)+ 6ABC
= 2(AAB+BBC+CCA)+ 2(ABB+BCC+CAA)+15ABC,

(右辺)-(左辺)≧ 2(ABB+BCC+CAA-3ABC)≧ 0,  (← AM-GM)

(4) a>>b,c では不成立?

(5)コーシーで
(ab+bb+aa)(bb+bc+cc)(aa+cc+ca)≧(ab+bc+ca)^3

(6)
9(st-u) - 8st = 9(a+b)(b+c)(c+a)- 8(a+b+c)(ab+bc+ca)
= a(b-c)^2 + b(c-a)^2 + c(a-b)^2
≧0,
(左辺)-2 = (ss-4t)/t + 8u/(st-u)
≧ 8s(ss-4t)/{9(st-u)} + 8u/(st-u)
= 8(s^3 -4st+9u)/{9(st-u)}
= 8F_1(a,b,c)/{9(st-u)}
≧0,
0768132人目の素数さん
垢版 |
2017/09/01(金) 14:40:29.27ID:QpLZW4eS
>>754

(2)
(左辺)-(右辺)=(aa+bb+cc)^3 -(a+b+c)(ab+bc+ca)(a^3+b^3+c^3)
= p'(b-c)^2 + q'(c-a)^2 + r'(a-b)^2
≧ 0,
ここに
p ' ={4a^4+b^4+c^4 +(a^4+a^4+b^4+c^4-4aabc)}/4 ≧(4a^4+b^4+c^4)/4,
q ' ={a^4+4b^4+c^4 +(a^4+b^4+b^4+c^4-4abbc)}/4 ≧(a^4+4b^4+c^4)/4,
r ' ={a^4+b^4+4c^4 +(a^4+b^4+c^4+c^4-4abcc)}/4 ≧(a^4+b^4+4c^4)/4,

(3)
(左辺)-(右辺)=(aa+bc)(bb+ca)(cc+ab)- abc(a+b)(b+c)(c+a)
= abc{a(a-b)(a-c)+b(b-c)(b-a)+c(c-a)(c-b)}+{(ab)^3 +(bc)^3 +(ca)^3 -3(abc)^2}
= u(s^3 -4st+9u)+ t(tt-3su)
= u・F_1(a,b,c)+ t・uF_{-1}(a,b,c)
≧ 0,
0769132人目の素数さん
垢版 |
2017/09/01(金) 15:02:15.49ID:QpLZW4eS
>>754
(7)
左辺の4つの因子のうち、負になれるのは高々1つだけ。
左辺が正のときは4つとも正。
GM-AMで
(a+b+c-d)(b+c+d-a)=(b+c)^2 -(a-d)^2 ≦(b+c)^2,
循環的に掛ける。
0782132人目の素数さん
垢版 |
2017/09/01(金) 22:46:45.99ID:QpLZW4eS
>>726 >>727
>>732 >>739
AM-GMやSchurは使えそうにないので...

a ≦ b,c とすると、G =(abc)^(1/3)≧ a,
m = √(bc)とおき、
(a,b,c)→(a,m,m)としたとき、Gは不変で、
A(a,b,c)- A(a,m,m)=(b+c-2m)/3,
H(a,b,c)- H(a,m,m)=(b+c-2m)/3{-H(a,b,c)H(a,m,m)/bc}
 ≧(b+c-2m)/3(-GG/bc)
 =(b+c-2m)/3(-a/G)
∴ A(a,b,c)+ H(a,b,c)≧ A(a,m,m)+ H(a,m,m)
等号成立は b=c のとき。 ……(1)
大きい方の2つが等しい場合を考えればよいので、
ほぼ1変数の問題に帰着する。
A(a,m,m)+ H(a,m,m)
= 2(aa+7am+mm)/{3(2a+m)}
={5/16^(1/3)}G + f(x)・mm/{24(2a+m)}
≧{5/16^(1/3)}G,
ここに、x =(4a/m)^(1/3)とおいた。
f(x)= x^6 - 15x^4 +28x^3 -30x +16
=(x-1)^2{(xx-4)^2 + 2x(x-1)^2},
等号成立は x=1,4a=m=√(bc)のとき。 ……(2)

(1)(2)より、(a,b,c)=λ(1,4,4)
0783132人目の素数さん
垢版 |
2017/09/01(金) 22:57:26.08ID:3P2EPmWz
>>757
昔のmemoの中に、>>754(5)を改造したものがあった。

a, b, c >0 に対して、
(a^2 + ab + b^2)(b^2 + bc + c^2)(c^2 + ca + a^2)
≧ (27/64)*[(a+b)(b+c)(c+a)]^2
≧ (1/3)*[(a+b+c)(ab+bc+ca)]^2
≧ (ab+bc+ca)^3.
0794132人目の素数さん
垢版 |
2017/09/01(金) 23:50:29.90ID:QpLZW4eS
>>726-727

〔類題〕
AM + 0.90096 HM ≧ 1.90096 GM

等号成立は(a,b,c)=λ( 0.3962570…,1,1)のとき

[第7章.897-903]
0795132人目の素数さん
垢版 |
2017/09/02(土) 01:00:08.66ID:Po7d73tU
>>388 (5) >>450 >>708

〔Hlawkaの不等式〕の拡張
m≧2 のとき、
(m-2)Σ[k=1,m]|x_k |^2 +|Σ[k=1,m] x_k |^2 = Σ[1≦i<j≦m]|x_i +x_j |^2.
(m-2)Σ[k=1,m]|x_k|+|Σ[k=1,m] x_k|≧ Σ[1≦i<j≦m]|x_i +x_j|.
(D.D.Adamovic)
[初代スレ.354-360,364]
文献[3] 大関、p.34
0808132人目の素数さん
垢版 |
2017/09/02(土) 02:38:52.19ID:Po7d73tU
>>755

QQ =(ss-2t)/3 ≦{ss - 2√(3su)}/3 = 3AA - 2G√(AG),

(5A-3G)^2 -(2Q)≧(5A-3G)^2 -12AA +8G√(AG)
= 13AA -30AG +8G√(AG) +9GG
=(√A -√G)^2{13A +26√(AG)+9G}
≧ 0,
∴ 5A-3G ≧ 2Q,
0819132人目の素数さん
垢版 |
2017/09/02(土) 04:25:20.11ID:ziPENgdW
>>757 (6)
(左辺)-(右辺) の計算過程で、
  9(a+b)(b+c)(c+a) ≧ 8st …(1)
の使うタイミングが上手いですね。

私は 左辺の第1項に対して使ってしまい、その後の変形で分子が
  8F_1 - 2E_1 ここで E_1 = st-9u
となって、ずっと悩んでいました。

(左辺の第1項-2)に対して使うことで、あっさり片付くとは! いと難し… ('A`)ヴォエァ!
0832132人目の素数さん
垢版 |
2017/09/02(土) 05:50:25.95ID:ziPENgdW
ここまでの荒らし数470くらい。 50%を超えているとは思わなんだ。
¥って何なんだ? 山崎パンかよ!
0833132人目の素数さん
垢版 |
2017/09/02(土) 07:24:51.83ID:ziPENgdW
>>757 (1)
左辺の変形は、同順序積の方が大きいことを利用して、瞬時に大きくしたのですかね?
0834132人目の素数さん
垢版 |
2017/09/02(土) 07:36:00.01ID:ziPENgdW
と思ったが、係数まで変わっているから、やっぱり分からないなあ。
0835132人目の素数さん
垢版 |
2017/09/02(土) 10:45:02.21ID:Po7d73tU
>>833 >>834
GM-AM で
 ab ≦(A+B)/2,bc ≦(B+C)/2,ca ≦(C+A)/2,
を使ったでござる。


>>820 >>821
 Q = RMS は Root Mean Square(二乗平均平方根)です。


>>808 の修正
(5A-3G)^2 -(2Q)^2 ≧(5A-3G)^2 -12AA -2G√(AG)
= 13AA -30AG +8G√(AG)+9GG
= 9(A-G)^2 + 4{AA + G√(AG)+ G√(AG)-3AG}
≧ 0,
0837132人目の素数さん
垢版 |
2017/09/02(土) 11:10:47.11ID:ziPENgdW
>>835
> >>808 の修正
> (5A-3G)^2 -(2Q)^2 ≧(5A-3G)^2 -12AA -2G√(AG)
> = 13AA -30AG +8G√(AG)+9GG
> = 9(A-G)^2 + 4{AA + G√(AG)+ G√(AG)-3AG}
> ≧ 0,

修正前の方が分かりやすいような希ガス…。
0838132人目の素数さん
垢版 |
2017/09/02(土) 12:54:10.72ID:Po7d73tU
>>833 >>834
たしかに
(ab+bc+ca)^3 ≦(8/7)(AAB+BBC+CCA)+(8/7)(ABB+BCC+CAA)+(141/7)ABC,
等号は(a,b,c)=(1,1,1)と(3/4,1,1)
が最良でしょうが、出すのが面倒でござる。

ここでは、簡単に出せる >>757 を使ったでござる。(これで十分だし)

>>837
すまぬ。あちらを正せばこちらが…でござった。
0839132人目の素数さん
垢版 |
2017/09/02(土) 14:08:10.03ID:ziPENgdW
>>768
>>754 (2) を F_0 を残したまま展開してみたなり。

(左辺)-(右辺)
= (F_0 + t)^3 - st*(sF_0 + 3u)
= (F_0)^3 + 2t*(F_0)^2 + t*(uF_{-1})
≧ 0
0840132人目の素数さん
垢版 |
2017/09/02(土) 14:29:19.80ID:Po7d73tU
>>819

>>757(6)は(左辺第1項 -2)< 0 の場合は?でしたね。
通分してSchurの拡張を使います。

(左辺)- 2 =(ss-4t)/t + 8u/(st-u)
={(ss-4t)(st-u)+8ut}/{t(st-u)}
={P(a-b)(a-c)+ Q(b-c)(b-a)+ R(c-a)(c-b)}/{t(st-u)},
ここで
P = aa(b+c)= at-u >0,
Q = bb(c+a)= bt-u >0,
R = cc(a+b)= ct-u >0,
(P,Q,R)は(x,y,z)と同順なので成立。
0843132人目の素数さん
垢版 |
2017/09/02(土) 20:18:37.53ID:VhdcIBK0
>>754
(1)
Holder の不等式
(b^2+b^2+a^2)(b^2+c^2+c^2)(a^2+c^2+a^2)(a^2+b^2+c^2) >= (ab+bc+ca)^4
から明らか

(2)
LHS >= sqrt(3(a^2+ab+b^2)(b^2+bc+c^2)(c^2+ca+a^2)) >= RHS

(3)
和積版並べ替え不等式から明らか
(a+x)(b+y)(c+z) >= (a+x’)(b+y’)(c+z’) >= (a+z)(b+y)(c+x)
for any positive a >= b >= c and x <= y <= z, {x’, y’, z’} = {x, y, z}

(5)
LHS >= 27/64 ((a+b)(b+c)(c+a)^2 >= RHS
0848132人目の素数さん
垢版 |
2017/09/03(日) 00:38:23.64ID:ueZS3BC0
【問題】
a, b, c >0 に対して、(a+b+c)^2 (a^2 + b^2 + c^2)^3 ≧ 27*{(ab)^2 + (bc)^2 + (ca)^2}^2


     ///////
    ///////____________
    ///////  ̄ ̄ ̄ ̄ ̄ ̄ ̄ ̄ ̄| ̄ ̄
   ///////      ___    (~) チリンチリン
   ///////     /  ≧ \  ノ,,
  ///////     |::::: (● (● |    
  ///////      ヽ::::... .ワ.....ノ    日本の夏
 ///////      (つ へへ つ      不等式の夏
0849132人目の素数さん
垢版 |
2017/09/03(日) 02:35:19.95ID:T+8hKHMc
>>845
>>846
>>847
(1) 834は間違え
Holderから LHS >= (a^2+ab+bc)*(b^2+bc+ca)*(c^2+ca+ab)
(a^2+ab+bc)*(b^2+bc+ca)*(c^2+ca+ab) - RHS
= abc(a^3+b^3+c^3-3abc) + (x^6+y^6+z^6-xyz(x^3+y^3+z^3))
>= 0
where x=(a^2b)^(1/3), …

(2)
正しくは
LHS >= sqrt(3(a^2+ab+b^2)(b^2+bc+c^2)(c^2+ca+a^2))(a^3+b^3+c^3) >= RHS
だった
右側はIndia2007(柳田先生の初等的な不等式I, 問題202)
左側は解析的にゴリゴリやればなんとか(上手い解法ありそうだけど)
いずれにしてもこの不等式を用いて解くというよりこれも成り立つというだけです
0850132人目の素数さん
垢版 |
2017/09/03(日) 12:20:38.50ID:UCZgMxaf
>>849
(1)
コーシーで
(aa+bb+bb)(aa+aa+cc)≧(aa+ab+bc)^2
これを巡回的に掛けたでござるな。

(2)
右側は
√(aa+ab+bb)≧((√3)/2)(a+b),
(a+b)(b+c)(c+a)≧(8/9)(a+b+c)(ab+bc+ca),
で簡単ですが左側は
b=c=1 のとき
LHS - MHS =(aa+2)^3 - 3(aa+a+1)(a^3 +2)
=(a-1)^3・(a^3 -2),
1<a<2^(1/3)でゴリ霧中…
0853132人目の素数さん
垢版 |
2017/09/04(月) 01:47:36.61ID:nXYDOT8Z
>>852
千手観音(千手千眼観自在菩薩)は、千本の手がありその手の掌には目が付いています。
へっへっへ
0854132人目の素数さん
垢版 |
2017/09/04(月) 13:23:44.99ID:nXYDOT8Z
>>754
(8)
f(x)=(1/a)^x は下に凸だから、0<x<1 で
f(x)- f(0)≦{f(1)- f(0)}x,
(1/a)^x - 1 ≦{(1/a)- 1}x,
∴ a^x ≧ a/(a+x-ax)= 1 - (1-a)x/(a+x-ax) …… ベルヌーイの式

x=bc を入れると、
a+x-ax = a+bc-abc = t-2u +a(1-b)(1-c)≧ t-2u,
∴ a^bc ≧ 1 -(bc-u)/(t-2u),
巡回的にたすと
(左辺)≧ 2 + u/(t-2u),
等号は u=abc=0 のとき。


【参考】
(8)の類題
a,b,c の中に1以上のものがあるときは明らか。
∴ 0< a,b,c <1 としてよい。
b+c,c+a,a+b の中に1より大きいものが無ければベルヌーイで一発なんだが…
0857132人目の素数さん
垢版 |
2017/09/04(月) 17:49:07.50ID:VCnnUpGA
>>848 反例 a=b=c=3^(-1/2)

クソが作ったクソ問を避けるために出典欲しくなるのもわかる
0871132人目の素数さん
垢版 |
2017/09/04(月) 22:31:06.87ID:r46JbgIy
>>870
左側はさらに厳密な
LHS >= 9((ab^3+bc^3+ca^3)^2 + (a^3b+b^3c+c^3a)^2 + ((ab)^2+(bc)^2+(ca)^2))^2
を示した方が簡単なおもしろい不等式
0882132人目の素数さん
垢版 |
2017/09/05(火) 02:53:45.95ID:3z9XJ0W/
>>870 >>871

LHS =(a+b+c)^2(aa+bb+cc)^3 ≧ 27(ab^3+bc^3+ca^3)(a^3b+b^3c+c^3a)
は無理ですね。


〔参考〕
(aa+bb+cc)^2 ≧ 3(ab^3+bc^3+ca^3)または 3(a^3b+b^3c+c^3a)

[第5章.268, 284-290]
[第2章.389]
文献[8]、安藤、§2.3.2 p.61 中段、g_{p,q}(a,b,c)≧0,
0883132人目の素数さん
垢版 |
2017/09/05(火) 03:34:06.76ID:3z9XJ0W/
>>754 (2)
>>768

s = a+b+c,
t = ab+bc+ca,
S2 = aa+bb+cc,
S3 = a^3 +b^3 +c^3,
とおく。

S2 - t ={(a-b)^2 +(b-c)^2 +(c-a)^2}/2 = F_0,
とおく。コーシーより
s・S3 - S2・S2 = ab(a-b)^2 + bc(b-c)^2 + ca(c-a)^2 ≦ F_0・S2,
∵ ab ≦(aa+bb)/2 ≦ S2 /2,etc.

LHS - RHS =(S2)^3 - st・S3
=(S2-t)S2・S2 - t(s・S3-S2・S2)
≧ F_0・S2・S2 - t・F_0・S2
=(F_0)^2・S2
≧ 0,
0894132人目の素数さん
垢版 |
2017/09/05(火) 05:07:23.93ID:q778+o9X
>>883
> コーシーより
> ab(a-b)^2 + bc(b-c)^2 + ca(c-a)^2 ≦ F_0・S2,

caushyをどう使ったんでせうか?
たしかに差をとれば (F_0)^2 + uF_{-1} ≧0 となりますが、caushyでパッと出す方法を知りたいです。
0896132人目の素数さん
垢版 |
2017/09/05(火) 11:21:44.67ID:3z9XJ0W/
>>894

〔補題〕(>>754 (2) のための)
a,b,c >0 とすると
(aa+bb+cc){2(aa+bb+cc)-(ab+bc+ca)}≧(a+b+c)(a^3+b^3+c^3)≧(aa+bb+cc)^2,

(略証)
左側は
S2(S2+F_0)- s・S3 ={(a-b)^2+cc}/2 (a-b)^2 + cyclic. ≧ 0,
右側がコーシーでしたね。
s・S3 -(S2)^2 = ab(a-b)^2 + bc(b-c)^2 + ca(c-a)^2 ≧ 0,(終)

あとは >>883 のとおり。
0897132人目の素数さん
垢版 |
2017/09/06(水) 06:00:26.89ID:AYr/rfmQ
>>848 >>870 >>871

(aa+bb+cc)^(3/2)={(ss + 2F_0)/3}^(3/2)
≧ √(ss/3)(ss/3 + F_0)   (← AM-GM)
= (4sss -9st)/(3√3)
≧(7st -36u)/(3√3)  (← F_1=sss-4st+9u≧0)
≧(3√3)(st -5u)/4   (← st-9u≧0)
= (3√3){(ab^3+bc^3+ca^3)+(a^3b+b^3c+c^3a)+ 2[(ab)^2+(bc)^2+(ca)^2]}/(4s)
≧(3√3){(ab)^2+(bc)^2+(ca)^2)}/s,  (← AM-GM)

を示した方が簡単なおもしろい不等式…
0898132人目の素数さん
垢版 |
2017/09/06(水) 06:40:45.04ID:XFngCi/7
>>897
ゴクリ…。弄り甲斐のある不等式ですね。

2行目のAM-GMの使い方が分かりませぬ。
0899132人目の素数さん
垢版 |
2017/09/06(水) 06:47:49.58ID:XFngCi/7
すみません、わかりました。
それにしても、その形になるように変形しようという発想を知りたいですね。
0900132人目の素数さん
垢版 |
2017/09/06(水) 09:11:04.47ID:VtL80ANE
[問題]
nを2以上の自然数として
σ(n)をnの約数の総和、H_n:=農{k=1}^n 1/k とする
このとき
σ(n)<H_n+exp(H_n)log(H_n)
が成り立つことを示せ
0901132人目の素数さん
垢版 |
2017/09/06(水) 09:27:53.49ID:XFngCi/7
a, b, c >0 に対して、

(a+b+c)^2 (a^2 + b^2 + c^2)^3 ≧ 27 {(ab)^2 + (bc)^2 + (ca)^2}^2
(a+b+c)^2 (a^2 + b^2 + c^2)^3 ≧ 27abc (a^2 + b^2 + c^2) (a^3 + b^3 + c^3)
(a+b+c)^2 (a^2 + b^2 + c^2)^3 ≧ (a+b+c)^3 (ab+bc+ca) (a^3 + b^3 + c^3)

などが得られるが、残念ながら、右辺の上中下の3式の大小は定まらないでおじゃる。
0912132人目の素数さん
垢版 |
2017/09/06(水) 13:05:19.96ID:AYr/rfmQ
>>899
左辺の無理式
(ss/3 + …)^(3/2)
を有理式で評価するために使ったでござる。

(ab^3+bc^3+ca^3)、(a^3b+b^3c+c^3a)を経由せずに直接
(4sss-9st)- 27(tt-3su)/s =((4ss+7t) F_1 + 21u F_0 + su F_{-1})/ss ≧ 0
も簡単でつが、面白いので入れますた。

F_n(a,b,c)=(a^n)(a-b)(a-c)+(b^n)(b-c)(b-a)+(c^n)(c-a)(c-b)≧0,
0923132人目の素数さん
垢版 |
2017/09/07(木) 02:11:20.92ID:Fuvmh2la
>>901

ならば 0.03826828245292 ≦ k ≦ 16/27 のとき

(下)≧(1-k)*(中)+ k*(上),

はいかがでござる?
0924132人目の素数さん
垢版 |
2017/09/07(木) 05:11:20.81ID:+sD3y4UN
>>923
なるほど、その発想はなかったでござるよ、ニンともカンとも。

0.03826828245292 ≦ k ≦ 16/27 をみたす k の中で、
(1-k)*(中)+ k*(上) がきれいな形に整理できるものがあれば、いい不等式が作れますな。

その k の範囲はどうやって求めたのですか。
kのままで差を取って計算したのですか?
0925132人目の素数さん
垢版 |
2017/09/07(木) 05:11:48.93ID:+sD3y4UN
>>754
> (8)の類題 [第5章.698, 708]
> a, b, c >0 に対して、a^(b+c) + b^(c+a) + c^(a+b) ≧ 1

[疑問]
a^(2a) * b^(2b) * c^(2c) ≧ a^(b+c) * b^(c+a) * c^(a+b) は余裕で成り立つけど、
a^(2a) + b^(2b) + c^(2c) ≧ a^(b+c) + b^(c+a) + c^(a+b) は成り立つでござるか?

下の式がうまく証明できませぬ…

 ..::::::,、_,、::: ::::: ::: : 
  /ヨミ゙ヽ)-、. :: ::::
─(ノ─ヽ.ソ┴─
0926132人目の素数さん
垢版 |
2017/09/07(木) 05:26:00.67ID:+sD3y4UN
A,B,C,D>0 に対して、AB ≧ CD ⇒ A+B ≧ C+D は無条件では成り立たないから、
上の式を弄って、下の式を導くのは無理そう。
0927132人目の素数さん
垢版 |
2017/09/07(木) 06:49:03.06ID:+sD3y4UN
(2^a + 2^b)/2 ≧ √(2^a*2^b) = 2^{(a+b)/2} ≧ 2^{√(ab)}

巡回させて加えて、2^a + 2^b +2^c ≧ 2^{√(ab)} + 2^{√(bc)} + 2^{√(ca)}

( ゚∀゚) OK?
0928132人目の素数さん
垢版 |
2017/09/07(木) 07:12:08.52ID:+sD3y4UN
a, b, c >0 に対して、

2^(a^2) + 2^(b^2) + 2^(c^2)
≧ 2^(ab) + 2^(bc) + 2^(ca)
≧ 2^{a√(bc)} + 2^{b√(ca)} + 2^{c√(ab)}
≧ 2^{abc√(ab)} + 2^{abc√(bc)} + 2^{abc√(ca)}
≧ …
(以下無限に続く)

( 'A`) 自作の不等式といふものは、見栄えも悪いし、作成方法もバレバレよのぅ。 もう少し綺麗にならんものかな。
0930132人目の素数さん
垢版 |
2017/09/08(金) 03:00:35.95ID:Xvh/PpT+
>>925
上は対数とってチェビシェフで。
下はどうでおじゃる?


〔補題〕
a,b>0 のとき a^a + b^b ≧ a^b + b^a,

(略証)

・1≦a≦b のとき
 b^b ≧(b^a)a^(b-a),
(左辺)-(右辺)≧ a^a +(b^a)a^(b-a)- a^b - b^a
=(b^a - a^a)(a^b - a^a)/(a^a)
≧ 0,

・0<a≦1≦b のとき、ベルヌーイより、
(左辺)≦ 1 + ab ≦ a + b ≦(右辺),

・Max{1,a}≦b のとき
b^x ≧ a^x より
(左辺)-(右辺)=∫[a,b]{log(b) b^x - log(a) a^x}dx ≧ 0,


・0<a,b≦1 のとき、
う〜む。。。思ったよりめんどくせえ。


〔ベルヌーイの式〕
0<a,b≦1 のとき、
1-b+ab ≧ a^b ≧ a/(a+b-ab),
0<a≦1≦b のとき
1-b+ab ≦ a^b ≦ a/(a+b-ab),
0931132人目の素数さん
垢版 |
2017/09/08(金) 08:37:49.22ID:iwl1FmH8
Cauchyより、
{a^(2b) + b^(2c) + c^(2a)}*{a^(2c) + b^(2a) + c^(2b)} ≧ {a^(b+c) + b^(c+a) + c^(a+b)}^2

そこで、
{a^(2a) + b^(2b) + c^(2c)}^2 ≧ {a^(2b) + b^(2c) + c^(2a)}*{a^(2c) + b^(2a) + c^(2b)}  …(★)

が成り立てば解決と考えたけど、(★)が証明できない。
試しに b=c=1 を代入してみたらいけるので、成り立っているような感じだけど、ニンともカンとも…。
0932132人目の素数さん
垢版 |
2017/09/08(金) 08:44:00.05ID:iwl1FmH8
>>930
ベルヌーイの式はどうやって証明するのですか?

ベルヌーイの不等式
r≦0 or 1≦r のとき、(1+x)^r ≧ 1+rx
0≦r≦1 のとき、(1+x)^r ≧ 1+rx

とは別物ですか?
0934132人目の素数さん
垢版 |
2017/09/08(金) 12:59:14.60ID:Xvh/PpT+
>>931

>>930 より
a^(2a)+ b^(2b)≧ a^(2b)+ b^(2a),
巡回的にたして AM-GMする。
a^(2a)+ b^(2b)+ c^(2a)≧{a^(2b)+ b^(2c)+ c^(2a)}/2 +{a^(2c)+ b^(2a)+ c^(2b)}/2
≧ √{a^(2b)+ b^(2c)+ c^(2a)} √{a^(2c)+ b^(2a)+ c^(2b)} ……(★)
0935132人目の素数さん
垢版 |
2017/09/08(金) 14:38:39.14ID:iwl1FmH8
>>928
> ≧ 2^{abc√(ab)} + 2^{abc√(bc)} + 2^{abc√(ca)}

≧ 2^{√(abc√(ab))} + 2^{√(abc√(bc))} + 2^{√(abc√(ca))}
の間違いだな。
0936132人目の素数さん
垢版 |
2017/09/08(金) 14:38:44.92ID:Xvh/PpT+
>>930 >>934

〔補題〕
0<a≦b, 0<c≦d のとき
a^c + b^d ≧ a^d + b^c,

(略証)
m =(c+d)/2,h=(d-c)/2 > 0 とおく。
題意より、0 < a^m < b^m,0 < a^h < b^h,
よって
a^c - a^d - b^c + b^d
= a^(m-h)- a^(m+h)- b^(m-h)+ b^(m+h)
= a^m{a^(-h)- a^h}+ b^m{b^h - b^(-h)}
≧ a^m(b^h - a^h){1 +(ab)^(-h)}
≧ 0,
簡単だった...
0939132人目の素数さん
垢版 |
2017/09/08(金) 16:10:35.21ID:iwl1FmH8
検索したら…

面白スレ六問目 208 (出題のみ解答なし)
a, b >0 のとき、(a^b+b^a)/(a^a+b^b) のとりうる範囲を求めよ。
0940132人目の素数さん
垢版 |
2017/09/08(金) 16:24:51.04ID:iwl1FmH8
>>930
> >>925
> 上は対数とってチェビシェフで。

私は (a-b)(log a - log b) ≧0 を巡回させて加えて整理しますた。

チェビシェフって、具体的にどうやるんですか? きっと前スレも同じ方法。

> 正の数a,b,cに対して (a^b)(b^c)(c^a)≦(a^a)(b^b)(c^c) を示せ。
> 対数とってチェビシェフ
0941132人目の素数さん
垢版 |
2017/09/08(金) 18:03:30.03ID:iwl1FmH8
>>930
> 〔補題〕
> a,b>0 のとき a^a + b^b ≧ a^b + b^a,

この間からずっと探していて、先程手書きメモから発掘。そのメモによると、

 a,b,c,d>0 かつ ab≧cd かつ b = min{a,b,c,d} のとき、a+b ≧ c+d ……(☆)

 対称性から a≧b として、(a^a)(b^b) ≧ (a^b)(b^a) かつ a^a, a^b, b^a ≧ b^b で、(☆)を適用。

とだけ書きなぐってあった。例によって出典メモもなく、数学板の過去ログを検索してもヒットせず。
0942132人目の素数さん
垢版 |
2017/09/08(金) 22:02:47.27ID:iwl1FmH8
>>936と、第2章 466-467 より、
a, b >0 に対して、a^a + b^b ≧ a^b + b^a >1

第3章 109-111 より、
a, b, c >0 に対して、a^b + b^c + c^a >1

[疑問]
次式は成り立ちそうだけど、証明が分かりませぬ。
a^a + b^b + c^c ≧ a^b + b^c + c^a
0943132人目の素数さん
垢版 |
2017/09/08(金) 23:40:24.31ID:iwl1FmH8
>>940
もしかして、並べ替え不等式のことを言っているのかな?
 同順序積の和 ≧ 乱順序積の和 ≧ 逆順序積の和

チェビシェフは、
 同順序積の和の平均 ≧ 平均の積 ≧ Σ 乱順序積の和の平均
0944132人目の素数さん
垢版 |
2017/09/09(土) 00:56:44.14ID:fG3xA4Le
>>936
簡単ぢゃなかった......orz
0<a,b≦1 のときは?だった。

凡例 0<a<1/3,b=2a,c=1, d=2,(c/a = d/b ≧3)

大風呂敷 広げすぎたけど、 c/a = d/b ≦ e に限れば成り立つかも。

懲りずに作るでござる。

〔補題〕
0<a,b,0≦k≦e のとき
 a^(ka)+ b^(kb)≧ a^(kb)+ b^(ka),

>>941
 a≧b ⇒ a^a,b^a ≧ b^b が成立たないところが…
0945132人目の素数さん
垢版 |
2017/09/09(土) 07:38:47.20ID:PPAy6pZb
>>930
左側 (a^b + b^a)≦ 1 + ab はどうやって出すんですか?

 1 + ab = (1-b+ab) + b

と分けて、ベルヌーイを使うのかなと思ったら、

 a^b ≧ 1-b+ab
 b^a ≦ b

で不等号の向きが揃わない…
0946132人目の素数さん
垢版 |
2017/09/09(土) 09:14:36.51ID:PPAy6pZb
>>930
> ・0<a≦1≦b のとき、ベルヌーイより、
> (左辺)≦ 1 + ab ≦ a + b ≦(右辺),

ここですが、a^a ≧ a^b、b^a ≧ b^b だから、差をとれば終わりでは?

(a^a + b^b) - (a^b + b^a)
= (a^a - a^b) + (b^b - b^a)
≧0
0947132人目の素数さん
垢版 |
2017/09/09(土) 17:18:16.44ID:fG3xA4Le
>>946
その通りでつ。


>>783 に追加

a,b,c>0 に対して、
(aa+bb+cc)^3 ≧(aa+2bb)(bb+2cc)(cc+2aa)≧(aa+ab+bb)(bb+bc+cc)(cc+ca+aa)≧…

>>754 (1)(5)より
0949132人目の素数さん
垢版 |
2017/09/09(土) 18:15:23.39ID:PPAy6pZb
>>947
すまぬ、不等号の向きが逆でござる。
>>757の証明では、修正済みですね。

>>754 (1) 【訂正】
a, b, c >0 に対して、(ab+bc+ca)^3 ≦ (a^2 + 2b^2)(b^2 + 2c^2)(c^2 + 2a^2)
0951132人目の素数さん
垢版 |
2017/09/11(月) 02:33:18.51ID:Ls/z+whG
[第3章 843、845] より、

a≧b≧0,c≧d≧0のとき、

√(a^2+ad+d^2)+√(b^2+bc+c^2)≧√(a^2+ac+c^2)+√(b^2+bd+d^2)
0952132人目の素数さん
垢版 |
2017/09/11(月) 07:41:49.10ID:Ls/z+whG
>>951 の類題
[第1章 68、71] より、

実数x,y,zに対して √(x^2+y^2-xy)+√(y^2+z^2-yz) ≧ √(z^2+x^2+zx)
0954389
垢版 |
2017/09/11(月) 09:18:52.80ID:Bpls46N5
>>389の不等式について

元の問題(>>515)の2は、その対偶に当たる
∃k, ∀(x,y)>0, (x^v)(y^w)≦k((x^p)(y^q)+(x^r)(y^s)+(x^t)(y^u) ⇒ (Dが△ABCの内部および周上)
(>>389の←)
を示せばよい?

近大発表の解答を探したが、既刊の2冊には載っていなかった

『21世紀無差別級数学バトル』
https://www.amazon.co.jp/dp/4894714248
『白熱!無差別級数学バトル』
https://www.amazon.co.jp/dp/4535786720
0956132人目の素数さん
垢版 |
2017/09/11(月) 14:27:23.93ID:lLjA+cjN
>>952

3直線 OA、OB、OC を
 ∠AOB = ∠BOC = ∠AOC/2 = π/3,
となるようにとる。
OA上、座標xの点をX,
OB上、座標yの点をY,
OC上、座標zの点をZ とする。
このとき
 XY = √(xx-xy+yy),
 YZ = √(yy-yz+zz),
 ZX = √(zz+zx+xx),
 XY + YZ ≧ ZX,

等号成立条件は y(x+z)=xz.{x=z=2y も含む.}

>>953 ?
0959132人目の素数さん
垢版 |
2017/09/11(月) 16:19:08.50ID:Ls/z+whG
う〜ん、私が理解できていないだけみたい。

>>956
> OA上、座標xの点をX,

この意味が分かりません。
0960132人目の素数さん
垢版 |
2017/09/11(月) 16:28:39.37ID:Ls/z+whG
>>42
> 〔問題216〕
> 実数a〜dについて
> (aa+ac+cc) (bb+bd+dd)≧(3/4) (ab+bc+cd)^2,
> (aa+ac+cc) (bb+bd+dd)≧(3/4) (ad-bc)^2,

上側
4(a^2 + ac + c^2)(b^2 + bd + d^2) - 3(ab + bc + cd)^2
= (ab - bc + cd + 2da)^2
≧ 0

下側は、Wolfram 先生に以下の2通りを処理させても、ずっと 『COMPUTING』 のまま結果を出さない。

factor 4(a^2 + ac + c^2)(b^2 + bd + d^2) - 3(ad - bc)^2
expand 4(a^2 + ac + c^2)(b^2 + bd + d^2) - 3(ad - bc)^2

つまり因数分解できないんだろうけど、長い式は展開してくれないのかな?
平方和になるのかな?
0961132人目の素数さん
垢版 |
2017/09/11(月) 16:38:45.88ID:Ls/z+whG
手計算で展開してから、Wolfram先生に因数分解してもらった。

4(a^2 + ac + c^2)(b^2 + bd + d^2) - 3(ad - bc)^2
= 4(a^2b^2 + b^2c^2 + c^2d^2 + d^2a^2 + a^2bd + ab^2c + acd^2 + bc^2d + abcd) - 3(a^2d^2 - 2abcd + b^2c^2)
= 4a^2b^2 + b^2c^2 + 4c^2d^2 + d^2a^2 + 4a^2bd + 4ab^2c + 4acd^2 + 4bc^2d + 10abcd
= (2ab+ad+bc+2cd)^2
≧0
0962132人目の素数さん
垢版 |
2017/09/11(月) 17:20:43.44ID:IDWqxmZH
>>959
OAを,Oを原点とする座標軸みたいに考えて言ってる

要するに 直線OA=直線OXであって |OX|=x となるような点Xを取りなさいということ.
0963132人目の素数さん
垢版 |
2017/09/11(月) 17:22:26.50ID:lLjA+cjN
>>956
直線OAをx軸とし、OAの向きを正とします。
もちろん、x軸,y軸,z軸は直交しません(斜交軸)


>>960-961

>>47-48 から
(aa+ac+cc)(bb+bd+dd)=(ad-bc)^2 +(ad-bc)(ab+bc+cd)+(ab+bc+cd)^2,
これと
xx+xy+yy ≧(3/4)xx,(3/4)yy
から出ますけど...
0964132人目の素数さん
垢版 |
2017/09/11(月) 17:52:18.68ID:lLjA+cjN
>>952
では図に頼らず代数的に...

LHS^2 - RHS^2 = 2√(xx-xy+yy)√(yy-yz+zz)+(2yy-t)
={4(xx-xy+yy)(yy-yz+zz)-(2yy-t)^2}/{2√(xx-xy+yy)√(yy-yz+zz)-2yy+t}
= 3DD /{2√(xx-xy+yy)√(yy-yz+zz)-2yy+t}
≧ 0,
ここに、t = xy+yz+zx,
等号成立条件は D = xy+yz-zx = 0,
0965132人目の素数さん
垢版 |
2017/09/11(月) 18:32:41.88ID:Ls/z+whG
>>962-963
ありがとうございます! 今から考えてみます。


>>963
じゃあ xx+xy+yy ≧3xy だから、次式も成り立ちますね。
(aa+ac+cc) (bb+bd+dd)≧ 3(ad-bc)(ab+bc+cd)
0966132人目の素数さん
垢版 |
2017/09/11(月) 21:29:10.93ID:Ls/z+whG
>>956
たとえば x>0, y<0 のときに、
XY = √(xx-xy+yy) じゃなく
XY = √(xx+xy+yy) になりませんか?
0967132人目の素数さん
垢版 |
2017/09/11(月) 21:30:28.09ID:Ls/z+whG
いやいやいや、>>966は忘れてくだされ。負のときは角度が変わるから、大丈夫なんだね。
0968132人目の素数さん
垢版 |
2017/09/12(火) 02:14:22.13ID:YsdDbYfo
>>389 >>954

⇒ は簡単なんでつが… >>568

三角形を回して考えるのかな。
p’,r’,t’< v’ ならば x→∞
p’,r’,t’ > v’ ならば x→0
q’,s’,u’< w’ ならば y→∞
q’,s’,u’ > w’ ならば y→0
として反例を探す。
0969132人目の素数さん
垢版 |
2017/09/12(火) 03:54:04.96ID:YsdDbYfo
>>947

AM-GM で
(aa+2bb)(bb+2cc)(cc+2aa)-(aa+ab+bb)(bb+bc+cc)(cc+ca+aa)
={(aabb+c^4)/2 +2ccaa}(a-b)^2 +{(bbcc+a^4)/2 +2aabb}(b-c)^2 +{(ccaa+b^4)/2 +2bbcc}(c-a)^2 + 2abc
≧ 2ccaa(a-b)^2 + 2aabb(b-c)^2 + 2bbcc(c-a)^2 +2abc
= 2abc{(ca/b)(a-b)^2 +(ab/c)(b-c)^2 +(bc/a)(c-a)^2 + 凩
≧ 0,
ここに、 =(a-b)(b-c)(c-a),

〔補題〕
-1/2 < 凵^{(ca/b)(a-b)^2 +(ab/c)(b-c)^2 +(bc/a)(c-a)^2}≦(7-3√3)/22 = 0.0819930717

左側は(a,b,c)=(a,1,1/a)で a→∞ のとき近づく。
さて、どうやって示すんでしょうね...
0971132人目の素数さん
垢版 |
2017/09/12(火) 14:13:31.04ID:YsdDbYfo
>>969

AM-GMで
(aabb+c^4)/2(a-b)^2 +(bbcc+a^4)/2(b-c)^2 +(ccaa+b^4)/2(c-a)^2 + abc
≧ abc{c(a-b)^2 + a(b-c)^2 + b(c-a)^2 + 凩
= 2(abb+bcc+caa - 3abc)
≧ 0,     [第4章.626]
を使うと、
(aa+2bb)(bb+2cc)(cc+2aa)-(aa+ab+bb)(bb+bc+cc)(cc+ca+aa)
≧ abc{2(ca/b)(a-b)^2 + 2(ab/c)(b-c)^2 + 2(bc/a)(c-a)^2 + 凩,
0972132人目の素数さん
垢版 |
2017/09/12(火) 20:07:31.54ID:bmf0+g5o
【問題】 (出典 2016 TOT)
a, b, c >0 に対して、a + (ab)^(1/2) + (abc)^(1/3) ≦ (4/3)*(a+b+c)

TOTって何ぞや?

       ___ 
彡     /  ≧ \    彡 ビュゥ……
  彡   |:::  \ ./ |  彡
      |:::: (● (●|    書店で立ち読み中に
      ヽ::::......ワ...ノ    見かけた問題でござる
        人つゝ 人,,         
      Yノ人 ノ ノノゞ⌒〜ゞ    
    .  ノ /ミ|\、    ノノ ( 彡
     `⌒  .U~U`ヾ    丿
             ⌒〜⌒
0973132人目の素数さん
垢版 |
2017/09/12(火) 20:10:11.31ID:bmf0+g5o
【おまけ】 難易度:鼻くそ
a,b,c,d,e>0 に対して、a^2 + b^2 + c^2 + d^2 + e^2 ≧ (a+b+c+d)e
0974132人目の素数さん
垢版 |
2017/09/12(火) 23:02:36.39ID:bmf0+g5o
>>972 を改造しようとして、λの最小値を出そうとしたが、挫折したでござる。

a, b, c, d >0 に対して、a + (ab)^(1/2) + (abc)^(1/3) + (abcd)^(1/4) ≦ λ*(a+b+c+d)
0976132人目の素数さん
垢版 |
2017/09/12(火) 23:30:54.46ID:bmf0+g5o
>>972
2文字なら簡単に作れるのでおじゃるが…

a, b >0 に対して、a + (ab)^(1/2) ≦ {(1+√2)/2}*(a+b)
0977132人目の素数さん
垢版 |
2017/09/13(水) 03:07:54.73ID:i1anpb+k
[疑問]-----------------------------------------------
a, b, c >0 に対して、
M(a,b,c) ≧ (a^2+ab+b^2)(b^2+bc+c^2)(c^2+ca+a^2) ≧ m(a,b,c)
-----------------------------------------------------

AM-GMで m(a,b,c) = 27(abc)^2 を得るけど、もっとキツく締め上げたいのでござる。

L = a^2b + b^2c + c^2a
R = ab^2 + bc^2 + ca^2

(a^2+ab+b^2)(b^2+bc+c^2)(c^2+ca+a^2)
= L^2 + LR + R^2
= (s^2)(t^2) - (s^3)u - t^3



 " ;ヾ ; ;";ヾ; ;"/" ; ;ヾ ;ヾ;ヾ ; ;ヾ ; ; ヾ ;ゞ  " ;ヾ ; ;";ヾゝゝ" ;ヾゞ           ヽ            /
,." ;ヾ ; ;";ヾ; ;"/" ; ;ヾ ;ヾ;ヾ ; ;ヾ ; ; ヾ ;ゞ  " ;ヾ ; ;";ヾゝゝ" ;ヾ ; ; ヾ ;ゞ;        \        /
 ゞヾ ; ;" ; ; ;; ;"iiiiii;;;;;::::: :)_/ヽ,.ゞ:,,ヾゞヾゞ__;::/        `      `        `   ー ─ ' `
   ゞヾゞ;\\iiiiii;;;;::::: :|;:/ヾ; ;ゞ "ゝゞ ; ;`
  `      ,|i;iiiiiii;;;;;;::: :| `    `         `     `      ` `   `
        ,|iiii;iiii;;;;:;_ _: :| ___  秋の夜長に不等式    `        `        `,
   `    |iiiiiii;;;;;;((,,,):::|/  ≧ \                    ヾ从//"
    `   |iiiiiiii;;;;ii;;;;;;;;::|::::: (● (● |           `  ゙  `    ヾ'./"
         |iiiiii;iii;;;;i;;:: ::::|ヽ::::......ワ...ノ                 ○     .||.       ,
    `   |iii;;iiiii;::;:;;;;::::::| ( つ且 ~      `              ○○   | |
  , , .,.. ,..M|M|iMii;;ii:i;;i:i;:; ゝ つつ.,.. ,...... ,.... ,,,.,.. ,.... ,,,.,.. ,..,,,,.,...,..,.,| ̄ ̄|,.,..(  ).. ,,,..,,.. ,.... ,,,.,...,.. .. ,.... ,,,.,.. ,.... ,,,
0978132人目の素数さん
垢版 |
2017/09/13(水) 06:13:10.00ID:HyiuMNX2
耳栓をしたら世界が変わってワロタ
0979132人目の素数さん
垢版 |
2017/09/13(水) 07:02:09.13ID:jekxCsX+
>>974
 a = a,
 √ab ≦{1/(2√p)}(a+pb),
(abc)^(1/3)≦{1/[3(pq)^(1/3)]}(a+pb+qc),
(abcd)^(1/4)≦{1/[4(pqr)^(1/4)]}(a+pb+qc+rd),
ここに、
p = 3.37617521979458
q = 9.55342152751350
r = 32.2851876698453

辺々たすと
λ = 1.42084438540961
0980132人目の素数さん
垢版 |
2017/09/13(水) 10:07:15.32ID:i1anpb+k
>>975
顔文字(ToT)の正体は Tournament of the town なのか…
幾つかの国でやっているようだから、出題年度だけでは見つけるのは大変でござるな。

wiki (Tournament of the town)
https://en.wikipedia.org/wiki/Tournament_of_the_Towns
AoPS
https://artofproblemsolving.com/community/c3239_tournament_of_towns

加奈陀
http://www.math.toronto.edu/oz/turgor/archives.php
独逸
http://www.math.uni-hamburg.de/stw/problems.html
仏蘭西
http://www.tournoidesvilles.fr/archives.html
以色列
http://www.taharut.org/

イスラエルは読めぬ…。右寄せになっているが右から左に書くのか?
0981132人目の素数さん
垢版 |
2017/09/13(水) 10:15:37.91ID:i1anpb+k
>>979
3変数でよかったのか…。次のように6変数でやっていますた。

a = a
√ab = √{(pa)(b/p)} ≦ {(pa)+(b/p)}/2
(abc)^(1/3) = {(qa)(rb)(c/pq))}^(1/3) ≦ {(qa)+(rb)+(c/pq)}/3
(abcd)^(1/4) = {(sa)(tb)(uc)(d/stu)}^(1/4) ≦ {(sa)+(tb)+(uc)+(d/stu)}/4

1 + p/2 + q/3 + s/4 = 1/2p + 3/r + t/4 = 1/3pq + u/4 = 1/4stu
pa = b/p
qa = rb = c/pq
sa = tb = uc = d/stu
0982132人目の素数さん
垢版 |
2017/09/13(水) 10:36:59.57ID:i1anpb+k
>>4
埋蔵地のリンクが切れているところが結構あるので修正中。
>>1の過去ログ・まとめサイト、>>2の和書以外は、まとめサイト参照でいいかもね。
0983132人目の素数さん
垢版 |
2017/09/13(水) 10:49:00.52ID:i1anpb+k
>>165
[不等式 第7章 241]
> 0<x<y<π/2の時
> (tanx/x)^x+(siny/y)^y<(tany/y)^y+(sinx/x)^x
> を示せ

これも未解決ですな
0984132人目の素数さん
垢版 |
2017/09/13(水) 10:49:48.18ID:i1anpb+k
>>469
> >>388
> >>456
> 相当な量の改良問題があった
>
> for reals
> [1] (a^2+1)(b^2+1)(c^2+1) >= (1+a+b)(1+b+c)(1+c+a)
> [2] ((a^2+3)(b^2+3)(c^2+3))^2 >= 512(a+b)(b+c)(c+a)
>
> for nonnegarives
> [3] (a^2+2)(b^2+2)(c^2+2) >= 3(a+b+c)^2+(abc-1)^2
> [4] (x^2+2)(y^2+2)(z^2+2) >= 4(x^2+y^2+z^2)+5(xy+yz+zx)+(xyz-1)^2
> [5] (a^2+2)(b^2+2)(c^2+2) >= 4(a^2+b^2+c^2)+5(ab+bc+ca)+(abc(a-1)^2(b-1)^2(c-1)^2)^(1/3)
>
> AOPS
> [1], [2] : c6h588096p3481394
> [3] : c6h4830p15309
> [4], [5] : c6h581954p3438879
>
> 他にもいろいろ

この辺に改造できそうなネタがたくさん埋もれていそう。
0985132人目の素数さん
垢版 |
2017/09/13(水) 11:15:48.43ID:i1anpb+k
数研通信に SMV-Theorem についての解説があった。

数検通信
http://www.chart.co.jp/subject/sugaku/suken_tsushin.html

89号、対称的な不等式の証明方法について、柳田五夫 ← コレ

他に不等式絡みの記事

80号、3次の同次対称式P(a,b,c)の不等式について、柳田五夫
76号、絶対値記号を含む不等式について、柳田五夫
75号、不等式の証明に役立つ不等式と接線の利用について、柳田五夫
66号、1/a + 1/b + 1/c + 1/d + 1/e (a,b,c,d,e∈N)の最大値について、柳田五夫
60号、接線を利用した台形の面積で,ある不等式を証明する、柳田五夫
08号、ある不等式の証明について、柳田五夫

89号、数学的帰納法とベルヌーイの不等式、大谷昌範
85号、モローの不等式の証明、藤岡優太
80号、n数の相加・相乗平均の関係の証明、西元教善
76号、ベクトルの三角不等式の活用、岡本雅史
66号、チェバ・メネラウスの定理から導く三角形の不等式、中村公一
60号、巡回不等式特集、大塚秀幸
50号、不等式をつくる、仁平政一
42号、いままで出会ったことのない「ある不等式」について、仁平政一
49号、相加・相乗平均の不等式を産み出す根源的不等式について 、西元教善
47号、不等式の証明の統一的方法、仁平政一
20号、チェビシェフの不等式について、遠藤一成、中島政彦
0988132人目の素数さん
垢版 |
2017/09/13(水) 14:14:15.00ID:HyiuMNX2
耳栓をしたら世界が変わってワロタ
0989132人目の素数さん
垢版 |
2017/09/13(水) 16:12:00.85ID:i1anpb+k
>>979
1 + 1/(2√p) + 1/[3(pq)^(1/3)] + 1/[4(pqr)^(1/4)]
= p/(2√p) + p/[3(pq)^(1/3)] + p/[4(pqr)^(1/4)]
= q/[3(pq)^(1/3)] + q/[4(pqr)^(1/4)]
= r/[4(pqr)^(1/4)]

をみたす正の数 p, q, r を求めればいいんだけど、うまく出せない…

p = 3.37617521979458
q = 9.55342152751350
r = 32.2851876698453

この値はどうやったら出せるんですか?
0990132人目の素数さん
垢版 |
2017/09/13(水) 17:42:14.45ID:jekxCsX+
>>981

>>974 の等号が a=pb=qc=rd で成立するならば、
このとき >>981 の3式も等号が成立するはず。

これを考慮すると、
a=A、pb=B、qc=C、rd=D とおくとき
 √AB ≦(A+B)/2,
(ABC)^(1/3)≦(A+B+C)/3,
(ABCD)^(1/4)≦(A+B+C+D)/4,
の定数倍になっている。

>>987-989
それは拙者も知りとうござる。

ところで、
λ_1 = 1.0
λ_2 =(1+√2)/2 = 1.20710678118655  >>976
λ_3 = 4/3 = 1.33333333333333     >>972
λ_4 = 1.42084438540961         >>979
単調に増加する....
lim_{n→∞}λ_n = ?
0991132人目の素数さん
垢版 |
2017/09/13(水) 19:04:41.16ID:i1anpb+k
>>977
とりあえず少し進展したのでパピコ。 Caushyの拡張より、

(a^2+ab+b^2)(b^2+bc+c^2)(c^2+ca+a^2)
= (ab+b^2+a^2)(b^2+bc+c^2)(a^2+c^2+ca)
≧ (ab+bc+ca)^3
= t^3

AM-GMで 27(abc)^2 = 27u^2 としたよりもマシになった。

 m(a,b,c) = (ab+bc+ca)^3 ≧ 27(abc)^2

が、以下のように分割すると、非負値の和ばかりで、ずいぶんとゆるゆるなうんちでござる。

(a^2+ab+b^2)(b^2+bc+c^2)(c^2+ca+a^2) - t^3
= (s^2)(t^2) - (s^3)u - 2(t^3)
= (t^2-3su)F_0 + 2suF_0 + (u^2)F_{-2} + u(st-9u)
≧ 0

まだまだ厳しくできるはず!
ちなみに M(a,b,c) の方は、どこから手をつけてよいか見当がつかぬ…。

 
    /⌒ヽ
  /⌒  ・ > ぬ〜ん…
  E ̄U) ε | 
  E ̄∩) ・ >
゛゛゛゛゛゛゛゛゛゛゛゛゛゛゛゛゛゛゛゛゛゛
0993132人目の素数さん
垢版 |
2017/09/13(水) 19:11:18.22ID:i1anpb+k
どうでもいいが、新スレ3のAAの元ネタは、「よろしい ならば戦争だ」

ニコ動で演説は見たが、元のアニメを見たことがなくてピンとこない。
0994132人目の素数さん
垢版 |
2017/09/13(水) 19:30:18.62ID:jekxCsX+
>>984
[1]
a=b=c=1 のとき?


[2]
(aa+3)(bb+3)=(ab-1)^2 +(3aa+2ab+3bb)+ 8
=(ab-1)^2 + (a-b)^2 + 2(a+b)^2 + 8
=(ab-1)^2 + (a-b)^2 + 2(a+b-2)^2 +8(a+b),
0995132人目の素数さん
垢版 |
2017/09/13(水) 19:36:01.04ID:i1anpb+k
>>994
たしかに (1) は成り立ちませんね。
AoPSの掲示板が元ネタだから、仕方ないでござる。
0996132人目の素数さん
垢版 |
2017/09/13(水) 19:56:41.83ID:S+/ABgCb
>>994
for positives
[1] (a^2+2)(b^2+2)(c^2+2) >= (1+a+b)(1+b+c)(1+c+a)
だった
なんで全然違う問題を掲載したんだろう

>>995
それはどうだろう
少なくともやばい連中はそこにたくさんいる
0998132人目の素数さん
垢版 |
2017/09/13(水) 23:06:15.42ID:i1anpb+k
 "; ;ヾ; ;ヾ; ;メヾ "ゞ ;ヾ ;ゞ ;" "ゞ ; ; ; ゞ ;" "ゞ";ヾ ; ヾ ;ゞ; ;ゞ ;ゞ ;" "ゞ        /.             ヽ
 ;" "ゞ ; ; ; ゞ ; ;ヾ ; ; ヾ ;ゞ;ヾ ; ;";ヾ; ;"/" ; ;ヾ ;ヾ; ヾ ; ヾ ;ゞ; ;ゞ ;" ";ゞ ; ;ヾ      l             l
 " ;ヾ ; ;";ヾ; ;"/" ; ;ヾ ;ヾ;ヾ ; ;ヾ ; ; ヾ ;ゞ  " ;ヾ ; ;";ヾゝゝ" ;ヾゞ           ヽ            /
,." ;ヾ ; ;";ヾ; ;"/" ; ;ヾ ;ヾ;ヾ ; ;ヾ ; ; ヾ ;ゞ  " ;ヾ ; ;";ヾゝゝ" ;ヾ ; ; ヾ ;ゞ;        \        /
 ゞヾ ; ;" ; ; ;; ;"iiiiii;;;;;::::: :)_/ヽ,.ゞ:,,ヾゞヾゞ__;::/        `      `        `   ー ─ ' `
   ゞヾゞ;\\iiiiii;;;;::::: :|;:/ヾ; ;ゞ "ゝゞ ; ;`
 " ;゛ ; ;" ; ;ゞ "|iiiiii;;;;::: : |:/ ヾゞ        `         `      ` `
  `      ,|i;iiiiiii;;;;;;::: :| `    `         `     `      ` `   `
        ,|iiii;iiii;;;;:;_ _: :| ___  秋の夜長に不等式    `        `        `,
   `    |iiiiiii;;;;;;((,,,):::|/  ≧ \                    ヾ从//"
    `   |iiiiiiii;;;;ii;;;;;;;;::|::::: (● (● |           `  ゙  `    ヾ'./"
         |iiiiii;iii;;;;i;;:: ::::|ヽ::::......ワ...ノ                 ○     .||.       ,
    `   |iii;;iiiii;::;:;;;;::::::| ( つ且 ~      `              ○○   | |
  , , .,.. ,..M|M|iMii;;ii:i;;i:i;:; ゝ つつ.,.. ,...... ,.... ,,,.,.. ,.... ,,,.,.. ,..,,,,.,...,..,.,| ̄ ̄|,.,..(  ).. ,,,..,,.. ,.... ,,,.,...,.. .. ,.... ,,,.,.. ,.... ,,,
0999132人目の素数さん
垢版 |
2017/09/13(水) 23:06:54.49ID:i1anpb+k
_| ::|_
 ̄| ::|/|           ┌──┐
  | ::|  |     .┌──┐| ∧_∧  いいな、俺たちの誰かが殉職したら・・
/|_|  |┌──┐| ∧_∧|(・ω・` )
  |文|  | | ∧_∧(    )⊂   )
  | ̄|  | | (    )⊂   ) (_Ο Ο :::
  | ::|  | | ⊂   ) (_Ο Ο わかってる、生き延びた奴が
  | ::|/ .|_ (_Ο Ο ::::::::: :::::: 不等式を収集し、証明する !
  | ::| :::::::::::::::::::::::::::::::: 俺たちゃ死んでも仲間だぜ !!
10011001
垢版 |
Over 1000Thread
このスレッドは1000を超えました。
もう書けないので、新しいスレッドを立ててくださいです。。。
life time: 80日 5時間 46分 15秒
10021002
垢版 |
Over 1000Thread
2ちゃんねるの運営はプレミアム会員の皆さまに支えられています。
運営にご協力お願いいたします。


───────────────────
《プレミアム会員の主な特典》
★ 2ちゃんねる専用ブラウザからの広告除去
★ 2ちゃんねるの過去ログを取得
★ 書き込み規制の緩和
───────────────────

会員登録には個人情報は一切必要ありません。
月300円から匿名でご購入いただけます。

▼ プレミアム会員登録はこちら ▼
https://premium.2ch.net/

▼ 浪人ログインはこちら ▼
https://login.2ch.net/login.php
レス数が1000を超えています。これ以上書き込みはできません。

ニューススポーツなんでも実況